Mental State/Intellectual Function/Behavioural Problems+Major Psychiatric Disorders/Drug & Alcohol Abuse Flashcards

1
Q

A 20-year-old girl presents with history of episodes of trembling, palpitation, shortness of breath, and sweating. Each episode lasts for 5 minutes and then gradually resolves. She has no history of heart disease, and is not on any regular medication. She denies any use of illicit drugs. Which one of the following would be the best course of action for management?

A. Reassurance, explanation, and support.
B. Selective serotonin reuptake inhibitors.
C. Breathing in and out of a paper bag when attacks develop.
D. Tricyclic antidepressants.
E. Referral to a psychiatrist.

A

Correct Answer Is A.

This patient has typical presentation of panic attacks.
Panic attack is characterized by the presence of at least 4 of the following features:
* Palpitations, pounding heart, or accelerated heart rate
* Sweating
* Trembling or shaking
* Sense of shortness of breath or smothering
* Feeling of choking
* Chest pain or discomfort
* Nausea or abdominal distress
* Feeling dizzy, unsteady, lightheaded, or faint
* Derealization or depersonalization (feeling detached from oneself)
* Fear of losing control or going crazy
* Fear of dying
* Numbness or tingling sensations
* Chills or hot flushes

The cornerstone of treating panic attacks is explaining the condition to the patient, reassurance and supporting the patient to understand the situation. For residual anxiety following a panic attack
benzodiazepines might be considered.

(Options B and D) Selective serotonin re-uptake inhibitors (SSRIs) and tricyclic antidepressants (TCAs) have been used as pharmacological treatment of panic disorder (not panic attacks) as complements to nonpharmacological management (e.g. CBT), which is first-line treatment of panic disorder.

(Option C) Breathing in and out of paper bag is helpful if patient is hyperventilating during a panic
attack. It can provide relief in short time and can be taught to the patient after explanation and reassurance.
(Option E) Referring to a psychiatrist is not needed at this stage because the diagnosis is completely straightforward and clear.

How well did you know this?
1
Not at all
2
3
4
5
Perfectly
2
Q

An 82-year-old man male presents to the emergency department with a knife in his hand. He claims that he hears voices saying his neighbors want to kill him and he should kill them before they kill him. You, as the attending physician, manage to verbally de-escalate him and convince him to surrender his knife. Which one of the following would be the next best step in management?

A. Call the hospital security.
B. Call the police.
C. Call his family to help you with dealing with him.
D. Calm him down and talk to him.
E. Offer him tea and biscuit.

A

Correct Answer Is A.

The presentation is typical of acute psychosis and command hallucinations. Any patient with command hallucinations to harm self or others is at significant risk of developing violence sooner or later.

Although apparently this patient is ready to surrender his knife now, with command hallucinations he may still pose risk to himself or others.

Under circumstances such as this one, calling the hospital security would be the next best step in
management. This will ensure the safety of you and the staff while you are planning further measures.

(Option B) Calling the police was an appropriate option if this situation happened in the community
and not in a medical facility.

(Option C) Calling his family will provide an opportunity to obtain more information about his
medical and psychiatric history; however, security and safety comes first.

(Options D and E) Verbal de-escalation to calm the patient or measures to establish rapport such as offering tea and biscuits is appropriate after safety is ensured by the hospital security.

How well did you know this?
1
Not at all
2
3
4
5
Perfectly
3
Q

John, 24 years, had a diagnosis of depression and was started on sertraline (Zoloft) 50 mg/day, 23 days ago, and was given a mental health care plan to see a psychologist for cognitive behavioral therapy (CBT). He is in your office today for follow-up, and believes there has been no improvement whatsoever. Which one of the following is the most appropriate** next step** in management?

A. Continue the same dose of sertraline.
B. Increase the dose of sertraline.
C. Switch to another SSRI.
D. Switch to an SNRI.
E. Add a different antidepressant medication to sertraline.

A

Correct Answer Is A.

Selective serotonin reuptake inhibitors (SSRIs) and tricyclic antidepressants (TCAs) are the two most prescribed medications for major depression.

Other major drug classes include serotonin norepinephrine reuptake inhibitors (SNRIs), (e.g.,
venlafaxine), reversible monoamine oxidase inhibitors (e.g., moclobemide), and non-selective monoamine oxidase inhibitors (e.g., phenelzine).

SSRIs have safer adverse effect profile and less likely to cause dropout compared to TCAs.

Antidepressant are started at initial dose and gradually and incrementally increased in dosed until
therapeutic doses are reached. For most SSRIs the initial dose is the same as therapeutic dose. As such, 50mg per day is both the initial and therapeutic dose for sertraline. The therapeutic response to therapeutic dose is often delayed for 1-2 weeks, and as a rule of thumb, all antidepressants should be trialed for at least 4-6 weeks after the therapeutic dose is reached before any change in treatment is considered.

John has been started on sertraline 20 days ago and it is still early to consider a change to treatment such as increasing the dose of sertraline (option B), switching to another SSRI (option C) or an SNRI (option D), unless for adverse effects issues which feel bothersome to the patient. John should be advised to continue the same dose for at least to 6 weeks under close monitoring for any complications (e.g., adverse effects, suicidality, etc.)

Using two antidepressants together is NOT recommended. Therefore, addition of another
antidepressant to sertraline (option E) is not a correct option.

How well did you know this?
1
Not at all
2
3
4
5
Perfectly
4
Q

Janet presents to you concerned about her 15-year-old son, Joey, after she found out he had been
wearing his older sister’s underwear. She is a single parent as her husband left them after Joey was born. She works two jobs and has been under a lot of stress and has not had enough time to spend with her kids. She is quite frustrated and distressed with Joey’s behaviour and believes there is something seriously wrong with him and wishes she has not given birth to him at all. Which of the following is the correct statement to share with Janet?

A. Joey may feel as if he is a girl trapped in a boy’s body.
B. He is most likely doing it for fun.
C. Not having a father figure in his life is the reason for this behavior.
D. Janet’s lifestyle may be the main reason for Joey’s behavior.
E. It is a part of normal development at this age.

A

Correct Answer Is A.

Joey’s interest in wearing clothing suggests differential diagnoses such as cross dressing, transvestic disorder, gender dysphoria, and transgenderism. All these diagnoses should be born in mind, discussed with Janet, and assessed thoroughly.

It is normal for children and teenagers to experiment with gender. For example, a girl might refuse to wear skirts or dresses, or a boy wants to play ‘mum’. For most children and teenagers, experimenting with gender does not mean they are gender diverse or transgender. Most children go on to feel comfortable with their birth gender. In contrast, gender dysphoria in children and adolescents is not a phase. Gender dysphoria is when one feels distressed because their gender identity differs from the birth sex. This distress might affect their school or home life. Those with gender dysphoria feel like they are trapped in a body of the opposite sex. It should be explained to
Janet while going over potential diagnoses and how one would feel if they were gender dysphoric or transgender.

Janet is quite worried about her son and his behavior; therefore, telling her that he is most likely doing this for fun (Option B) would not be reassuring to her, especially when you do not have all the facts to draw such conclusion. It would be more prudent to counsel her and explain to her Joey may be experiencing and encourage her to bring him in for counselling if he wants it. Personal counselling and family insight therapy should be recommended.
There is not enough information in the scenario to suggest that not having a father figure in his life
(option C) or Janet’s lifestyle (option D) could be the reason for Joey’s manner. Although both
these reasons are disruptive to the normal functioning of their family life, Joey does not exhibit any other characteristics or behaviors to suggest that he is acting out in response to these disruptions.

By about the age 6 or 7 years, children begin to understand that sex is permanent across situations
and over time. Once they develop this understanding, they begin to act as members of their sex. Therefore, it would be inaccurate to tell Janet that Joey’s behavior is a part of normal development (Option E) at the age 15.

TOPIC REVIEW

Cross-dressing/ transvestitism
Cross-dressing is defined as typically heterosexual men wearing women’s clothing. There is no correlation between crossdressing and transgenderism or homosexuality. Cross dressers choose to dress as women only some of the time and enjoy experiencing both the masculine and feminine
parts of themselves. As opposed to transvestitism, a transgender person lives fulltime in the gender that they identify with.

DSM-5 considers cross-dressing as a psychiatric disorder (transvestic disorder) if cross-dressing
or thoughts of cross-dressing are always or often accompanied by sexual excitement. The main difference between cross-dressing and transvestitism is that the latter is associated with sexual arousal.

Transgenderism
Children whose gender identity differs from the gender they were assigned at birth are known as
transgender or gender diverse. Transgender is a non-medical term describing individuals whose gender identity (inner sense of gender) or gender expression (outward performance of gender) differs from the sex or gender to which they were assigned at birth. Not all transgender people suffer from gender dysphoria and that distinction is important to keep in mind. Gender dysphoria and/or coming out as transgender can occur at any age.

People who are transgender may pursue multiple domains of gender affirmation, including social
affirmation (e.g., changing one’s name and pronouns), legal affirmation (e.g., changing gender markers on one’s government-issued documents), medical affirmation (e.g., pubertal suppression or gender-affirming hormones), and/or surgical affirmation (e.g., vaginoplasty, facial feminization surgery, breast augmentation, masculine chest reconstruction, etc.). Of note, not all people who are transgender will desire all domains of gender affirmation, as these are highly personal and individual decisions.

How well did you know this?
1
Not at all
2
3
4
5
Perfectly
5
Q

Eve is 45 years old and has presented to your clinic for consultation in tears. Her only son died 6 weeks ago from a car accident, and since then she has been having difficulty sleeping and terrible headaches. She does not enjoy doing her life routines anymore and does not feel like eating.

Further probing reveals that she hears her son speaking to her and calling her name and sometimes she feels his presence in her bedroom at night. She wants something to help her with her sleep. Which one of the following is the most appropriate action in this situation?

A. Referral to hospital for admission, assessment, and treatment.
B. Referral to a psychiatrist.
C. A short course of benzodiazepines and referral for grief counselling.
D. A short course of selective serotonin re-uptake inhibitors (SSRI) and referral for grief
counselling.
E. A short course of antipsychotics and referral for grief counselling.

A

Correct Answer Is C.

Given the symptoms and their duration, a normal grief reaction is the most likely explanation to this woman’s problem. Janet is grieving for her deceased son, and this has affected her in many ways, including a disruption of her daily routine, weight loss and decreased sleep and appetite. Patients with normal grief reaction return to normal social functioning within 2 months. The symptoms however might last up to one year with waxing and waning.

Eve has come to your clinic seeking help; in particular, she wants something to help her with her sleep. In the absence of any red flags (suicidality, drug seeking behaviour, etc.), the most appropriate action would be to prescribe a short course of benzodiazepines (< 7 days) for her insomnia and referral to grief counselling to help her cope with her loss.

Sadness, despair, tearfulness, decreased sleep, decreased appetite and decreased interest in life
and the world, are some of the common findings in normal grief reaction. While guilt and shame are not common in normal grief reaction, they are still possible. Suicidality is not usually a concern unless the patient has said something indicative of her intent and plans to do so which is not the case here.

Eve might require referral to psychiatrist (option B) down the line if grief counselling does not
resolve the issue and symptoms persist. It is not necessary for now.

SSRIs would be indicated if Eve had a diagnosis of moderate to severe depression. Eve does not fulfill criteria for a diagnosis of major depression. While referral for grief counselling is necessary, SSRIs are not indicated neither for depressive symptoms nor for the sleep issue.

Simple visual and auditory hallucinations of the deceased person are common and may lead the bereaved person to fear he/she is losing their mind. These are a result of her grief reaction, and not new onset psychosis. Similarly, Eve is hearing her son speaking to her and calling her name and sometimes, she feels his presence in her bedroom at night. This could be considered normal in grief as long as there are no other psychotic features necessitating antipsychotic treatment. Eve does not have psychotic illness and antipsychotics (option E) would not be indicated.

How well did you know this?
1
Not at all
2
3
4
5
Perfectly
6
Q

A middle-age famer from Queensland presents with symptoms of poor sleep, lack of concentration and energy, decreased appetite, and low mood for the past few weeks. He believes that all these started after the drought and blames the government for being reckless and irresponsible. When you mention that his symptoms are of depressive nature, he objects it and says he is just exhausted and not depressed. Which one the following defense mechanisms is he using?

A. Displacement.
B. Projection.
C. Denial.
D. Rationalization.
E. Reaction formation.

A

Correct Answer Is C.

This patient is experiencing symptoms of depression (poor sleep, lack of concentration and energy, decreased appetite, and low mood) brought on by his recent losses after the drought. As the doctor attempts to explain to him that his symptoms are of a depressive nature, he refuses to accept it and insists that his symptoms are a result of exhaustion and not depression. Based on the facts in the scenario, this patient is most likely utilizing denial as a defense mechanism. Denial is a form of psychotic defense mechanism evident by the replacement of external reality with wishful fantasy (behaving as if an aspect of reality does not exist).

When counselled by the doctor, he said he blamed the government for being reckless and irresponsible. This is an example of projection (Option B), another form of a defense mechanism.

Projection refers to the interpretation of internal impulses as though they are outside oneself (attributing one’s own feelings to others). In this scenario, the drought could have been a cause for his symptoms as it might have affected his livelihood as a farmer, but the patient places blame on the government without accepting his share of responsibility to have taken adequate safety
measures in case of natural causes. Displacing his feelings helps him subconsciously place the root cause of this feeling onto someone or something else. In other words, he is projecting the negative feelings of recklessness and irresponsibility elsewhere to protect his ego. However, this question refers to the patient’s reaction to being informed that his symptoms are of a depressive nature. Therefore, projection (option B) is not the correct option.
Displacement (option A), another defense mechanism and occurs when a person represses
affection, fear, or impulses that they feel towards another person as they believe it is irrational or socially unacceptable to demonstrate such feelings; therefore, these feelings are displaced toward another person or thing.

Rationalization (option D) defense mechanism involves using rational explanations to justify behaviors that are unacceptable (justifying behaviour to avoid difficult truths). In doing so, they avoid accepting the true cause or reason resulting in the present situation. This patient would be using rationalization if, for example, he said: “of course I am depressed. The drought has destroyed all my crops and left me with losses”. In this scenario, however, he is denying his depressive symptoms altogether.

In reaction formation (option E) an individual expresses the opposite of their true feelings, sometimes to an exaggerated extent. It is an intentional effort to compensate for conscious dislikes. For example, if he felt that his self-esteem was threatened by being diagnosed with depression, he would have acted overly aggressive. This is not the case here, and therefore reaction formation is not the correct option.

How well did you know this?
1
Not at all
2
3
4
5
Perfectly
7
Q

Ali, 45 years old is in your office with his wife who is concerned about him and insisted that he sees a doctor because he seems to have been ‘lost’ and ‘confused’ since he had a severe car accident and lost one of his best friends who was in the car with him 5 months ago. He only sustained a head injury in the accident which was cleared as a minor one with no serious complications at that time. Which of the following could be the most likely cause to this presentation?

A. Major depressive disorder.
B. Post-traumatic stress disorder.
C. Post-concussion syndrome.
D. Late-onset schizophrenia.
E. Dementia.

A

Correct Answer Is C.

This Scenario represents a common undiagnosed and mistreated condition seen in general practice after a head trauma, namely post-concussion syndrome (PCS).

Concussion is a temporary disturbance in brain function following a trauma to the head. It can also
occur after a blow to the body. Concussion can present with a variety of signs and symptoms
including:
* Cognitive related symptoms:
* Difficulty concentrating
* Difficulty finding things
* Difficulty reading
* Memory problems
* Brain fog
* Easily distracted
* Mood-related symptoms:
* Anxiety
* Depression
* Feeling overwhelmed
* Irritability
* Low-energy or motivation
* Various other mood/personality changes
* Sensory-related symptoms:
* Blurred vision
* Car sickness or nausea with motion
* Change in (or loss of) taste or smell
* Ringing ears
* Blood dysregulation symptoms:
* Headache
* Fatigue
* Nausea
* Dizziness
* Sensitivity to light and noise
* Sleep problems
* Persistent neck pain
* Pressure in the head
* Tried eyes

Almost 90% of patients with concussive symptoms experience a quick recovery within few days to weeks. Those in whom symptoms persist beyond 3 moths are defined as having post-concussion syndrome (PCS).

Since Ali has been in a car accident resulting in his friend’s death, post-traumatic stress disorder (PTSD) (option B) should be considered as well. Symptoms may include flashbacks, nightmares, and severe anxiety, as well as uncontrollable thoughts about the event. Symptoms should persist for at least 1 month before a diagnosis of PTSD is made. The DSM-5 has a more expansive set of criteria including 20 different symptoms across the domains of re-experiencing, avoidance, negative cognitions and moods, and hyperarousal. Ali has none of such symptoms; therefore, unlikely to have PTSD.

Major depressive disorder (option A) requires the persistence of 5 or more of the 9 Criteria A symptoms listed in the Diagnostic and Statistical Manual of Mental Disorders, 5th edition (DSM-5) – depression, loss of pleasure, weight change, sleep change, retardation, loss of energy, feelings of worthlessness, diminished concentration, thoughts of death – in a person who has never had an episode of mania. Ali does not meet the criteria for major depressive disorder.

Late-onset schizophrenia (option D) is defined as schizophrenia starting after the age of 45 years.
Patients with late-onset schizophrenia typically present with the same positive psychotic symptoms (paranoid delusions and hallucinations) as do younger schizophrenic patients, although these symptoms are less severe in older patients. In contrast with early-onset disease, disorganised thoughts and negative symptoms (flat affect, alogia, avolition) are less likely to occur in patients with late-onset disease. Late-onset schizophrenia is also less likely to be associated with impaired learning and cognitive functions. Ali has no paranoia, delusions or hallucinations; therefore, not likely to have late-onset schizophrenia.

Dementia is a clinical syndrome that is caused by a number of underlying diseases. Such as vascular dementia, frontotemporal dementia, dementia with Lewy body, Alzheimer’s. The DSM-5 diagnostic criteria for dementia include the following:
* Significant cognitive decline from a previous level of performance in one or more cognitive domains (i.e., complex attention, executive function, learning and memory, language,perceptual-motor or social cognition.
* The cognitive deficits interfere with independence in everyday activities (paying bills, managing medications).
* The cognitive deficits are not better explained by another mental disorder (e.g., major depressive disorder, schizophrenia).

Ali has no memory loss, language problem, or cognitive deficit; therefore, unlikely to have dementia
(option E).

How well did you know this?
1
Not at all
2
3
4
5
Perfectly
8
Q

During taking a history from a patient in a psychiatry ward, you ask him if there is any history of mental illnesses in the family. He answers: ‘Good question. Now that you’ve asked, I must tell you something. Whales are good creatures. We take their oil and use it to light the world. My father hunted whales as did my grandpa. The business runs in the family and of course, we are all sane and sound. Do you think we are crazy?” Which one of the following is present in this patient’s speech?

A. Flight of ideas.
B. Word salad.
C. Circumstantiality.
D. Derailment.
E. Tangentiality.

A

Correct Answer Is C.

The speech given by this patient as the answer to the question of whether there is a family history of mental illnesses starts with a comment on an unrelated topic, whales. Thereafter, each next sentence uses a clue in the previous sentence for continuation. At the end and after many detours the answer to the question is provided: ‘We are all sane and sound’ (meaning there is no family history of mental illness and they all have been sane and sound.

This pattern fits ‘circumstantiality’ best.

Circumstantiality occurs when the patient drifts from one topic to the other but eventually returns to the starting point. In other words, if a question is imagined as a destination, there are many detours, but the destination is reached at the end. Unlike
circumstantiality, patients with derailment (option D) (loosening of association) never come back
to the topic they started off with. Tangentiality (option E) or tangential speech is a milder form of derailment in which there is a hint linking two consecutive topics (whales and the whale oil that
light up the world [whale oil in the past had industrial use including for lighting and as fuel]).

Flight of ideas (option A) is characterized by over-productive speech with rapid shifting from one
topic to another. There is often a hint in the previous topic leading to the next one. In the flight of ideas, there is a subjective feeling that the thoughts are racing. In the flight of ideas, the topic spoken by the patient is organized but over-productive and in excess of details. In other words, the general concept of the current topic is adequately understood, but a hint in one part leads the patient to another. This patient follows a direct line made of pieces that are relevant and justify his sanity and that of his family for that matter, rather than jumping from one topic to another.

In word salad (option B), the patient throws words together without any sensible and intelligible
meaning.

How well did you know this?
1
Not at all
2
3
4
5
Perfectly
9
Q

Jarred, 15 years old, is brought to you by his concerned parents for assessment. According to them, he recently has significantly declined school performance. He is quite withdrawn and barely
leaves his room. He does not engage in social activities and even rarely talks with his parents. Which one of the following, if present in the history of the mental exam, is most likely to help reach a diagnosis?

A. Sleep issues.
B. Anxiety.
C. Loosening of association.
D. Use of recreational drugs.
E. Shyness.

A

Correct Answer Is C.

The scenario illustrates social withdrawal represented by not engaging in social activities, barely talking with the parents, and not leaving the room. He also has a significant decline in school performance. This constellation of problems is likely to have been caused either by a mood disorder or by the prodromal phase of early-onset schizophrenia. Of the options, the presence of loosened association is highly predictive and suggestive of the latter.

“Loose associations” is a psychological term to describe a lack of connection between ideas. This
can manifest in speech as an individual moving quickly from one idea to an unrelated one in the same sentence, expressing a random jumble of words and phrases. An example of a loose association would be: “I like to dance; my feet are wet.” Loosening of association is a key symptom in psychotic disorders such as schizophrenia.

Schizophrenia typically manifests with a prodrome of negative symptoms and psychosis (e.g., social withdrawal) that precedes the positive psychotic symptoms (e.g., hallucinations and bizarre delusions).
Childhood-onset and early-onset schizophrenia are more severe and debilitating forms of schizophrenia. Early signs and symptoms may include problems with thinking, behavior, and emotions:

Thinking:
* Problems with thinking and reasoning
* Bizarre ideas or speech
* Confusing dreams or television for reality

Behavior:
* Withdrawal from friends and family
* Trouble sleeping
* Lack of motivation e.g., presenting with a drop in performance at school
* Not meeting daily expectations, such as bathing or dressing
* Bizarre behavior
* Violent or aggressive behavior or agitation
* Recreational drug or nicotine use

Emotions:
* Irritability or depressed mood
* Lack of emotion, or emotions inappropriate for the situation
* Strange anxieties and fears
* Excessive suspicion of others

Sleep issues (option A) could be present in schizophrenia and other psychotic disorders as well as a wide variety of other mental conditions such as mood disorders or anxiety disorders. In and of itself, sleep issues are neither specific nor diagnostic.

Anxiety (option B) may indicate anxiety disorders; however, other symptoms are not consistent with anxiety disorders. Therefore, its presence would not help with justifying this clinical
presentation and in fact will add more of a diagnostic challenge.

Individuals with mental illnesses are more prone to using recreational drugs. Also, the use of recreational drugs may lead to mental issues (e.g., acute psychosis, withdrawal syndromes, etc.) If there is a history of recreational drug use in Jarred (option D), it could be a behavioral issue associated with a mental illness such as schizophrenia, or completely irrelevant to his presentation. Either way, it is neither specific nor diagnostic.

Shyness (option E) is not uncommon among children and teenagers and could be normal behavior. Shyness in history has no diagnostic significance or importance.

TOPIC REVIEW
According to DSM-V, diagnostic criteria for schizophrenia are as follows:

At least two of the following symptoms of which at least one is the first three:
1. Delusions
2. Hallucinations (almost always auditor)
3. Disorganized speech (e.g., frequent derailment or incoherence)
4. Grossly disorganized or catatonic behavior
5. Negative symptoms (i.e., flattened affect, alogia, or avolition)

Active symptoms must persist for ≥ 1 month (or less if successfully treated) while the continuous disturbance for ≥ 6 months.

Symptoms must cause social, occupational, or personal function impairment lasting ≥ 6 months.

Other possible causes for the symptoms are excluded.

Terminology
Childhood-onset schizophrenia – Childhood-onset (or very early-onset) schizophrenia starts prior to the age of 13 years.
Early-onset schizophrenia – Early-onset schizophrenia starts prior to age 18.
Adult-onset schizophrenia – Adult-onset schizophrenia starts at or after age 18.

How well did you know this?
1
Not at all
2
3
4
5
Perfectly
10
Q

A 17-year-old girl is diagnosed with major depression associated with psychotic features. She is planned to be started on selective serotonin reuptake inhibitors (SSRIs). For how long, should the treatment be continued?

A. Six months.
B. One year.
C. Five years.
D. Life-long.
E. Two years.

A

Correct Answer Is C.

All patients with major depression require continuation of treatment for up to 6 to 12 months;
however, the course of therapy should be extended in the following situations:
* Two depressive episodes within 5 years
* Three prior episodes
* Severe psychotic depression
* Serious suicidal attempt

If a patient is started on long-term treatment, this should probably be continued for at least 3 to 5
years, after which time the need for further management should be reviewed. Some patients may
even need life-long treatment.

How well did you know this?
1
Not at all
2
3
4
5
Perfectly
11
Q

A 27-year-old man is found to have major depression after he has psychiatric evaluation following an attempted suicide. He is started on sertraline. For how long he should receive the medication?

A. Six months.
B. One year.
C. Two years.
D. Five years.
E. Life-long.

A

Correct Answer Is D.

For most patients with depression the medical treatment should be continued for at least 6 months to ideally 12 months.

In the following situations, however, the duration of therapy should be extended to 3 to 5 years:
* Two episodes of major depression in 5 years
* Three previous episodes of major depression
* Depression with psychotic features
* Depression with a serious suicidal attempt

After 3-5 years, the patient should be reassessed for the need for further management. Some patient might need lifelong antidepressant therapy.

As this patient has had a suicidal attempt, he should receive antidepressant therapy for at least 3-5 years.

How well did you know this?
1
Not at all
2
3
4
5
Perfectly
12
Q

Which one of the following does not increase the risk of suicide?

A. Alcohol abuse.
B. Poor social support.
C. Inquiring the patient about suicidal ideation.
D. Conduct disorder.
E. Schizophrenia.

A

Correct Answer Is C.

A variety of factors are associated with an increased risk of suicide:
* Psychiatric disorders - Psychiatric illness is a strong predictor of suicide. More than 90% of patients who attempt suicide have a major psychiatric disorder, and 95% of patients who successfully commit suicide have a psychiatric diagnosis. Patients with psychiatric diagnoses kill themselves at rates 3 to 12 times higher than other patients.
* Hopelessness and impulsivity - Across psychiatric disorders, hopelessness is strongly associated with suicide. Hopelessness can persist even when other symptoms of depression have remitted. Impulsivity, particularly among adolescents and young adults, is also associated with acting on suicidal thoughts.
* History of previous suicide attempts or threats - The strongest single factor predictive of suicide is prior history of attempted suicide. Patients with a previous history of suicide attempts are 5 to 6 times more likely to make another attempt; furthermore, up to 50% of successful victims have made a prior attempt. One of every 100 suicide attempt survivors will die by suicide within one year of the previous attempt which amounts to 100 times that of the general population.
Age, sex, and race - The risk of suicide increases with increasing age; however young adults attempt suicide more often than older adults. Females attempt suicide 4 times more frequently than males, but males are successful 3 times more often. These age and sex differences appear to be primarily related to the lethality of the method chosen (e.g. firearms, hanging, jumping, etc.) rather than a difference in completion rates for the same method. Elderly white men, aged 85 years and older, and young black males have the highest suicide rate.
* Marital status - based on marital status the suicide risk in descending order is increased in:
1. Those who never married
2. Widowed, separated, or divorced
3. Married without children
4. Married with children
* Occupation - Unemployed and unskilled individuals are at higher risk for suicide than those who are
employed and skilled.
* Health - Suicide risk increases with physical illness such as chronic pain, recent surgery, and chronic or terminal disease.
* Adverse childhood experiences - Childhood abuse and other adverse childhood experiences appear to increase the risk of suicide in adults. Conduct disorders in children is shown to be associated with an increased risk of suicide
* Family history and genetics - The risk of suicide increases in patients with a family history of suicide. A first-degree relative who committed suicide increases the risk six-fold.
* Antidepressants - Antidepressants can have potential association with suicide.
* Other - The risk of suicide increases in following situations:
* Accessibility to weapons
* Sociopolitical, cultural, and economic forces
* Violence and political coercion
* Economic downturns
* Living in rural areas
* Being lesbian, gay, or bisexual
* Lower intelligence

Of the given options, inquiring the patient about suicidal ideation is not associated with an increased risk of attempting suicide. In fact, asking the patient directly about suicidal ideation is an essential part in assessment of suicide risk.

How well did you know this?
1
Not at all
2
3
4
5
Perfectly
13
Q

Which one of the following will not increase the risk of depression?

A. High socioeconomic status.
B. Elderly male with cognitive decline.
C. Unemployment.
D. Family history of depression.
E. Substance misuse.

A

Correct Answer Is A.

The following are the condition associated with increased risk of depression:
* Family history of depression
* Chronic illness
* Co-occurring mental conditions such as anxiety, personality disorders, etc
* Physical illness, physical or intellectual disability
* Low self-esteem, distorted body image, social incompetence
* Entering puberty and schooling, transition into workforce and independent living (adolescents)
* Language problems, generational culture clashes, cultural nonrecognition of mental health problems, stresses from living between two cultures
* Uncertainty, fear of rejection by family and friends, desire to ‘fit in’ with perceived societal expectations, being bullied, being subjected to homophobic abuse (more in adolescents)
* Domestic violence, poverty, family discord, sexual or physical abuse
* Bereavement, separation from loved ones, divorce, trauma
* Smoking, alcohol, drug use, internet use affecting sleep
* Marginalisation, homelessness, refugee status, fostering, unemployment

The high socioeconomic status is a protective factor against depression, not a risk factor.

How well did you know this?
1
Not at all
2
3
4
5
Perfectly
14
Q

A 24-year-old woman presents with history of low mood, psychomotor retardation, decreased appetite and decreased sleep for the past 2 weeks. She has family history of bipolar disorder.

Which one of the following is less common in bipolar depression than unipolar depression?

A. Psychomotor retardation.
B. Hypersomnia.
C. Decreased appetite.
D. Positive family history of bipolar disorder.
E. Delusions and hallucinations.

A

Correct Answer Is C.

The following features are more commonly seen in bipolar depression than unipolar depression:
* Psychomotor retardation
* Increased appetite (hyperphagia)
* Increased sleep (hypersomnia)
* Early onset of first depression before 25 years of age
* Delusions and hallucinations
* Positive family history of bipolar disorder

It is very important to identify these features in patients with possible diagnosis of bipolar disorder,
who initially present with depression. This patient has positive family history of bipolar disorder, psycho-motor retardation and age of onset of depression below 25 years, all favoring bipolar depression.

NOTE - Psychomotor retardation is seen in depressive phase of bipolar disorder. It is not a feature of mania associated with bipolar disorder. Decreased sleep and decreased appetite are not common features of bipolar depression and are seen in major depression more frequently.

How well did you know this?
1
Not at all
2
3
4
5
Perfectly
15
Q

Which of the following does not increase the risk of postaprtum depression?
A. Adverse life events.
B. Lack of social support.
C. Past history of depression.
D. Emergency cesarean section.
E. Elective cesarean section.

A

Correct Answer Is E.

Postpartum depression is most commonly seen during the first 1-8 weeks after delivery. The risk factors for development of postpartum depression (and postpartum anxiety disorders) include:
* Psychological
* Antenatal anxiety, depression or mood swings
* Previous history of anxiety, depression (option C), or mood swings, especially if occurred perinatally
* Family history of anxiety, depression or alcohol abuse, especially in first degree relatives
* Severe baby blues
* Personal characteristics like guilt-prone, perfectionistic, feeling unable to achieve, low selfesteem
* EPDS (Edinburgh postnatal depression) score ≥ 12
* Social Lack of emotional and practical support from partner and/or others (option B)
* Domestic violence, history of trauma or abuse (including childhood sexual assault)
* Many stressful life events recently (option A)
* Low socioeconomic status, unemployment
* Unplanned or unwanted pregnancy
* Expecting first child or has many children already
* Child care stress
* Biological / medical
* Recent cessation of psychotropic medications
* Medical history of serious pregnancy or birth complications (including emergency cesarean section (option D), neonatal loss, poor physical health, chronic pain or disability, or premenstrual syndrome
* Perinatal sleep deprivation
* Neonatal medical problems

Elective cesarean section does not increase the risk of postnatal depression.

How well did you know this?
1
Not at all
2
3
4
5
Perfectly
16
Q

A 76-year-old man is brought to your clinic by his son because of progressive decline in memory for the past 18 months, as well as weakness of his upper and lower limbs and deterioration of motor function. On examination, right hemiparesis is evident. Which one of the following could be the most likely diagnosis?

A. Alzheimer’s disease.
B. Lewy body dementia.
C. Multi-infarct dementia.
D. Parkinson’s disease.
E. Pick’s disease.

A

Correct Answer Is C.

The clinical picture is suggestive of multi-infarct dementia as the most likely cause. Multi-infarct
dementia is the second most common cause of dementia after Alzheimer disease (10% of all cases with dementia).

Alzheimer disease (option A), Lewy body dementia (option B) and Pick’s disease (frontotemporal dementia) (option E) are not associated with focal motor dysfunction. Parkinson’s disease (option D) as a cause of dementia is associated with motor dysfunction in the form of tremors and extrapyramidal symptoms. Features include gate disturbances, tremor, rigidity and micrographia.

Hemiparesis is not a feature.

TOPIC REVIEW
Diseases associated with dementia:

Alzheimer disease - Alzheimer disease is the most common cause of dementia. Typically, patients
with Alzheimer disease present with problems in memory and visuospatial abilities that occur early in the course of the disease. Despite severe memory impairment, social grace remains intact until very late in the course of the disease when hallucinations and personality changes develop.

Alzheimer disease is not associated with motor or sensory dysfunction at least not very late in the
course of the disease.

Lewy body dementia - Lewy body dementia is characterized by fluctuating level of consciousness,
social disinhibition and Parkinsonism. Dementia often follows later. Lewy body dementia can be confused with delirium.

Vascular dementia - Vascular dementia is divided into multi-infarct dementia, which typically has a stepwise progression associated with frequent discrete cerebrovascular events, and Binswanger
disease, involving the subcortical white matter, that presents with a slowly progressive course.

Normal pressure hydrocephalus - It presents with prominent gait abnormalities early in the course
of the disease that usually precedes the onset of memory impairment. Urinary incontinence is another distinguishing feature.

Pick’s disease (frontotemporal dementia) - Patients with Pick’s disease present with personality
changes early in the course of the disease, with relative sparing of visuospatial function. Social,
interpersonal, and emotional abnormalities precede memory impairment. The condition is first noted by the family because the patient does not have insight into their problem.

Parkinson’s disease - Dementia secondary to Parkinson disease is associated with typical features
of the disease such as gate disturbances, rigidity, tremors, micrographia, etc. Recurrent visual hallucinations can be a feature that usually develop later in the course of the disease.

Creutzfeldt – Jacob disease (CJD) - Dementia of CJD develops in shorter time (weeks to months)
and has a course more aggressive than Alzheimer’s disease. Myoclonus is a distinguishing feature.
Diagnosis of CJD is by rapidly progressive dementia, myoclonus and the presence of 14-3-3 protein
in the CSF.

How well did you know this?
1
Not at all
2
3
4
5
Perfectly
17
Q

Accompanied by his wife, a 63-year-old man presents to your practice for consultation. She is concerned about her husband because he has been recently behaving childish and bizarre. Last week he was dismissed from his job as a manager in a local restaurant, because of treating rude to customers and shouting at his colleagues. He does not shave, bathe or change his clothes as he did before and is disheveled and unkempt all the time. She denies any falls, gait abnormalities, or hallucinations in her husband. His memory is not significantly affected.

Which one of the following could be the most likely diagnosis?

A. Depression.
B. Alzheimer disease.
C. Lewy body dementia.
D. Frontotemporal dementia.
E. Schizophrenia.

A

Correct Answer Is D.
Cognitive and behavioral changes in aged people are frequently faced in general practice, with dementia and delirium being the most common underlying etiologies.

Cognitive function is measured by various mental functions, including memory, concentration,
praxis, language, executive functions, and visuospatial skills. Dementia refers to memory loss with
impairment of any other cognitive function that can interfere with social or occupational functioning.

A myriad of causes have been identified for dementia. These causes can be reversible or irreversible.

The most common reversible causes of dementia include:
* Hypothyroidism
* Vitamin B12 deficiency
* Hepatic or uremic encephalopathy
* Vasculitides affecting CNS
* Space occupying brain lesions i.e. abscess/tumors either primary or metastatic
* Medications – anticholinergics in particular
* Normal pressure hydrocephalus
* Central or obstructive sleep apnea
* Subdural hematoma
* Trauma
* Depression

Some of the most common irreversible causes of dementia are:
* Alzheimer disease (60-80% of cases )
* Vascular dementia including multi-infarct dementia and Binswanger disease
* Lewy body dementia
* Frontotemporal degeneration (dementia) including Pick disease
* Multifocal leukoencephalopathy

The case scenario describes a patient with social inappropriateness as the most concerning presenting symptom without memory being significantly involved. Of the options, the most consistent one with such scenario is frontotemporal dementia (Pick disease). In this disease, social disgrace is the earliest symptom with memory impairment and forgetfulness following later.

Frontotemporal dementia is characterized by focal degeneration of the frontal and/or temporal lobes. The typical age of onset is in the late 50s or early 60s, and the primary initial clinical manifestations are changes in personality and social behavior or language, progressing over time to a more global dementia. Other features include impaired initiation and planning, disinhibited behavior and social disgrace and mild abnormalities on cognitive testing. Apathy and memory deficits develop later in the course of the disease. A subset of patients may also exhibit symptoms of extrapyramidal or motor neuron involvement at some point in the disease process.

(Option A) Patients with depression may present with pseudodementia which is different from dementia in some aspects. It is less common for patients with pseudodementia to have disinhibition or social disgrace. The history of disturbances in pseudodementia is often short and abrupt onset, while dementia is more insidious. On cognitive testing, people with pseudodementia often answer that they do not know the answer to a question, and their attention and concentration are intact and they may appear upset or distressed. Those with true dementia will often give wrong answers, have poor attention and concentration, and appear indifferent or unconcerned.

(Option B) In Alzheimer disease, forgetfulness is usually the presenting symptom. It is very unlikely
for a patient with Alzheimer disease to present with disinhibition and social inappropriateness early in the course of the disease.

(Option C) although misbehavior and disinhibition is a common early feature in patients with Lewy body dementia, the absence of other manifestations such as fluctuating cognition, hallucinations, extrapyramidal deficits (Parkinsonism) and repeat falls makes this diagnosis less likely.

(Option E) Psychotic features such as hallucinations and delusion are a significant diagnostic component in schizophrenia that is absent here. Moreover, development of schizophrenia at this
age is unusual.

How well did you know this?
1
Not at all
2
3
4
5
Perfectly
18
Q

Which one of the following diseases of the central nervous system is caused by infectious proteins?

A. Alzheimer’s disease.
B. Creutzfeldt- Jakob disease.
C. Parkinson’s disease.
D. Pick’s disease.
E. Guillain-Barre syndrome.

A

Correct Answer Is B
Creutzfeldt-Jakob disease is caused by an infectious protein particle called prion. The disease is often contracted at about mid-seventies and presents with dementia and myoclonus. It has a progressive course and death follows in one year.

How well did you know this?
1
Not at all
2
3
4
5
Perfectly
19
Q

A 36-year-old man is brought to the emergency department by his relatives due to what they call an outburst. When you step in the examining room, you realise that he is very agitated and is yelling at the staff angrily and threatening to kill anyone who touches him. You are informed by the staff that he has the past history of schizophrenia and has been aggressive and violent at previous presentations. Which one of the following would be the next best step in management?

A. Tell the hospital security guards to hold him until he is calm.
B. Alert the hospital security to intervene and sedate him with haloperidol or benzodiazepine.
C. Call hospital security and request the patient to behave properly.
D. Ask the hospital security to escort the patient out of the hospital.
E. Hand him over to the police.

A

Correct Answer Is B.

Angry and agitated patients usually present a challenge to the health care providers. The best approach is to calm down the patient in tactful professional manners and ideally without use of sedatives; however when the patient is feared to pose harm to him/her or medical staff, hospital security is required to be called and the patient sedated.

Since this patient is high-risk for violence due to his psychiatric condition and past history of violence, calling the hospital security and sedating him would be the next best step in management.

In all Australian emergency departments, the’ zero tolerance policy to violence’ is adopted. This policy mandates sedation on the slightest concern of harm from the patients either to themselves or the staff. The medications of choice for this purpose include haloperidol (preferred) and midazolam. Midazolam carries the risk of respiratory depression and hypotension and should be used with great caution.

Benzodiazepines are first line when agitation is likely to have caused by alcohol intoxication or withdrawal. Patients in need of medical attention should be treated, not handed over to the Police, or escorted out.

How well did you know this?
1
Not at all
2
3
4
5
Perfectly
20
Q

During assessment of a 32-year-old woman, she says ‘Oh well, my food! My food is cheese, cheese is in the air, air is blue, and I came by bus’. Which one of the following can be the most likely condition she is suffering from?

A. Schizophrenia.
B. Depression.
C. Dissociation.
D. Psychosis.
E. Adjustment disorder.

A

Correct Answer Is D.

The scenario describes a typical example of ‘tangentiality’. Tangentiality occurs when one idea
connects to the next with one word or phrase, but the thoughts become confusing because they go off on a tangent and end in a different subject. In the above example the word ‘food’ ends in another comment about food unrelated to the previous sentence. The word ‘food’ is the only connection. Loosening of association (derailment), on the other hand, happens when one idea does not connect to the next at all.

Following are sentences told by patients in real situations. The first two are examples of derailment
(loosening of association), while the third is ‘tangentiality’:

  • The next day when I’d be going out you know, I took control. Like uh, I put bleach on my hair …
  • The traffic is rumbling along the main road. They are going to the north. Why do girls always play pantomime heroes?
  • I think someone has infiltrated my copies of the cases. We’ve got to case the joint. I don’t believe in joints, but they do hold your body together.

Derailment, loosening of association, poverty of content of speech, and thought blocking are
examples of disorganized thought content as characteristic features of psychosis and psychotic
disorders. A brief psychotic episode, schizophreniform disorder, schizoaffective disorder and schizophrenia can have tangentiality/derailment as a presentation. Psychosis is the option encompassing all these disorders and the correct answer for this question.

How well did you know this?
1
Not at all
2
3
4
5
Perfectly
21
Q

An alcoholic man is brought to your clinic because of alcohol intoxication. He is successfully resuscitated. Which one of the following conditions in the history will direct you towards alcohol dependence in this patient?

A. Any compulsory alcohol drinking first thing in the morning.
B. He drinks when he is anxious.
C. He drinks socially.
D. He drinks when he is driving.
E. He drinks more than 4 standard drinks in one single session.

A

Correct Answer Is A.

There are screening tools to assess the likelihood of alcohol dependence in primary care setting.
CAGE questionnaire and AUDIT (Alcohol Use Disorders Identification Test) questionnaires are the
two most commonly use tools for this purpose.
The CAGE test consists of 4 questions. The letters of the acronym are the initial letters of key
words in questions:
1. Have you ever felt the need to CUT down on your drinking?
2. Have you ever felt ANNOYED by others asking you about your drinking?
3. Do you feel GUILTY about your drinking?
4. Do you ever have an EYE-OPENER in t the morning?

A positive response to any item on the CAGE questionnaire is a pointer towards alcohol
dependence and warrants a detailed assessment.
AUDIT questionnaire consists of 10 questions and a maximum score of 40. A score of 8 or more out of the total score of 40 suggests alcohol dependence. AUDIT questionnaire has minimal false positive or negative results.

Of the given options, compulsory morning drinking (eye opener) is suggestive of alcohol dependence.
Other options may or may not be seen in alcohol dependence and are not useful in determining the
possibility of alcohol dependence.

How well did you know this?
1
Not at all
2
3
4
5
Perfectly
22
Q

Jane, 32 years, gave birth to her child 3 days ago and has been in the ward since. On the second day, she went to the nurses and told them that she is very worried and afraid about her child and needs to keep her safe in the room. Last night she was seen walking around and moving objects whole night. She took an insecticide and sprayed on the floor to get rid of imaginary bugs. On examination, she is agitated with a heart rate of 110 bpm, respiratory rate of 22 breaths per minute and temperature of 37.5°C. Which one of the following is most important initial step in management?

A. Blood culture.
B. Full blood count (FBC).
C. Urine drug screen.
D. CT scan of the head.
E. Urine analysis (UA).

A

Correct Answer Is C.

The scenario is consistent with diagnosis of an acute psychotic episode in early postpartum period. A few explanations should be considered and investigated. The most important ones include:
* Substance-induced psychotic disorder (substance intoxication/withdrawal)
* Brief psychotic disorder
* Postpartum psychosis

Considering the time of the presentation (shortly after delivery) and other physical finding, drug intoxication as a likely cause should be excluded first. For this purpose, a urine drug screen test is the most appropriate measure.

A rare yet important diagnosis to consider is postpartum psychosis. The condition most commonly
presents within 2 weeks of childbirth. Hallucinations and delusions are usually present, often with thought disorganization and/or bizarre behavior.

Although one should have postpartum psychosis as one possible diagnosis, more prevalent etiologies for an acute psychotic episode should be consider and excluded first, especially with the very early onset of symptoms which seems somewhat unusual (but not impossible) for postpartum psychosis.

(Options A, B and E) With a temperature of 37.5°C, it is very unlikely that an infectious process is
the cause of this presentation; hence, blood culture, FBC, and UA are not appropriate steps; at least not as the most important initial approach.

(Option D) CT scan of the head would have been indicated as a part of initial management if there was a pointer towards an intracranial pathology.

How well did you know this?
1
Not at all
2
3
4
5
Perfectly
23
Q

Alcohol-related dementia acounts for what percent of all cases of dementia in Australia?

A. 10%.
B. 5%.
C. 90%.
D. 80%.
E. 35%.

A

Correct Answer Is B.

Incidence of alcohol-related dementia in Australia is about 5% (5.4% according to a study by Panegyres and Frencham (2000)) of total demented population. Vascular dementia accounts for 10% and Alzheimer for 80-85% of all cases of dementia in Australia.

How well did you know this?
1
Not at all
2
3
4
5
Perfectly
24
Q

A 23-year-old woman is referred to you for psychiatric assessment. When you ask her what she
thinks to be her problem, she answers: “Oh, isn’t it a long story? The train always runs on rails; the birds keep flying up the hills and I have to remember to count my chickens before they hatch.” At the end she does not come back to answer the question you asked. Which one of the following is the most likely thought disorder she has?

A. Derailment.
B. Flight of ideas.
C. Pressured speech.
D. Circumstantiality.
E. Word salad.

A

Correct Answer Is A.

The speech pattern given in the question is characteristic of derailment, also known as “loosening
of associations”, in which there is no connection between one topic and the next. One sentence is spoken and then another sentence unrelated to the previous one follows.

Tangentiality is a milder form of derailment in which there is a linking hint between the two consecutive sentences. An example is: I think someone has infiltrated my copies of the CASEs. We’ve got to CASE the JOINT. I don’t believe in JOINTs, but they do hold your body together.

Words in upper case are the links between each sentence and the previous one. These connecting hints are characteristic of tangentiality. In tangentiality (like derailment) there is no returning to the initial topic.

(Option B) Flight of ideas is characterized by over-productive speech with rapid shifting from one topic to another. There is often a hint in the previous topic leading to the next one. In the flight of ideas, there is a subjective feeling that the thoughts are racing. In flight of ideas, the topic spoken by the patient is organized but over-productive and in excess details. In other words, the general concept of the current topic is adequately understood, but a hint in one part leads the patient to another topic. The flight of idea is a matter of switching between coherent ideas.

(Option C) Pressure of speech (pressured speech) is a tendency to speak rapidly and excitedly, as if motivated by an urgency not apparent to the listener. The speech is difficult to interrupt. It may be too fast, or too tangential for the listener to understand.

(Option D) Circumstantiality occurs when the patient drifts from one topic to the other, but eventually returns to the starting point. In other words, if a question is imagined as a destination, there are many detours, but the destination is reached at the end. Unlike circumstantiality, patients with derailment (loosening of association) never come back to the topic they started off with.

Tangentiality or tangential speech is a milder form of derailment in which there is a hint linking two consecutive topics.

(Option E) In word salad, the patient throws words together without any sensible and intelligible
meaning.

How well did you know this?
1
Not at all
2
3
4
5
Perfectly
25
Q

A 47-year-old homeless man is brought to the Emergency Department by the police after he was
found agitated and fearful in the streets while carrying a bag full of rubbish. He says to you, as the
treating doctor, that he was caught by special military forces because he did not returned the DVDs to a shop that belongs to the Jewish brotherhood. He also mentions that he has been followed by demons because he has not done his prayers. He does not seem to be drunk but looks exhausted and agitated. Which one of the following options is the most appropriate next step in management of this patient?

A. Admit him to psychiatric unit for evaluation and treatment.
B. Ask the police about him.
C. Arrange urgent parenteral antipsychotics.
D. Arrange admission to general ward for assessment of his medical status.
E. Perform a urinary toxicology screen.

A

Correct Answer Is B.

Psychiatric evaluation of the agitated patient includes:
De-escalation (verbal de-escalation; medical deescalation [e.g. sedation] if necessary)

Visual observation of the patient before direct patient interview and paying careful attention to the patient’s verbal and nonverbal interaction with the examiner during de-escalation.

Collecting collateral information - Collecting collateral information can be very helpful. While de-escalation is in process, another team member can obtain verbal reports from family, paramedics, or police officers or review written material that may accompany the patient.

Medical records are also an important source of information, and electronic records, if available, can be readily accessed to determine previous diagnoses and medications. These sources of information can be invaluable in determining the cause of agitation. Of the given options, asking the police about him has the highest priority as it can provide additional information for further management.

Once it is determined that the patient does not have an acute medical problem, there are several important questions, the answers to which will guide the next step in management of the patient. The first question is whether the patient has a delirium. It is not uncommon for a patient to go through initial screening and have a diagnosis of delirium overlooked. The patient may be mistakenly diagnosed as being psychotic, or the signs and symptoms of delirium may be subtle and easily overlooked. In delirium, the patient has an altered level of awareness and problems directing, focusing, sustaining, or shifting attention. The examiner must pay close attention to how the patient interacts during the encounter and recognize these often subtle signs. Does the patient seem confused and unable to focus? Are there perseverative behaviors? Does the patient appear to be responding to visual hallucinations? Are there signs of language impairment, problems naming, or other cognitive deficits? If agitation is associated with any of these findings, especially in the
setting of drug or medication use or medical illness, the presumptive diagnosis is delirium.

Risk of suicide or self-harm should be assessed and if presents managed accordingly. The possibility of substance intoxication/withdrawal should be borne in mind and investigated if indicated.

Other measures include assessment of general medical condition, followed by a detailed formal
psychiatric assessment if an underlying psychiatric condition is suspected. Antipsychotics may be used for treatment of an agitated patient if verbal de-escalation fails.

How well did you know this?
1
Not at all
2
3
4
5
Perfectly
26
Q

A 35-year-old man presents to your practice with complaint of impotence for the past few months. He smokes 20 cigarettes and regularly drinks 3-4 glasses of wine a day. He is marrying his fiancee in one month. Which one of the following would be the most appropriate management option for him at this stage?

A. Cessation of smoking.
B. Cessation of alcohol drinking.
C. Referral for psychotherapy.
D. Prescribe sildenafil.
E. Prescribe clomipramine.

A

Correct Answer Is D.

Unless contraindicated, oral phosphodiesterase type 5 (PDE5) inhibitors such as sildenafil, vardenafil, tadalafil and avanafil are first-line therapy for erectile dysfunction, regardless of the etiology. In patients refractory to oral PDE5 inhibitors, one of these agents can be combined with an injection of the prostaglandin PGE1.

Although not proven, it is likely that erectile dysfunction can be prevented by good general health,
paying particular attention to body weight, exercise, and cigarette smoking and alcohol consumption. These measures should be considered but are not first-line treatment plan.

TOPIC REVIEW
Impotence is defined as male erectile dysfunction (ED), that is, inability to achieve or maintain an erection sufficient for satisfactory sexual performance.

ED usually has a multifactorial etiology. Organic, physiologic, endocrine, and psychogenic factors are involved in the ability to obtain and maintain erections. In general, ED is divided into 2 broad categories, organic and psychogenic. Although psychologic factors have been mostly implicated
as the contributing factor, pure psychogenic ED is in fact uncommon; however, many men with organic etiologies may also have an associated psychogenic component.

Conditions that may be associated with ED include diabetes, hypertension, and coronary artery disease as well as neurologic disorders, endocrinopathies, benign prostatic hyperplasia, sleep apnea, COPD, and depression. In fact, almost any disease may affect erectile function by altering the nervous, vascular, or hormonal systems. Various diseases may produce changes in the smooth muscle tissue of the corpora cavernosa or influence the patient’s psychological mood and behavior.

Conditions associated with reduced nerve and endothelium function (e.g. aging, hypertension, smoking, hypercholesterolemia, and diabetes) alter the balance between contraction and relaxation factors. These conditions cause circulatory and structural changes in penile tissues, resulting in
arterial insufficiency and defective smooth muscle relaxation. In some patients, sexual dysfunction may be the presenting symptom of these disorders. Given the multiplicity of possible etiologic factors, it may be difficult to determine how much any given factor is contributing to the problem. A thorough evaluation is necessary for correct identification of the specific cause or causes in any given individual.

NOTE – hypertension has been found to have the highest association with ED. Benign prostatic hyperplasia (BPH) has also a strong association. The cause exact mechanism, however, is unclear.

The first step in the management of ED is a thorough history that includes the following:
* Sexual history
* Medical history
* Psychosocial history

A physical examination is necessary for every patient, emphasizing the genitourinary, vascular, and neurologic systems. A focused examination entails evaluation of the following:
* Blood pressure
* Peripheral pulses
* Sensation
* Status of the genitalia and prostate
* Size and texture of the testes
* Presence of the epididymis and vas deferens
* Abnormalities of the penis (e.g. hypospadias, peyronie plaques)

Management options for ED include the following:
* Sexual counseling, if no organic causes can be found for the dysfunction. Sexual counselling, however, can be used as an ongoing treatment along with other therapies
* Oral medications
* Injected, implanted, or topically applied medications
* External vacuum and constriction devices
* Surgery (penile prosthesis, vascular surgery)

How well did you know this?
1
Not at all
2
3
4
5
Perfectly
27
Q

A 17-year-old boy is brought to your office by his mother for evaluation. History revleas that he had
always been a good student until the age 15 when his grades began dropping. His mother has also noticed that he has stopped socializing with friends and begun spending more time alone in his room. He dropped the school earlier this year due to poor school performance. His mother reports that he has been trying to home-school him, but he does not seem interested in learning. Days go before he takes bath and he does not seem to care about his appearance. What concerns the mother most and has led her to bringing him to see a psychiatrist is the fact that for the past 2 months, she has overheard him having what sounds like conversations with someone imaginary in his room. Which one of the following could be the most likely diagnosis?

A. Major depressive disorder.
B. Substance abuse disorder.
C. Schizophrenia.
D. Brief psychotic disorder.
E. Schizophreniform disorder.

A

Correct Answer Is C.

The scenario, describes a condition of 2-year duration, with social withdrawal as the initiating event (dropping grades and poor school performance), refusing to home-learn, unkempt appearance, and more importantly conversations with an imaginary individual that is a strong pointer towards hallucinations. These features are characteristic of schizophrenia as the most likely diagnosis.
To establish the diagnosis of schizophrenia the following criteria should be met:
Two (or more) of the following symptoms:
* Hallucinations
* Delusions
* Disorganized speech
* Grossly disorganized or catatonic behavior
* Negative symptoms
AND
* The symptoms are severe enough to cause significant social or occupation disability
AND
* The symptoms have lasted for at least 6 months
AND
* Other psychotic conditions such as schizophreniform disorder, brief psychotic episode, etc. are excluded
AND
* Drugs or medical conditions as the cause of symptoms are excluded
AND
* The condition could not be solely attributed to pervasive developmental disorders such as
autistic disorders

This young man’s conversations with imaginary represents hallucinations. His lack of motivation,
isolation, and poor self-care are examples of negative symptoms for 2 years or more. He has been like this for more than 6 months now and there is marked social or occupational dysfunction as evidenced by his poor school performance. These fulfill the criteria for schizophrenia as the diagnosis.

(Option A) Major depressive disorder can present with psychotic symptoms occurring in the context of a major depressive episode. The patient’s history does not suggest major depression. He has had 2 years of negative symptoms followed by positive symptoms, suggesting schizophrenia rather than major depression.

(Option B) A medical or substance-related condition is always among the differential diagnoses in patients with psychotic features. There is no hint about a medical condition as the underlying cause in the scenario, and it is unlikely that substance abuse of a teenager who lives with his parents goes unnoticed for 2 years.

(Option D) Brief psychotic disorder can share many features with schizophrenia. The distinctive
feature; however, is the symptoms duration of less than one month with eventual return to premorbid functioning.

(Option E) Although schizophreniform disorder shares many features with schizophrenia, the symptoms last no longer than 6 months in schizophreniform disorder.

How well did you know this?
1
Not at all
2
3
4
5
Perfectly
28
Q

A 50-year-old man presents to your practice, significantly concerned about amnesia. He relates that he has been to a place for work for the first time with one of his colleagues. He thought that this is
his first time he had ever visited that place but his colleague was surprised and told him that he had visited this place before and wondered why he could not remember. His past medical history is significant for electroconvulsive therapy (ECT) sessions due to major depression with last session being 3 months ago. Which one of the following in the history best justifies his presenting complaint?

A. History of dementia in his cousin.
B. History of depression.
C. Previous ECT sessions.
D. De javu.
E. Relapse of his major depression.

A

Correct Answer Is C.

Of the given options, previous ECT sessions are the most appropriate and justifying explanation to this man’s current problem that is forgetting past events i.e. retrograde amnesia.

Electroconvulsive therapy (ECT) is a treatment option for several psychiatric conditions. Major depression associated with psychotic features, strong suicidal ideation, unresponsiveness to other therapies and previous favorable response to ECT are the most frequent indications.

Most patients report some adverse cognitive effects during and after a course of ECT. The incidence depends upon electrode placement, stimulus type and dose, anesthesia, and the patient’s pretreatment cognitive status. A systematic review found four studies in which the proportion of patients who reported any memory loss ranged from 51 -79%.

ECT can cause three types of cognitive impairment:
Acute confusion - the acute confusional state is the result of both the seizure and the anesthesia. It
typically resolves 10 to 30 minutes after the procedure.
Anterograde amnesia - anterograde amnesia is the decreased ability to retain newly acquired information. It occurs during a course of ECT and typically resolves within two weeks after completing the course.
Retrograde amnesia - retrograde amnesia is defined as forgetting recent memories and is the most anxiety-producing and frightening cognitive effect of ECT. The affected memories are for events
that occur during the course of ECT and a period of weeks to a few months prior to that. The deficits are greatest and most persistent for knowledge about public or world events compared with knowledge about the self (personal memory). Bilateral ECT causes more retrograde amnesia than right unilateral. Retrograde amnesia recovers more slowly than anterograde amnesia. Some of the lost memories of events prior to the course of ECT may be expected to return, while others may not.

(Option A) Although dementia starts by disorientation, typically first to places, this man is young for dementia, and while dementia in a first-degree relative can be risk factor, a demented cousin does not appear to be of importance.

(Options B and E) Neither the history of depression in itself, nor a relapse of depressive episode justifies dementia.

(Option D) In Déjà vu is the sense familiarity with unfamiliar things. For example the patient feels that he/she has been in place before while he/she has not. This is not the case here.

How well did you know this?
1
Not at all
2
3
4
5
Perfectly
29
Q

Which one of the following is not considered a basic investigation for patients with anorexia nervosa?

A. Electrolytes.
B. Urinalysis.
C. Liver function tests.
D. DXA scan.
E. Electrocardiogram (ECG).

A

Correct Answer Is D.

Anorexia nervosa is characterized by failure to maintain a normal body weight, fear of weight gain and preoccupation with a distorted body image and unrealistic self-evaluation as overweight. There are two types: (1) restrictive and (2) binge eating/purging.

In restrictive type, the patient strictly reduces food intake and maintains diets of low-calorie foods. In purging type, weight loss is achieved through vomiting, laxatives, diuretics, or enemas.

Clinical manifestations of anorexia nervosa are mostly related to prolonged starvation and malnutrition, and include:
* Hypothermia
* Acrocyanosis
* Resting bradycardia (resting heart rate often 40-49 beats per minute)
* Hypotension
* Orthostatic lowered blood pressure or pulse
* Loss of muscle mass
* Low blood glucose (impaired insulin clearance)
* Low parathyroid hormone levels
* Elevated liver function tests
* Low white blood cell (WBC) count

Since eating disorders are clinical diagnoses, no definitive diagnostic tests are available for anorexia nervosa; however, with negative impacts of starvation on different multiple organ systems, the following basic tests should be performed for every patient with anorexia nervosa as baseline tests:
* Physical and mental status evaluation
* Full blood exam (FBE) and ESR
* Metabolic panel (Na, K, Bun, urea, Cl, BS, HCO3)
* Urinalysis for ketones and specific gravity
* Liver function tests
* Pregnancy test (in patients of childbearing age)

Both dehydration and increased fluid intake can occur in anorexia nervosa, leading to electrolytes imbalances with negative impacts on cardiac conduction system, central nervous system, etc.

Therefore, electrolyte assessment (option A) should be considered in all patients with anorexia
nervosa and bulimia nervosa.

Cardiac problems is the leading cause of death in patients with anorexia nervosa and an ECG (option E) should be obtained in all such patients as a very important basic assessment. In anorexic patients with BMI less than 14kg/m2, an echocardiography is mandated.

Urinalysis (option B) is another component of basic assessment. Specific gravity reflects hydration
status, and ketonuria indicates starvation.

Prolonged starvation has deleterious effects on liver function; hence, liver function tests (option C)
are indicated as basic investigations in all patients with anorexia nervosa.

Patients with anorexia nervosa and bulimia nervosa may have central hypogonadism and estrogen deficiency. Estrogen deficiency can result in osteopenia and osteoporosis. A Bones scan (DXA
scan) is indicated for all patients with anorexia nervosa of longer than 6 months duration; this test however is not among base line investigations. It can be considered later in the course of management.

How well did you know this?
1
Not at all
2
3
4
5
Perfectly
30
Q

Which one of the following features does not help to differentiate mania from hypomania?

A. Delusions.
B. Hallucinations.
C. Functional impairment.
D. Need for hospitalization.
E. Elevated mood.

A

Correct Answer Is E.

Features of mania versus hypomania is summarized in the following table: See table below.

Elevated mood is seen both in mania and hypomania and does not help to differentiate between these two.

Mania versus hypomania is the parameter by which bipolar disorder is classified into I and II.

Patients with mania are calssified as bipolar I disorder, whereas those with hypomania are labeled as having bipolar II disorder.

How well did you know this?
1
Not at all
2
3
4
5
Perfectly
31
Q

A 17-year-old girl has bizarre eating behavior noted by her mother. She indulges in episodes of binge eating at least once or twice a week. She spends several hours a day doing excessive workouts, and follows a restrict low-calorie diet because she believes she is fat. She also spends considerable amount of time in front of mirror checking her body. On examination, she has a BMI of 19 kg/m2. Which one of the following could be the most likely predisposing factor to this presentation?

A. Alcohol abuse.
B. Childhood sexual abuse.
C. Family history of obesity.
D. Family history of athleticism.
E. History of substance abuse.

A

Correct Answer Is B.

The case scenario describes a typical case of bulimia nervosa, indicated by binge eating, compensatory behavior to lose weight, and normal BMI that differentiates it from binging/purging type of anorexia nervosa.

There is no consensus on the causes of eating disorders. A combination of genetic, biologic, psychological, family, environmental, and social factors probably contribute to developing an eating
disorder.

Factors associated with the development of eating disorders include:
* Genetics – genetic factors are involved in the pathogenesis of eating disorders. As an example, concordance for either anorexia nervosa or bulimia nervosa is greater in monozygotic twins compared with dizygotic twins.
* Family distress – family characteristics associated with eating disorders may include high perceived parental expectations for achievement and appearance, families who communicate poorly, have members who are enmeshed with or estranged from each other, devalue the mother or maternal role, have marital tension, or have difficulty managing conflicts.
* Sexual abuse
* History of dieting
* Preoccupation with a thin body and social pressure about weight
* Athletic and artistic endeavors such as running, ballet, etc. that emphasizes leanness, and sports in which scoring is partly subjective (e.g., skating or gymnastics)

Of the given options, sexual abuse is the most important risk factor for development of eating disorders, as well as other psychiatric conditions such as substance abuse, depressive disorders, and anxiety disorders.

Sexual abuse can have many different effects on eating habits and body image of survivors. Sexual abuse violates the boundaries of the self so dramatically that inner sensations of hunger, fatigue,
or sexuality become difficult to identify. Victims of sexual abuse may turn to food to relieve a wide range of different states of tension unrelated to hunger. It is their confusion and uncertainty about their inner perceptions that leads them to focus on the food.

Many survivors of sexual abuse often work to become very fat or very thin in an attempt to render themselves unattractive. In this way, they try to de-sexualize themselves. Other survivors obsessively diet, starve, or purge to make their bodies “perfect.” A perfect body is their attempt to feel more powerful, invulnerable, and in control, so as not to re-experience the powerlessness they felt as children.

Several studies have found that childhood trauma such as childhood sexual abuse is a risk factor
for onset of bulimia nervosa as well as discontinuation of psychotherapy for bulimia nervosa; In addition, a history of childhood maltreatment in patients with bulimia nervosa is associated with comorbid depressive disorders, anxiety disorders, and borderline personality disorder.

In all eating disorders, there is an increased genetic heritability and frequency of a family history. A family history of ‘leanness’ or ‘thinness’ or ‘elite athleticism’ (option D) may be associated with anorexia nervosa, whereas bulimic eating disorders are associated with a personal or family history of obesity (option C). Although these risk factors are specific to anorexia nervosa and bulimia nervosa, they are less likely to trigger the eating disorder compared to sexual child abuse.

Substance abuse (option E) for weight reduction has been frequently observed among patients with anorexia nervosa and bulimia nervosa as a consequent of the disorder, but not as an underlying etiology.

Alcohol abuse (option A) does not seem to increase the risk of developing anorexia/bulimia nervosa.

How well did you know this?
1
Not at all
2
3
4
5
Perfectly
32
Q

Which one of the following is the most important risk factor for developing anorexia nervosa?
A. Alcohol abuse.
B. Childhood sexual abuse.
C. Family history of obesity.
D. Family history of athleticism.
E. History of substance abuse.

A

Correct Answer Is B
Of the given options, sexual abuse is the most important risk factor for development of bulimia
nervosa, anorexia nervosa and other eating disorders, as well as other psychiatric conditions such
as substance abuse, depressive disorders, and anxiety disorders.
Sexual abuse can have many different effects on the eating habits and body image of survivors.
Sexual abuse violates the boundaries of the self so dramatically that inner sensations of hunger,
fatigue, or sexuality become difficult to identify. Victims of sexual abuse may turn to food to relieve
a wide range of different states of tension unrelated to hunger. It is their confusion and uncertainty
about their inner perceptions that leads them to focus on the food.
Many survivors of sexual abuse often work to become very fat or very thin in an attempt to render
themselves unattractive. In this way, they try to desexualize themselves. Other survivors
obsessively diet, starve, or purge to make their bodies “perfect.” A perfect body is their attempt to
feel more powerful, invulnerable, and in control, so as not to re-experience the powerlessness they
felt as children.
Several studies have found that childhood trauma such as childhood sexual abuse is a risk factor
for onset of bulimia nervosa as well as discontinuation of psychotherapy for bulimia nervosa.
Studies show the connection between childhood sexual abuse and anorexia nervosa is not as
strong as that observed in bulimia nervosa; however, child sexual abuse remains a very important
risk factor for all eating disorders including anorexia nervosa.
Two studies showed that a history of childhood sexual abuse was present in 47-50% of patients
with bulimia nervosa and a less strong connection between childhood sexual abuse and anorexia
nervosa when compared with that of bulimia nervosa (23-27%); In addition, a history of childhood
maltreatment in patients with bulimia nervosa is associated with comorbid depressive disorders,
anxiety disorders, and borderline personality disorder.
In all eating disorders there is an increased genetic heritability and frequency of a family history. A
family history of ‘leanness’ or ‘thinness’ or ‘elite athleticism’ (option D) may be associated with
anorexia nervosa, whereas bulimic eating disorders are associated with a personal or family history
of obesity (option C). Although these risk factors are specific to anorexia nervosa and bulimia
nervosa, they are less likely to be trigger the eating disorder compared to sexual child abuse.
Substance abuse (option E) for weight reduction has been frequently observed among patients
with anorexia nervosa and bulimia nervosa as a consequent of the disorder, but it is not the
underlying etiology.
Alcohol abuse (option A) does not seem to increase the risk of developing anorexia/bulimia
nervosa.

How well did you know this?
1
Not at all
2
3
4
5
Perfectly
33
Q

A 67-year-old woman comes to your GP clinic 6 weeks after her husband passed away due to
prostate cancer. She is sad and tearful and complains that she cannot sleep at night and is also
afraid of the dark. She mentions similar feelings 32 years ago after she had a still birth. Which one
of the following will help with prompt relief of her symptoms?
A. Venlafaxine.
B. Electroconvulsive therapy (ECT).
C. Olanzapine.
D. Temazepam.
E. Citalopram.

A

Correct Answer Is D
Given the symptoms and their duration, a normal grief reaction is the most likely explanation to this
woman’s problem.
Sadness, despair, tearfulness, decreased sleep, decreased appetite, and decreased interest in life
and the world are present as common findings in normal grief reaction. Visual and auditory
hallucinations of the deceased person are common and may lead the bereaved person to fear he or
she is ‘going crazy’. Guilt and shame are not common in normal grief reaction, yet possible.
Suicidality is not usually a concern
Patients with normal grief reaction return to normal social functioning within 2 months; the
symptoms, however, might last up to one year with waxing and waning.
Conventional support (e.g., family, social, religious, social) are usually sufficient to help the patient
over her grief. Although there is normally little place for pharmacotherapy, anxiolytics and
hypnotics can provide prompt relief where indicated due to excessive insomnia and anxiety. A
short course of benzodiazepines (< 7 days) can be used to help with the insomnia and anxiety.
The patient has no psychotic feature necessitating antipsychotics such as olanzapine (option C).
Antidepressants such as venlafaxine (option A) and citalopram (option E) might be indicated in
patients with complicated grief or grief associated with major depression, none of which is the
case here.
ECT (option B) might very infrequently be indicated in selected patients with complicated grief
reaction.

How well did you know this?
1
Not at all
2
3
4
5
Perfectly
34
Q

A 32-year-old woman presents to your practice with complaint of back pain, felt between her
shoulder blades. Her past medical history is remarkable for anorexia nervosa for 16 years. On
examination, she is severely emaciated, weighs 38 kg and has a body mass index (BMI) of 14
kg/m2
. Palpation of the interscapular region elicits tenderness over the vertebral column. Which
one of the following could be the most likely diagnosis?
A. Vertebral compression fracture.
B. Pneumonia.
C. Pulmonary embolism.
D. Pericardial effusion.
E. Metastatic bone lesion.

A

Correct Answer Is A
Patients with longstanding anorexia nervosa (>6 months) are at significantly increased risk of
pathological fractures due to osteopenia and osteoporosis caused by malnutrition and estrogen
deficiency.
Osteopenia and osteoporosis is a complication of long-standing anorexia nervosa. These changes
are often treated once anorexia nervosa are successfully treated and adequate weight gain is
obtained. Vertebral column is the most common site of pathologic fractures due to osteoporosis.
In this patient with long-standing anorexia nervosa, the pain and tenderness over thoracic vertebrae
makes compression fracture the most likely diagnosis.
Pneumonia (option B), pulmonary embolism (option C), and pericardial effusion (option D) may be
associated with referred pain to the back, but there is be no tenderness.
Metastatic bone lesions (option E) can cause pathologic fractures; however, with the presence of
anorexia nervosa as a significant risk factor on one hand, and no pointers towards an underlying
primary malignancy on the other, metastatic bone disease is less likely.

How well did you know this?
1
Not at all
2
3
4
5
Perfectly
35
Q

Which one of the following is a common feature seen in anorexia nervosa?
A. Bradycardia.
B. Normally-developed secondary sexual characteristics.
C. Calloused knuckles.
D. Hirsutism.
E. Heat intolerance.

A

Correct Answer Is A
Of the options, bradycardia is the most common finding in patient with anorexia nervosa.
(Option B) Sexual characteristics in patients with anorexia nervosa are often absent or
underdeveloped depending on time of the onset of the disease.
(Option C) Calloused knuckles are seen in patients with binge eating and purging subtype of
anorexia nervosa where vomiting induction frequently occurs as a compensatory means. It is not
as common as bradycardia.
(Option D) Hirsutism is not a feature of anorexia nervosa; rather such patients have lanugo (the fine
thin hair covering the body) which is different from hirsutism.
(Option E) Cold intolerance (not heat tolerance) is a feature of anorexia nervosa; however, shivering
in the cold is not seen).
TOPIC REVIEW
Different complications associated with anorexia nervosa are as follows:
Endocrine/metabolic
Menstrual dysfunction
Delayed menarche/ secondary amenorrhea
Delayed growth and development
Regression of secondary sexual characteristics
Decreased metabolic rate and decreased resting energy expenditure
Appetite and thirst dysregulation
Low parathyroid hormone levels
Elevated liver function
Anemia
Hypoglycemia
Decreased white cell count
Protein-calorie malnutrition
Vitamin, mineral and essential fatty acids deficiencies
Ketonuria
Cold intolerance
Depressed immune function
Atrophied breasts
Musculoskeletal
Decreased bone mass
Osteopenia
Stress fractures
Decreased muscle mass
Cardiovascular
Cardiac arrhythmias
ECG abnormalities
Bradycardia (HR<60bpm)
Hypotension
Orthostatic hypotension
Decreased left ventricular mass and stroke volume
Congestive heart failure
Cardiac arrest
Peripheral edema
Gastrointestinal
Parotid and submandibular gland hypertrophy (usually in binge eating/purging type)
Delayed gastric emptying
Constipation/obstipation
Hypoactive bowel sounds
Postprandial discomfort, bloating after small meals
Psychoneurological
Depression
Anxiety
Structural brain abnormalities/brain tissue loss
Impaired concentration
Lack of insight
Sleep disturbances
Dermatological / other
Acrocyanosis
Lanugo – fine soft hair
Xerosis (dry, scaly skin)
Brittle hair
Hair loss
Calloused knuckles (in binge eating/purging type where vomiting induction occurs as a
compensatory behavior)
Eroded enamel (in binge eating/purging type)
Negative signs
Normal fundi or visual fields
No organomegaly
No lymphadenopathy

How well did you know this?
1
Not at all
2
3
4
5
Perfectly
36
Q

A 19-year-old woman with a two-year history of anorexia nervosa is brought for assessment. She is extremely thin, weighs 37.5 kg and has a body mass index of 15 kg/m2. Her menses started at 14 years of age, but stopped one year ago. Which one of the following investigations is not indicated
in this patient?
A. Liver function tests (LFTs).
B. Electrocardiogram.
C. Serum gonadotropins (FSH and LH).
D. Electrolytes.
E. DXA scan.

A

Correct Answer Is C.
The following are the investigations to consider in every patients presenting with anorexia
nervosa:
* Serum electrolytes (option D)
* Blood urea nitrogen
* Serum creatinine
* Serum glucose
* Serum calcium, phosphorous, and magnesium
* Thiamine
* Serum albumin
* Liver function tests (option A) (aspartate aminotransferase, alanine aminotransferase, and alkaline phosphatase)
* Internationalized Normalized Ratio (INR)
* Complete blood count (CBC) including differential
* Erythrocyte sedimentation rate
* Thyroid stimulating hormone (TSH)
* Electrocardiogram (ECG) (option B)
* Urinalysis for specific gravity
A dual energy X-ray absorptiometry (DXA) (option E) scan is also indicated for all patients with
anorexia nervosa of longer than 6 months duration. For this patient with 2-year history of anorexia
nervosa, a DXA scan should be performed to assess the presence of osteopenia or osteoporosis.

An echocardiography should be performed on all patients with anorexia nervosa with body mass
index (BMI) of less than 14 kg/m2.
FSH and LH are not diagnostically valuable where diagnosis of anorexia nervosa is clear based on
the presence of emaciation in conjunction with self-induced starvation and intense fear of weight
gain. In such cases, amenorrhea is the result of the underlying malnutrition and estrogen deficiency
(hypopituitary hypogonadism).

Measurement of FSH/LH is only indicated if the diagnosis of anorexia nervosa cannot be clinched through physical exams and history and other cause of amenorrhea such as ovarian failure have been considered.

NOTE - If the diagnosis of anorexia nervosa is not clear, other causes that can present with weight loss, malabsorption, or amenorrhea should be investigated.

The following conditions are
possible causes to consider:
* Neoplasm
* Chronic infections (e.g. tuberculosis or acquired immunodeficiency syndrome)
* Uncontrolled diabetes mellitus
* Hyperthyroidism
* Malabsorption syndromes (e.g., celiac disease)
* Inflammatory bowel disease
* Pregnancy
* Primary ovarian failure
* Polycystic ovarian syndrome
* Pituitary prolactinoma

How well did you know this?
1
Not at all
2
3
4
5
Perfectly
37
Q

A 57-year-old man, who is a chronic alcohol user, is admitted to the hospital after he was found out to have developed chronic liver disease. Three days after the admission, he progressively becomes aggressive and starts to shout at the ward staff and threatening them. Which one of the following is the most appropriate medication to give him?

A. Diazepam.
B. Lorazepam.
C. Haloperidol.
D. Olanzapine.
E. Droperidol.

A

Correct Answer Is B.

With chronic alcohol use in this patient, the presentation is most likely due to alcohol withdrawal syndrome (AWS). AWS is characterized by a range of signs and symptoms including tremor, sweating, nausea and vomiting, anxiety, agitation, headache, diarrhea, tachycardia, delirium, perceptual disturbances, tachycardia and hypertension. Seizures occasionally occur. In 50%, there is only one episode of seizure.

Clinical presentation begins within the first 6 to 24 hours after last ingestion and typically persists 72 hours, but may last for weeks.When medications are indicated for treatment of AWS, benzodiazepines are first-line. Diazepam 10-20 mg is given orally every 2 hours until symptoms subside or the maximum daily dose of 120mg is reached. Diazepam can be given on subsequent days if symptoms persist.

In patients with severe liver disease, however, a short-acting benzodiazepine without active metabolite such as lorazepam or oxazepam should replace diazepam.

Diazepam is first metabolized by hepatic oxidation resulting in production of active metabolite. The drug then undergoes glucuronidation. In the elderly and patients with liver disease, benzodiazepine oxidation is decreased and can results in accumulation of the drug, resulting in excessive sedation and respiratory suppression.

Lorazepam and oxazepam metabolism is minimally affected by age and liver disease; therefore, these drugs appear to be safest choices among the various benzodiazepines for treating AWS in the elderly and patients with liver disease.

Haloperidol and droperidol are used when psychotic features or agitation persists despite treatment with benzodiazepine.

NOTE – Benzodiazepines and, to a lesser extent, haloperidol and droperidol, may worsen the symptoms of hepatic encephalopathy.

How well did you know this?
1
Not at all
2
3
4
5
Perfectly
38
Q

A 73-year-old woman with end-stage renal disease (ESRD), who has been well-controlled on routine
peritoneal dialysis, and has been coping well thus far, suddenly declines to have her dialysis
session today. On examination, she is found to have a temperature of 38.3°C and mild tenderness
in upper abdomen. Which one of the following could be the most likely additional finding in this
patient?
A. Disorientation.
B. Depressed mood.
C. Blunted affect.
D. Delusions.
E. Hypervigilance.

A

Correct Answer Is A
Depression is common among patients with ESRD and on dialysis. It is a major cause of treatment
withdrawal in such patients. However, sudden decline associated with findings of fever and
abdominal tenderness are against depression as the cause of treatment refusal in this patient.
This patient’s fever is a pointer towards infection, as is the abdominal tenderness. Infections may
cause delirium, especially in aging people. For delirium to be the underlying cause to this
presentation, disorientation should be present on examination.
Depressed mood (option B) is an unlikely finding in a patient who has been coping well so far, but
all of the sudden refuses treatment. Blunted affect (option C), delusions (option D) and
hypervigilance (option E) would have been expected in psychotic mental disorders. Such disorders
usually have a more insidious onset making them less likely explanations in this scenario.

How well did you know this?
1
Not at all
2
3
4
5
Perfectly
39
Q

A 37-year-old man presents to your practice with complaint of erectile dysfunction. A few months
ago, he found out that her wife had an extramarital affair and developed low mood. After 5 weeks,
he went to see a doctor and was diagnosed with depression and prescribed sertraline. Recently, he
has started a new relationship but is experiencing erectile dysfunction. Which one of the following
would be the most appropriate option for management of his current problem?
A. Stop sertraline.
B. Switch to fluoxetine.
C. Switch to fluvoxamine.
D. Prescribe sildenafil.
E. Prescribe clomipramine.

A

Correct Answer Is D
Many drugs are capable of producing sexual dysfunction. The reported sexual issues by the
patients include: decreased libido, erectile or ejaculatory problems, and decreased lubrication in
women.
The most important drug classes associated with such problems are: antihypertensives,
antipsychotics, antidepressants and recreational drugs.
Selective serotonin reuptake inhibitors (SSRIs) such as sertraline, fluoxetine, paroxetine,
fluvoxamine, citalopram, etc are all well-known to be associated with sexual dysfunction as one of
their most prominent adverse effects that could lead to noncompliance.
The reported sexual adverse outcomes of SSRIs are:
Decreased libido
Delayed ejaculation (this effect has been used for treatment of premature ejaculation)
Erectile dysfunction
Painful ejaculation
The issue of sexual dysfunction associated with antidepressants is complex. The problem may be
caused by other factors than the medication, while strongly believed otherwise by the patient.
There might be an unrecognizable interaction between the drug and psychological effects. Several
strategies approaches have been proposed
Where the medication is considered drug-induced, several strategies might be tried to reverse the
condition including:
Dose reduction
Switching to another drug from a different drug group
Drug holidays
However, prescribing a phosphodiesterase type 5 inhibitor such as sildenafil or tadalafil in
anticipation of intercourse has become the standard of care for men. In women, these drugs have
shown some benefits such as increased lubrication.
Switching to other SSRIs such as fluoxetine, fluvoxamine, paroxetine, etc is unlikely to resolve the
problem because these drugs are associated with sexual dysfunction as well.
Low-dose clomipramine has proven effective in treatment of premature ejaculation. Recent studies
suggest that it could have an effect in delayed ejaculation; however, sildenafil remains the standard
of care in most patients.
‘NOTE - For patients with depression who develop sexual side effects while on SSRIs, switching to nafazodone, bupropion, or mirtazapine is an alternative. These drugs have minimal, if any, impact
on sexual function.’

How well did you know this?
1
Not at all
2
3
4
5
Perfectly
40
Q

A 48-year-old man presents with complaint of significantly delayed ejaculation for the past 2
months. He has the history of major depression, for which he is on sertraline. Four months ago, the
dose of sertraline was increased to 100mg, daily, after he felt that his previously well controlled
symptoms are returning. At present, he is stable and symptom free. Which one of the following
would be the most appropriate management option regarding his ejaculation problem?
A. Switch to fluoxetine.
B. Stop sertraline.
C. Advise sildenafil in anticipation of intercourse.
D. Reduce the dose of sertraline.
E. Prescribe clomipramine.

A

Correct Answer Is C
Selective serotonin reuptake inhibitors (SSRIs) such as sertraline, fluoxetine, paroxetine,
fluvoxamine, citalopram, etc are all well-known to be associated with sexual dysfunction as one of
their most prominent adverse effects that could lead to noncompliance to the treatment from the
patients.
The reported sexual adverse outcomes of SSRIs are:
Decreased libido
Delayed ejaculation (this effect has been used for treatment of premature ejaculation)
Erectile dysfunction
Painful ejaculation
Strategies have been proposed for dealing with sexual adverse effects of SSRIs, including:
Drug holidays
Dose reduction
Switching to other antidepressant with minimal or no impact on sexual functioning such as
bupropion, mirtazapine, or nafazodone
Use of phosphodiesterase type 5 inhibitors such as sildenafil or tadalafil
Cognitive behavioral therapy
Of these options, however, using a phosphodiesterase type 5 inhibitor such as sildenafil before
intercourse has become the standard of care.
(Option A) Switching to other SSRIs such as fluoxetine, fluvoxamine, paroxetine, etc is unlikely to
resolve the problem because these drugs are associated with sexual dysfunction as well.
(Option B) Cessation of sertraline will result in relapse of the recently-controlled depression.
(Option D) Decreasing the current dose of sertraline in a patient who has just responded the
incremental dose increase due to uncontrolled depressive symptoms is very likely to result in
relapse; therefore, not an appropriate option.
(Option E) Clomipramine is a tricyclic antidepressant used for treatment of premature ejaculation.
Addition of clomipramine will deteriorate delayed ejaculation.

How well did you know this?
1
Not at all
2
3
4
5
Perfectly
41
Q

A 24-year-old woman presents with elevated mood, pressured speech, agitation and flight of ideas over the past 2 weeks. There is no history of drug abuse in the past. The patient is fully cooperative. Which one of the following is the drug of choice in this situation?

A. Sodium valproate.
B. Lithium.
C. Carbamazepine.
D. Haloperidol.
E. Quetiapine.

A

Correct Answer Is B.

The clinical history of this patient is consistent with acute mania, probably due to bipolar I disorder.
Bipolar I disorder is a mood disturbance characterized by manic symptoms of at least one week duration. The symptoms should be significant enough to cause considerable impairment or distress in the level of functioning.
Characteristic features of bipolar disorder include:
* Persistent abnormally elevated mood lasting for at least one week
* Increased self-esteem and grandiosity
* Distractibility
* Excessive involvement in activities
* More talkative than usual – pressured speech is often noticed on examination
* Psychomotor agitation
* Flights of ideas
* Increased sexual activity
* Increase in goal-directed activities

Of the given options, lithium is the drug of choice for both acute treatment and maintenace therapy.
Antipsychotics such as olanzapine, quetiapine (option E), risperidone and aripiprazole can be used in combination with lithium or alone for acute mania. Sodium valproate (option A) in combination with atypical antipsychotics is an alternative in specific patients.
Haloperidole (option D) is used for patients with acute manic features, who need sedation. If the patient is not cooperative and acutely agitated, intramuscular haloperidol would be the best initial management. Carbamazepine (option C) and some other anticonvulsants has been used as a mood-stabilzer but evidence shows it is less effective than lithium or atypical antipsychotics in management of acute mania in bipolar disorder.

How well did you know this?
1
Not at all
2
3
4
5
Perfectly
42
Q

1.

A 32-year-old is brought to the emergency department after she was arrested by the police in a
shopping mall. She is dressed in colorful dresses. According to the police, she was shouting at
everyone in the mall. She was claiming in an excited manner that her mother was virgin and
married God. In the emergency department and during the interview she keeps saying that she
hears the voice of God, and that she is planning to become pregnant from him. Which one of the
following is the most likely cause to her presentation?
A. Antisocial personality disorder.
B. Opium abuse.
C. Alcohol intoxication.
D. Schizophrenia.
E. Borderline personality disorder.

A

Correct Answer Is D
Two points are of diagnostic significance in this case scenario. First is the delusional thought
content evident by strongly-held bizarre false beliefs that the patient’s mother married God, and
that she is planning to be conceived by him. Second is the presence of auditory hallucinations.
These two together makes a psychotic episode the most likely diagnosis. Of the given option, the
only one that can represent these is schizophrenia.
(Option A) Antisocial personality disorder is characterized by continuous antisocial or criminal
acts, inability to conform to social rules, impulsivity, disregard for the rights of others,
aggressiveness, lack of remorse, and deceitfulness. There are no delusions or hallucinations,
unless caused by another reason.
(Option B) Opium abuse is not associated with delusions or hallucinations.
(Option C) Acute alcohol intoxication presents varying manifestations depending on the severity,
and can include slurred speech, nystagmus, disinhibited behavior, incoordination, unsteady gait,
memory impairment, stupor or even coma. Hallucinations and delusions are not typical
presentations.
(Option E) Borderline personality disorder is often characterised by an unstable affect, mood
swings, marked impulsivity, unstable relationships, recurrent suicidal behaviours and self-inflicted
injures, chronic feelings of emptiness and boredom, identity disturbances, and inappropriate anger.
If they are stressed they may become psychotic. They use splitting as their main defence
mechanism.

How well did you know this?
1
Not at all
2
3
4
5
Perfectly
43
Q

A nurse calls you to see a 33-year-old woman, who has just been admitted to the general ward and
apparently is making trouble by arguing with the nursing staff. When you step into her room and
she sees you, she says seductively: ‘Oh, my lovely doctor. I am sure you are a gentleman and
understand my needs. I want a private room with classy and beautiful furniture. I can’t eat hospital
foods because they are rubbish. I need to order French dishes as they are my favorite. I need
access to my makeup artist, and I want my little dog here for comfort. I am sure you are brilliant
and know what I am talking about. You are the one who understands me. These idiot nurses don’t’.
Which of the following conditions is most likely to be present in this woman?
A. Narcissistic personality disorder.
B. Histrionic personality disorder.
C. Schizophrenia.
D. Hallucinations.
E. Schizotypal personality disorder.

A

Correct Answer Is B
The behavioral pattern in this woman is most likely due to histrionic personality disorder.
Remarkable findings are seductiveness and trying to be charming, exaggeration in paying
compliments to you as a person she has met for the first time, and self-centeredness and selfdramatization.
Individuals with histrionic personality disorder exhibit emotionalism, which is a tendency to regard
things emotionally. They are attention seekers and uncomfortable in situations where they are not
the center of attention. Behaviors may include constant seeking of approval or attention, selfdramatization, theatricality (acting out as if they are on the theatre stage), and striking selfcenteredness or sexual seductiveness in inappropriate situations including social, occupational,
and professional relationships beyond what is appropriate for the social context. They may be lively
and dramatic and initially charm new acquaintances with their passion, enthusiasm, apparent
openness, or flirtatiousness. Personal interests and conversations will be self-focused. They use
physical appearance to draw attention to themselves. Emotional expression may be shallow and
rapidly shifting. Their style of speech is excessively impressionistic and lacking in detail.
This patient is also using a splitting defense mechanism by seeing all white or black. Although
splitting is commonly seen in individuals with borderline personality disorder, other behavioral
findings are not consistent with the diagnosis.
(Option A) This patient is unlikely to have a narcissistic personality disorder. Although obvious selfcenteredness in this patient may resemble grandiosity, the characteristic feature in narcissistic
personality disorder, this personality disorder is associated with condescending behavior, while this
woman is trying to be in the center of attention and charming at the same time. This makes
histrionic personality disorder more likely.
(Option D) Patients with personality disorders do not hallucinate. No hallucinations are evident in
the presentation.
(Option C) Schizophrenia is characterized by bizarre delusions and hallucinations, mostly auditory,
none of which are present in this case.
(Option E) A schizotypal personality disorder is characterized by significant difficulty in establishing
and maintaining close relationships with others. A person with a schizotypal personality disorder
may have extreme discomfort with such relationships. Someone with this disorder usually has
cognitive or perceptual distortions as well as eccentricities in their everyday behavior. Individuals
with schizotypal personality disorder often have ideas of reference (e.g., they have incorrect
interpretations of casual incidents and external events as having a particular and unusual meaning
specifically for the person). People with this disorder may be unusually superstitious or
preoccupied with paranormal phenomena outside their subculture’s norms.

44
Q

A 27-year-old woman is about to get discharged from the hospital where she was admitted 2
weeks ago due to a psychological breakdown. When you tell her about the discharge plan, she
says: ‘doctor, you cannot discharge me. If you do, I will hurt myself and you will be responsible for
that. Doctor Barkley knows me very well”. Which one of the following best describes the underlying
cause of her behavior?
A. Narcissistic personality disorder.
B. Histrionic personality disorder.
C. Borderline personality disorder.
D. Factitious disorder.
E. Malingering.

A

Correct Answer Is C
Of the options, borderline personality disorder (BPD) is the best explanation for this scenario. The
excessive fear of being abandoned is one of the key features of BPD. These abandonment fears are
related to intolerance of being alone and the need to have other people around. Frantic efforts are
made to avoid imagined abandonment. Some of these efforts include impulsive actions such as
self-mutilating or suicidal behavior. To this woman, discharge conveys abandonment, and she is
making every effort to prevent it.
(Option A) Narcissistic personality disorder is characterized by a sense of self-importance,
grandiosity, and preoccupation with fantasies of success. Someone with this personality disorder
believes they are special, require excessive admiration, and reacts with anger when criticized. They
lack empathy, are envious of others, and are interpersonally exploitative.
(Option B) Histrionic personality disorder is characterized by colorful, exaggerated behavior and
shallow expression of emotions. People with this disorder use their appearance to draw attention,
are sexually seductive and uncomfortable when they are not the center of attention.
(Option D) Factitious disorder is characterized by the conscious production of signs and symptoms
of medical or mental disorders with the main objective of assuming the sick role and eventually
hospitalization. The motivation behind this behavior is not clear to the patient and occurs at a
subconscious level. This patient is not producing any signs or symptoms.
(Option E) Malingering is also the conscious production of signs and symptoms, but unlike
factitious disorder, a clear gain such as money, avoidance of work, free bed and board, etc. is
intended. Again, this patient is not producing any signs or symptoms.
NOTE – although not an option, dependent personality disorder (DPD) can be another diagnosis.
Individuals with DPD are preoccupied with the need to be taken care of. They have clinging
behavior and are unrealistically worried about abandonment. They feel inadequate and helpless
and avoid disagreements with others. They usually focus dependency on a family member or
spouse and desperately seek substitutes if this person becomes unavailable. Associated
features include self-doubt, excessive humility and humbleness, poor independent functioning,
mood disorders, anxiety disorders, adjustment disorder, and other personality disorders.
Patients with dependent personality disorder lack opinion and are distressed when they must
make decisions for themselves. Studies suggest that over 50% of patients with BPD also meet
the criteria for DPD. The prevalence of these comorbidities could owe to the fact that some of
the features of DPD are very similar to the features of BPD. For example, people with BPD
experience rejection sensitivity. They tend to feel desperate at even the slightest perceived
rejection. Individuals with DPD may react similarly to criticism or perceived abandonment by
loved ones.

45
Q

A 32-year-old woman has brought her 10-week-old male infant to your attention for consultation for
the fifth time since his birth. She complains that the baby is crying too much, is irritable, and very
hard to console. On examination, he looks well with a weight on 90 percentile. The rest of exam is
unremarkable. Which one of the following is the most important thing to look for in history or
clinical findings?
A. Mood.
B. Marital conflicts with her husband.
C. Weight.
D. Previous attitude.
E. Financial issues.

A

Correct Answer Is A
One in five Australian mothers of full-term infants suffers from a perinatal mental health disorder
with depression and anxiety being more encountered. Postpartum blues is quite common, but
often has a benign and self-limiting course. Postpartum psychosis and postpartum obsessions are
other conditions encountered. Psychiatric problems pose both physiologic and psychologic risks to
mother and the baby. Therefore, it is of paramount importance to check for the presence of such
conditions in perinatal period.
Depression and anxiety are common in postnatal period and are associated with poor parenting
and attachment and adverse outcomes. Interestingly, many postnatal women will not identify
themselves as depressed or anxious; rather they may visit for “other” reasons such as an
inconsolable or unsettled babies (such as in this case). In such cases, healthcare professionals
such as GPs or Child Health Nurses should routinely inquire about mood, anxiety, and how the
woman is coping, and if deemed necessary, use Edinburgh Postnatal Depression Scale (EPDS)
questionnaire as a screening tool.
Marital conflicts, support from husband or partner, or other issues that might have triggered or
exacerbated such a psychological breakdown should be inquired judiciously if indicated, but
probing for mood problems, anxiety or other psychiatric issues take precedence over such matters.
Previous attitude is another important point to be taken into consideration during evaluation. A
woman may have a previous history of depression or anxiety posing her at greater risk of
recurrences, but more important is her current condition.
Weight loss, anorexia and decreased sleep time are frequently seen in depressed persons. Weight
gain, an increase in appetite and increased sleep time are atypical features in some depressed
patients, the presence of which is associated with an unfavorable outcome. Weight changes may
be seen in depression, but it could be also physiologic or caused by other medical conditions.
NOTE – Frequent visits to the GP or obstetrician can signal underlying depression.
Common signs and symptoms of mental health problems which can be used to guide questioning
at clinical interview include the following:
Depressed / irritable / anxious mood most of the day nearly every day
Mood swings
Loss of interest, pleasure or motivation in usual activities
Problems with thinking clearly, concentration, memory, making decisions
Negative thoughts (death, disaster)
Suicidal ideation or plans
Feelings of worthlessness or guilt; low self-esteem
Unusual change in weight or appetite
Significant change in sleep
Significant fatigue or loss of energy
Significant speeding up or slowing down of psychomotor activity
Excessive anxiety, panicky feelings, worry or rumination
Avoidance (of people, places, things, activities)
Misuse of alcohol or other substances
Very unusual behavior or thinking: psychotic symptoms

46
Q

A 28-year-old woman presents 3 months after giving birth to her daughter. She is concerned about her ‘silly’ thoughts of harming her baby. This has led her to avoid her baby and keeping her away from her. When you ask her to give you an example of how she is afraid of harm to the baby, she says ‘I am afraid my baby would fall off my hand if I lift her’. Which one of the following could be the most likely diagnosis?

A. Postpartum depression.
B. Postpartum blues.
C. Postpartum obsession.
D. Postpartum psychosis.
E. Overvalued ideas.

A

Correct Answer Is C

The history is completely consistent with postpartum obsessions as the most likely diagnosis.

Women are at increased risk for obsessive-compulsive disorder (OCD) during or following pregnancy, including new-onset OCD, recurrent OCD, or exacerbation of a chronic OCD.

The obsessional thinking and compulsive behaviors often focus on the pregnancy or baby. In postpartum period, obsessive thoughts or mental images of harming and fears of contamination of the baby are common. Examples of such thoughts or images include dropping the baby onto the floor, drowning the baby in bathtub, throwing the baby out of the window ,crushing the baby’s skull, microwaving the baby, etc.

Prenatally, obsessions are often about fears of fetal death or contamination. An example of contamination is a mother’s belief that she is infected and if she continues the pregnancy, the infection will pass on to her baby too.

Compulsive behaviors may include the mother’s repeated requests for ultrasound to check fetal
well-being prior to birth, or subsequent avoidance of touching the baby, or repeated washing or hanging diapers.
Other examples are shown in the following table:

By definition, patients with OCD have insight into their obsessions, recognizing them as intrusive and inappropriate, like in this women, by describing her thoughts as ‘silly’; however, their degree of insight can vary. When severe obsessive thoughts persist over a long period of time, they can resemble fixed delusional thinking. On very rare occasions, these thoughts can progress to psychosis, and the patient believes the ideas/thoughts are real.

NOTE - The likelihood that aggressive obsessions toward the baby results in harmful behaviors in
not known, but generally is believed to be very small.
47
Q

Which one of the following is the treatment of choice for and acute manic episode of bipolar
disorder during the first trimester of pregnancy?
A. Carbamazepine.
B. Lithium.
C. Clozapine.
D. Sodium valproate.
E. Quetiapine.

A

Correct Answer Is B
Of the options, lithium is both the most effective and safest medication to be used for treatment of
acute mania during pregnancy.
Despite recent controversy about lithium’s efficacy, a series of meta-analyses confirms its value for
both acute episodes and for prophylaxis, and lithium remains the cornerstone of therapy in patients
with bipolar disorder (BPD).
Although lithium use during the first trimester of pregnancy has been reported to be associated
with fetal cardiovascular anomalies (e.g. Epstein’s anomaly) and midfacial and other defects,
maternal benefits substantially outweigh the risks of fetal anomaly. Generally speaking, lithium can
be used rather safely during all trimesters of pregnancy.
However, if a pregnant woman has been exposed to lithium during the first trimester, an ultrasound
and echocardiogram should be performed at 16-20 weeks gestation to exclude fetal anomalies,
especially cardiac ones.
Lithium is even safer during the second and third trimesters, because cardiogenesis has already
occurred and the risk of fetal malformations is significantly decreased. If lithium has been tapered
during the first trimester or before the conception, it can be safely resumed in second or third
trimester if indicated.
Lithium requirements increase in the third trimester; however, dose reduction by 25% is
recommended before delivery to prevent possible neonatal toxicity that can present with a floppy
baby syndrome (hypotonia, cyanosis, poor suckling). The dose should then be increased after
delivery.
NOTE - Lithium is contraindicated in breast-feeding women. Renal clearance of lihtium in infant is
low and may result in high levels of lithium and life-threatening toxicity.
(Option A) With a few exceptions, the quality of the randomized controlled trials (RCTs) of
carbamazepine in acute mania has been surprisingly poor; therefore, it is not an appropriate option
for treatment of acute mania in this patient.
(Option D) Sodium valproate has significant teratogenic effect and should be avoided during
pregnancy.
(Options C and E) Clozapine or mirtazapine is not indicated or useful for treatment of an acute
manic episode in bipolar disorder.
NOTE – A recent meta-analysis has confirmed that the most efficacious acute treatments for
mania are the antipsychotics rather than the traditional ‘mood stabilizers’ lithium, valproate and
carbamazepine. Specifically, that report found that the preferred options – after taking into account
both efficacy and tolerability – were risperidone, olanzapine and haloperidol. An added benefit of
haloperidol in acute management of a pregnant woman is the rapid-onset therapeutic effect
compared to a delayed effect of other mood stabilizers. These drugs, on the other hand, are
associated with potential adverse effects such as metabolic syndrome with olanzapine and
extrapyramidal side effects with haloperidol, requiring a careful balance between risks and benefits,
often in consultation with an expert in the field. Generally, where antipsychotics (as mentioned
above) are among options, it is safest to choose them as the preferred treatment options and
correct answer for a pregnant woman, especially during the first trimester.

48
Q

Which one of the following is the treatment of choice for an acute manic episode of bipolar
disorder during the third trimester of pregnancy?
A. Carbamazepine.
B. Lithium.
C. Clozapine.
D. Sodium valproate.
E. Mirtazapine.

A

Correct Answer Is B
Of the given options, lithium is the only acceptable one in terms of both safety and efficacy during
pregnancy. However, it is contraindicated in breast feeding woman because reports suggest that
there is low renal clearance of lithium in infants resulting in toxic levels of lithium. This may result
in life-treathening infantile complications.
Reportedly, lithium use during the first trimester of pregnancy has been associated with
cardiovascular anomalies (e.g. Epstein’s anomaly), midfacial defects, and other defects;
nonetheless, the benefits to the mother substantially outweigh the risks of fetal anomaly. However,
if a pregnant woman has been exposed to lithium during the first trimester, an ultrasound and
echocardiogram should be performed at 16-20 weeks gestation to exclude fetal anomalies,
especially cardiac ones. Lithium is even safer during the second and third trimesters when
cardiogensis has already occurred and the risk of fetal malformations is significantly decreased. If
lithium has been tapered during the first trimester or before the conception, it can be safely
resumed in second or third trimester if indicated.
Lithium requirements increase in the third trimester; however, dose reduction by 25% is
recommended before delivery to prevent possible neonatal toxicity presenting with floppy baby
syndrome (hypotonia, cyanosis, poor suckling). The dose should then be increased after delivery.
(Option A) Carbamazepine has been shown poor efficacy in controlling an acute manic episode in
bipolar disorder.
(Options C and E) Clozapine or mirtazapine is not indicated or useful for treatment of an acute
manic episode in bipolar disorder.
(Option D) Sodium valproate has significant teratogenic effect and should not be used during
pregnancy.
NOTE - A recent meta-analysis has confirmed that the most efficacious acute treatments for mania
are the antipsychotics rather than the traditional ‘mood stabilizers’ lithium, valproate and
carbamazepine. Specifically, that report found that the preferred options – after taking into account
both efficacy and tolerability – were risperidone, olanzapine and haloperidol.
An added benefit of haloperidol in acute management of a pregnant woman is the rapid-onset
therapeutic effect compared to a delayed effect of other mood stabilizers. These drugs, on the
other hand, are associated with potential adverse effects such as metabolic syndrome with
olanzapine and extrapyramidal side effects with haloperidol, requiring a careful balance between
risks and benefits, often in consultation with an expert in the field.
Generally, where antipsychotics (as mentioned above) are among options, it is safest to choose
them as the preferred treatment options and correct answer for a pregnant woman, especially
during the first trimester or if the woman desires to breastfeed.

49
Q

Which one of the following is the treatment of choice for an acute manic episode of bipolar
disorder in a breastfeeding woman?
A. Carbamazepine.
B. Lithium.
C. Clozapine.
D. Sodium valproate.
E. Lamotrigine.

A

Correct Answer Is D
Postpartum period is a critical time during which a woman is at increased risk of emotional
challenges. The risk is particularly higher in those with bipolar disorder. In such women,
postpartum period is the time they may experience exacerbations of bipolar disorder in forms of
mania, depression and psychosis.
Recurrence rate of BPD in the first 3 to 6 months is 20-50%. Recurrence rates without treatment
may be as high as 70%. Symptoms often develop rapidly and may occur in late pregnancy or within
the first few days to weeks after delivery.
Lithium has been classically used for treatment of both manic and depressive espisodes as well as
maintenance therapy in patients with bipolar disorder. It is considered safe for use in pregnant
women, especially in the second and third trimesters of pregnancy. There are concerns regarding
Epstein anomaly as well as other congenital defects if used in the first trimester. The absolute risk
is insignificant though. Recently, haloperidol, risperidone and olanzapine have been proved more
effective than lithium and other mood stabilizers for acute treatment of mania in bipolar disorder.
There are concerns of adverse effects such as extrapyramidal effects of haloperidol as a firstgeneration antipsychotic and weight gain and metabolic derangements with risperidone and
olanzapine as atypical antipsychotics. Sodium valproate is another drug used for treatment of
manic episodes in bipolar disorder; however, it is not an effective treatment if psychotic features
such as hallucinations or delusions are present.
On making a decision as to whether which drug is prescribed, the clinician should balance risks
and benefits of each, often in consultation with an expert in the field, and also consider specific
circumstances such as pregnancy and breastfeeding.
Of the options, sodium valproate is the only accpetable one for treatment of this patient. It hase
been proven as efficacious as lithium for acute management of mania. Although its use is
contraindicated in pregnancy, it can be effectively used in breastfeeding women. However, if there
are psychotic features such as delusions or hallucinations, sodium valproate will not be an
appropriate option either.
(Option A) Thus far, carbamazepine has not been shown to be associated with harm to the
breastfed baby, but studies suggest it is not more effective than placebo in management of an
acute manic episode.
(Options B and C) Lithium and clozapine are absolutely contraindicated in breastfeeding women.
Furthermore, clozapine is not recommended or approved for treatment of mania, depression or
maintenance therapy for bipolar disorder.
(Option E) Lamotrigine has being increasingly used to prevent relapses of bipolar depression;
however, the effects for treatment or prevention of manic episodes are minimal. It is also secreted
in breast milk and should not be used in breastfeeding woman due to lack of data regarding its
safety profile.
NOTE - If a pregnant woman has been stable on lithium or another mood stabilizer or
antipsychotic, with no contraindication in pregnant women, but not recommended during
breastfeeding, withdrawal from the drug or switching to another drug that is safe in breastfeeding
is not advisable, as any alteration in medication(s) may result in relapse of bipolar disorder. In such
circumstance, the most appropriate approach is continuation of the drug, avoiding breastfeeding,
and feeding of the baby with formula.

50
Q

A man brings his wife to your practice for your opinion, because she has started excessive cleaning of the house and alcohol drinking. She has a lot of fights with him and flirts with his friends. During consultation, she is initially flirtatious, but then becomes angry and starts talking to you very rudely. According to her husband, 4 days ago she was quite normal. Which one of the following is the most likely diagnosis?
A. Obsessive compulsive disorder.
B. Hypomania.
C. Histrionic personality disorder.
D. Passive-aggressive personality disorder.
E. Mania.

A

Correct Answer Is B
The patient seems to have some characteristic features of a manic episode. According to DSM-IV, diagnostic criteria for a manic episode include:
A. A distinct period of abnormally and persistently elevated, expansive, or irritable mood, lasting at least 1 week (or any duration if the symptoms are severe enough to result in hospitalization).
B. during the period of mood disturbance, three (or more) of the following symptoms have persisted (four if the mood only irritable) and has been present to a significant degree:
1. Inflated self-esteem or grandiosity
2. Decreased need for sleep (e.g. feels rested after only 3 hours of sleep)
3. More talkative than usual or pressure to keep talking
4. Flight of ideas or subjective experience that thoughts are racing
5. Distractibility (i.e. attention too easily drawn to unimportant or irrelevant external stimuli)
6. Increase in goal-directed activity (either socially, at work or school, or sexually) or
psychomotor agitation
7. Excessive involvement in pleasurable activities that have a high potential for painful
consequences (e.g. engaging in unrestrained buying sprees, sexual indiscretions, or foolish
business investments)
C. The symptoms do not meet criteria for a Mixed Episode.
D. The mood disturbance is sufficiently severe to cause marked impairment in occupational or in usual social activities or relationships with others, or to necessitate hospitalization to prevent harm to self or others, or there are psychotic features.
E. the symptoms are not due to the direct physiological effects of a substance (e.g. a drug of abuse, a medication or other treatments) or a general medical condition (e.g. hyperthyroidism).

NOTE - Manic-like episodes that are clearly caused by somatic antidepressants treatment (e.g. medication, light therapy) should not count toward a diagnosis of bipolar disorder.
The patient’s symptoms do not appear to be severe enough to need hospitalization and the duration of the symptoms has not lasted for at least 7 days; therefore, hypomania will be the most likely diagnosis for now.
A hypomanic episode has the same symptoms of a manic episode with 2 important differences:

  1. The mood disturbances usually are not severe enough to cause problems with the person
    working or socializing with others, or to require hospitalization
  2. There is no psychotic feature

A hypomanic episode is characterized by a distinct period of persistently elevated, expansive, or
irritable mood, lasting at least 4 days and present for most of the day nearly every day. This
hypomanic mood is clearly different from the person’s usual mood.
(Option A) In this scenario, engagement in excessive cleaning is different from what happens in
obsessive-compulsive disorder which is repetitive cleaning (that might not be
excessive). Obsessive-compulsive disorder (OCD) is characterized by recurrent intrusive thoughts,
images, or urges (obsessions) that typically cause anxiety or distress, and by repetitive mental or
behavioral acts (compulsions) that the individual feels driven to perform, either in response to an
obsession or according to rules that he or she believes must be applied rigidly.
(Option C) Some symptoms of hypomania overlap with those of histrionic personality disorder,
including shallow emotions and flirtatiousness. However, flirting in hypomania is due to increased
sexual drive, whereas, flirting in histrionic personality disorder is for attention seeking without any
intention for sex. Features of a person with histrionic personality disorder include being selfdramatic, egocentric, immature, seductive and attention seeking.
(Option D) Features of passive-aggressive personality disorder include childish stubbornness,
argumentativeness, egocentricity, deliberate inefficiency, and hypercriticism of authority figures.
The scenario is not consistent with passive aggressive personality disorder.
(Option E) A minimum duration of 7 days is required for symptoms in order to make a diagnosis of
mania. With symptoms present for only 4 days this patient has hypomania for now.

51
Q

A man brings his wife to your practice for consultation, because in the past 5 days has been excessively cleaning the house and drinking alcohol. She has a lot of fights with him and flirts with his friends. During consultation, she initially is flirtatious, but then becomes angry and starts talking to you very rudely. According to her husband, she was previously quite normal. Which one of the following is the next best step in management?
A. Lithium.
B. Lamotrigine.
C. Olanzapine
D. Carbamazepine.
E. Diazepam.

A

C. Olanzapine

The scenario fulfills the classic definition of hypomania. In fact this woman can have her first hypomanic episode of a bipolar disorder (type II) disorder. A hypomanic episode is characterized by a distinct period of persistently elevated, expansive, or irritable mood, lasting throughout at least 4 days and present for most of the day nearly every day. This hypomanic mood is clearly different from the person’s usual mood.

**Features seen in hypomania (as well as mania) include inflated self-esteem and grandiosity, decreased need for sleep, more talkativeness than usual and pressures speech, distractibility, increased goal-directed activities, increased sexual drive, and excessive involvement in pleasurable activities that have a high potential for painful consequences (e.g. engaging in unrestrained buying sprees, sexual indiscretions, or foolish business investments). **

Hypomania is distinct from mania by milder magnitude of symptoms and less interference with
level of social and/or occupational. Patients with hypomania are unlikely to require hospitalization.

Some symptoms of hypomania overlap with those of histrionic personality disorder, including shallow emotions and flirtatiousness. However, flirting in hypomania is due to increased sexual drive, whereas flirting in histrionic personality disorder is for attention seeking without any intention for sex. In this scenario, engagement in excessive cleaning is different from what occurs in obsessive compulsive disorder which is repetitive cleaning (that might not be excessive).

For patients with hypomania and mild to moderate manic episodes, monotherapy with risperidone or olanzapine is currently the first-line recommendation. Reasonable alternatives are lithium, sodium valproate, carbamazepine, aripiprazole, quetiapine, ziprasidone and haloperidol.

(Options A and D) According to a recent metanalysis, the antipsychotics risperidone and olanzapine are superior to lithium or carbamazepine for treatment of acute mania. Lithium, however, may be added for maintenance therapy.

(Option B) Lamotrigine is an anticonvulsant successfully used for prevention of depression episodes in patients with bipolar disorder. Its effect in treatment or prophylaxis of mania or hypomania has not proved satisfactory.

(Option E) Benzodiazepines are considered in manic patients who cannot tolerate any of other mood stabilzers. With the wide range of mood stabilizers available, monotherapy with benzodiaepines is almost never indicated; however, lorazepam and clonazepam are used in short term and in combination with mood stabilizers for rapid resolution of mania. Diazepam is not used for such purpose.

52
Q

Which one of the following is not associated with body dysmorphic disorder?
A. Anorexia nervosa.
B. Delusional disorders.
C. Social phobia.
D. Depression.
E. Decrease alcohol intake.

A

E. Decrease alcohol intake.

Body dysmorphic disorder is characterized by the belief that some body part is abnormal, defective, or misshapen. Patients are preoccupied with an imagined defect in their appearance or excessive concern about even the slightest physical anomaly. This preoccupation causes significant distress or impairment in social, occupational, or other areas of function.

This disorder affects women more than men, typically between ages of 15 and 20 years.

The characteristic features of body dysmorphic disorder are as follow:

  • Preoccupation with alleged physical flaws - of all body parts, imagined facial flaws are most common
  • Constant mirror checking
  • Attempt to hide the imagined deformity
  • Housebound and avoiding social situations
  • Impaired levels of functioning

The following comorbid conditions are often found as an association with body dysmorphic disorder:

  • Depression
  • Delusional disorders
  • Obsessive compulsive disorder
  • Social phobia
  • Increased alcohol intake

Body dysmorphic disorder is not associated with decreased alcohol intake.

53
Q

A 37-year-old woman is in your office accompanied by her husband for consultation. The man is concerned about her wife because for the past 8 months, she has been progressively edgy at home. She is always fighting with their 8-year-old daughter as to why she is not doing her homework, and extremely anxious and distressed about her health. She worries a lot about financial issues and is concerned about not having money for the future despite the stable financial state of the family. She feels tense and is easily irritated. She has lost all her interest in hobbies and leisure time, but denies depressed mood. Which one of the following is the most likely diagnosis?

A. Major depression.
B. Generalized anxiety disorder.
C. Adjustment disorder.
D. Dysthymic disorder.
E. Obsessive compulsive disorder.

A

B. Generalized anxiety disorder.

The scenario is a typical presentation of general anxiety disorder (GAD). GAD is characterized by experiencing excessive anxiety and worry, often about health, family, financial issues, or work. This worrying goes on every day, possibly all day long. It negatively interferes with social activities and is associated with impaired function at work, school, or family.

GAD is diagnosed in adults when they experience at least 3 of the following symptoms on more days for at least 6 months:

  • Restlessness or feeling keyed up or on edge
  • Being easily fatigued
  • Difficulty concentrating or mind going blank
  • Irritability
  • Muscle tension
  • Sleep disturbance (difficulty falling or maintaining, unrefreshing sleep, etc.)

To make the diagnosis in children, only one symptom is required.

(Option A) Major depression is associated with depressed mood which is not present in this woman. However, depression and dysthymic disorder can be seen in patients with GAD.

(Option C) For adjustment disorder to be the diagnosis, one provocative event should be present. There is no recent stressor in the history.

(Option D) Dysthymic disorder is characterized by mild to moderate symptoms of depression lasting for at least 2 years.

(Option E) OCD is associated with obsessions and impulse. There is no clue in the history to suggest such diagnosis.

54
Q

Aida is a pregnant patient of yours, who has presented for an antenatal visit in her first trimester. This is her second pregnancy. Her previous pregnancy, 3 years ago, was followed by development of postpartum psychosis, for which she was successfully treated with risperidone. The medication was then tapered and stopped. She has been stable since. She wants to know if there is anything
she can do to prevent postpartum psychosis in this pregnancy. Which one of the following would be the most appropriate advice?

A. Review her later in pregnancy.
B. Start prophylactic dose of antidepressants.
C. Start her on antipsychotics now.
D. Start her on risperidone if she develops postpartum psychosis again.
E. Start risperidone immediately after delivery.

A

E. Start risperidone immediately after delivery.

With either of the following conditions present in history, a woman is at increased risk of postpartum psychosis:

  • History of postpartum psychosis
  • History of bipolar disorder
  • Family history of postpartum psychosis
  • First pregnancy
  • Recent discontinuation of lithium or other mood stabilizers

The risk of recurrence in woman with previous postpartum psychosis is high in subsequent pregnancies. The period during which the recurrence risk of post-partum psychosis is the highest, is the first 28 days after delivery. The symptoms generally appear within days or even hours of delivery.

It is strongly recommend that, in a woman with history of post-partum psychosis, prophylactic mood-stabilizers be started immediately after delivery to prevent relapse. Choice of mood stabilizer depends on different factors including breastfeeding, or previous response.

Aida had desirable response to risperidone in the past; therefore, commencement of risperidone immediately after delivery would be the most appropriate option for her. Risperidone has not been associated with
increased risk to the breastfed baby; however, the data are limited. This approach prevents unnecessary treatment of an asymptomatic woman and potential adverse outcomes associated with psychotropic medications.

(Option A) This woman should also be closely monitored for development of psychiatric problems throughout pregnancy; however, this does not prevent postpartum psychosis.

(Option B) Antidepressants, either therapeutically or prophylactically, have no effect on postpartum psychosis. A mood stabilizer is indicated in such situations.

(Option C) In an asymptomatic woman, who is currently stable, initiation of prophylactic treatment is not likely to be of significance benefit compared to immediately after delivery.

(Option D) With a high recurrence rate as high as 30-50%, waiting for the postpartum psychosis to develop as an indication for starting treatment is not appropriate and prophylactic measures should be contemplated.

55
Q

Which one of the following is not a risk factor for schizophrenia?

A. Genetic susceptibility.
B. Winter birth.
C. Advanced parental age.
D. Fetal hypoxia.
E. High educational achievements.

A

E. High educational achievements.

The genetic linkage in schizophrenia is well-established. Although schizophrenia appears to be multifactorial, genetics plays a significant role.

Winter births are statistically associated with increased occurrence of schizophrenia. This fact may augment the hypothesis that schizophrenia may be associated with viral infections.

Antenatal and perinatal complications such as hypoxia, neonatal sepsis, neonatal jaundice, etc are associated with increased risk of schizophrenia. Advanced parental age at conception is associated with increased risk of development of schizophrenia in offsprings.

High educational achievements are not risk factors for schizophrenia. In fact, they are protective against schizophrenia.

56
Q

A 30-year-old male is found not to interact with people and prefers to stay at home. He has no desire to become involved in any relationships and feels uncomfortable around people. A female psychologist recently saw him at his family’s request, and he wishes he could make friends with her by means of spells and supernatural solutions. Which one of the following options is the most likely explanation for his behavior?

A. Schizoid personality disorder.
B. Schizotypal personality disorder.
C. Paranoid personality disorder.
D. Avoidant personality disorder.
E. Dependent personality disorder.

A

B. Schizotypal personality disorder.

This patient’s clinical and historical features are suggestive of schizotypal personality disorder.

Schizotypal personality disorder is characterized by significant discomfort in social relationships; thought distortions, and eccentricity. Individuals with this disorder are socially isolated and uncomfortable with others.

The most differentiating feature of this disorder from schizoid personality disorder is the presence of particular patterns of thinking such as ideas of reference, persecution, and magical thinking that are absent in schizoid personality disorder. Bizarre preoccupations, odd affection, and peculiar speech are other features. Eccentric attire is another commonly observed feature.

(Option A) Schizoid personality is characterized by social withdrawal, isolation, and emotional coldness. Affected individuals usually remain absorbed and sunken in their thoughts and feelings.

They fear closeness and intimacy with other people. They prefer theoretical speculations (such as math and physics) to practical action. However, the thinking pattern is normal.

(Option C) Paranoid individuals find offense in even the most benign remarks or circumstances. These individuals are often defending an extremely fragile self-concept.

(Option D) A person with an avoidant personality disorder is interpersonally sensitive, is afraid of being criticized, and is constantly concerned about saying or doing something foolish or humiliating. They have an intense desire to connect with others; however, they will never let anyone get close unless they are sure of being liked.

(Option E) Individuals with dependent personality disorder are passive, submissive, and have the fear of abandonment. They need constant reassurance. Dependent patients assume that they are incapable of functioning independently and that being assertive will be experienced by others as aggressiveness.

57
Q

Which one of the following is the preferred medical treatment for obsessive compulsive disorder (OCD)?

A. Selective serotonin re-uptake inhibitors.
B. Tricyclic antidepressants.
C. Serotonin norepinephrine re-uptake inhibitors.
D. Benzodiazepines.
E. Alcohol.

A

A. Selective serotonin re-uptake inhibitors.

Obsessive-compulsive disorder (OCD) is characterized by recurrent obsessions or compulsions that are recognized by the individual as unreasonable.There are two components in OCD:

  • Obsessions: obsessions are anxiety provoking, intrusive thoughts, commonly concerning contamination, doubt, guilt, aggression and sex.
  • Compulsions: peculiar behaviors that reduce the anxiety such as hand washing, organizing, checking, counting, etc.

Abnormalities of serotonin metabolism are most implicated as the underlying cause of OCD. The
prevalence is 2% in general population, and there is a 1:1 male-to-female ratio. The onset is insidious and occurs during childhood, adolescence, or early adulthood. The symptoms are usually of waxing and waning nature. Comorbid conditions such as depression, anxiety, and substance abuse are frequently seen.

OCD is best treated with a specific form of behavior therapy - exposure and preventive response and/or selective serotonin reuptake inhibitors (SSRIs) as the first-line medications. All SSRIs have about the same efficacy. The TCA clomipramine has also been used as a treatment option and was the medication of choice before SSRIs take over.

NOTE - Antipsychotics have been shown to add to the benefits of SSRIs or clomipramine. Although adjunctive therapy with antipsychotics may be considered, they are not used for single therapy in OCD.

(Option C) Serotonin norepinephrine re-uptakes inhibitors (SNRIs) e.g., venlafaxine can be used as an alternative, but SSRIs remain the preferred option.

(Option B) Among tricyclic antidepressants (TCAs), clomipramine is the only one that seems to have an impact on serotonin metabolism and can be used as second-line management in treatment of OCD. The side effects of clomipramine include sedation, urinary retention, confusion, delirium, and hypotension.

(Option D) Occasionally benzodiazepines may be considered for anxiety relief in short term, but they are not effective in treatment of OCD.

(Option E) Although alcohol consumption is associated with decreased compulsive behavior in short-term, its use is not recommended for treatment of OCD due to the high risk of dependence and abuse.

58
Q

A 30-year-old woman presents with complaint of binge eating, after which she often induces vomiting. She has a body mass index (BMI) of 21 kg/m2 . Which one of the following is most likely to be present in her family history?

A. Pica.
B. Family history of obesity.
C. Family history of elite athleticism.
D. Schizophrenia.
E. Substance abuse.

A

B. Family history of obesity.

Based on the history and the BMI, bulimia nervosa is the most likely diagnosis. Both bulimia nervosa and binge eating/purging type anorexia nervosa are associated with episodes of binge eating, followed by compensatory purging with laxative or diuretic use, induction of vomiting, etc. Patients with bulimia nervosa, however, tend to have normal weight or even can be overweight.

Of the given options, a family history of obesity is the most relevant finding in a patient with bulimia
nervosa. In all eating disorders, there is an increased genetic heritability and frequency of a family history. A
family history of leanness or athleticism may be associated with anorexia nervosa; whereas bulimic eating disorders are associated with a personal or family history of obesity.

Sexual abuse (not an option) is also a significant risk factor for development of eating disorders as well as a wide range of other psychiatric disturbances.

(Option A) Pica is defined as persistent ingestion of nonnutritive substances for at least one month at an age for which this behavior is developmentally inappropriate. Personal or family history of pica has not been shown to be a risk factor for eating disorders.

(Option C) Family history of elite athleticism has been proposed as a risk factor for anorexia nervosa.

(Option D) Although abnormal eating patterns can be seen in schizophrenic patients, family history of schizophrenia does not appear to be a risk factor for development of eating disorders.

(Option E) Substance abuse is a comorbid conditions often noticed in family or personal history of patients with eating disorders; this, however, is less likely to be directly related to development of eating disorders.

59
Q

A 25-year-old woman presents to your practice , accompanied by her husband, because the husband believes that she has not been herself in the past few weeks. She also has been suffering from delusional states at times. She is 20 weeks’ pregnant. On examination, she is noticed to have pressured speech and flight of ideas represented by swift changes between irrelevant topics. Which of the following will be the most appropriate treatment option for her?

A. Sodium valproate.
B. Clozapine.
C. Diazepam.
D. Lithium.
E. Carbamazepine.

A

D. Lithium.

The scenario is classic for a manic episode. Features seen in mania (as well as hypomania) include inflated self-esteem and grandiosity, decreased need for sleep, more talkativeness than usual and pressured speech, flight of ideas, distractibility, increased goal-directed activities, increased sexual drive, and excessive involvement in pleasurable activities that have a high potential for painful consequences (e.g. engaging in unrestrained buying sprees, sexual indiscretions, or foolish business investments). Hypomania has the same symptoms but with milder intensity.

The presence of delusions in the scenario, however, is consistent with severe mania. In fact, this woman is very likely to have presented with a manic episode of bipolar disorder.

Mood stabilizers are the cornerstone of management of a manic episode in mild to moderate disease. Mood stabilizers include, the antipsychotic medication, such as haloperidole, risperidone, aripiprazole, quetiapine, ziprasidone, or lithium, or anticonvulsants such as carbamazepine and
sodium valproate.

According to a recent meta-analysis, antipsychotics, especially risperidone, haloperidol and olanzapine (not among options) are recommended as first-line treatment of acute mania in bipolar disorder both in pregnant and nonpregnant patients.

Lithium can be used as well and is the most appropriate option among others, considering the fact that at 20 weeks gestation (second trimester) fetal cardiogenesis has already taken place and the risk of cardiac anomalies, a feared adverse outcome of lithium on the developing fetus, is minimal if any.

Reportedly, lithium use during the first trimester of pregnancy has been associated with cardiovascular anomalies (e.g. Epstein’s anomaly), midfacial defects, and other defects; nonetheless, the benefits to the mother substantially outweigh the risks of fetal anomaly.

Lithium is even safer during the second and third trimesters when cardiogenesis has already
occurred and the risk of fetal malformations is significantly decreased.
Lithium requirements increase in the third trimester; however, dose reduction by 25% is recommended before delivery to prevent possible neonatal toxicity that can present with a floppy baby syndrome (hypotonia, cyanosis, poor suckling). The dose should then be increased after delivery.

(Option A) Sodium valproate has major teratogenic effects and is contraindicated throughout the pregnancy.

(Option B) Although an atypical antipsychotic, clozapine is not among recommended medications for treatment of mania.

(Option C) Benzodiazepines, mostly clonazepam or lorazepam are used as monotherapy for patients with mild to moderate manic or mixed episodes who cannot tolerate lithium, anticonvulsant or antipsychotics. This is very unusual to occur, given the large number of available options for treatment of bipolar disorder. However, benzodiazepines are generally used as adjunctive therapy for treatment of insomnia, anxiety and agitation in patients with elevated mood.

(Option E) Carbamazepine is an antiepileptic medication used as a mood stabilizer in treatment of bipolar disorder. Recent studies, however, has questioned its efficacy. There is also concern about teratogenicity pregnancy due to reported cases of neural tube defects.

60
Q

A 26-year-old woman with background history of difficult-to-treat bipolar disorder is planning to conceive. She has been stable on lithium and quetiapine for the last 6 months. Which one of the following would be the next best step in management?

A. Explain the pros and cons of lithium use during pregnancy and continue the medication.
B. Switch to sodium valproate.
C. Switch to lamotrigine.
D. Start the patient on antipsychotics.
E. Taper and stop lihtium before conception.

A

A. Explain the pros and cons of lithium use during pregnancy and continue the medication.

Although lithium use during the first trimester of pregnancy has been reported to be associated with fetal cardiovascular anomalies (e.g. Epstein’s anomaly) and midfacial and other defects, benefits to the mother substantially outweigh the risks of fetal anomaly.

For pregnant women stable on lithium, one approach can be careful withdrawal of lithium (or other mood stabilizers) to avoid potential risks to the developing fetus. Some authorities discourage this method because of the risk or relapse.

In this patient, however, the history of difficult-to-treat bipolar disorder that has just recently responded to lithium after failed trials of other mood stabilizers makes such approach less favorable. She should be maintained on lithium to avoid relapse. In such situations, the woman should be fully informed of the risks, and decision making left to the patient. If the woman decides to conceive, lithium should be continued throughout the pregnancy and an ultrasound and echocardiogram performed at 16-20 weeks gestation to exclude fetal anomalies, especially cardiac anomalies.

(Option B) Sodium valproate is teratogenic and contraindicated in pregnancy. Switching to sodium valproate can be associated with high risk of relapse as well as more harm to the fetus.

(Option C) Lamotrigine is an option for prevention of bipolar depression. It is not effective in prevention or treatment of manic episodes.

(Option D) According to a recent meta-analysis, antipsychotics, especially risperidone, olanzapine, and haloperidol are superior to other mood stabilizers such as lithium, sodium valproate, and carbamazepine for treatment of manic episodes and are recommended as first-line options for this purpose, but if the risk of relapse is high, lithium should be used concomitantly for a maintained therapeutic response. While this patient is stable on lithium, switching to withdrawal of lithium and commencement of antipsychotics is likely to result in a relapse.

(Option E) As mentioned earlier, stopping medications in a stable patient might be considered, but for this patient such action is can result in a relapse of the disease.

61
Q

A 30-year-old woman decides to conceive while she is stable on lithium for severe relapsing bipolar disorder. Lithium is continued after discussion with the patient. Which one of the following would be most important to consider during pregnancy?

A. Monitoring the liver function.
B. High resolution ultrasound at 16-20 weeks gestation.
C. Increasing the dose of the lithium 25% in late third trimester.
D. Giving a diet rich in vitamin C.
E. Monitoring vitamin B12 level.

A

B. High resolution ultrasound at 16-20 weeks gestation.

The benefits of lithium prophylaxis during pregnancy, in cases of severe bipolar disorder, may outweigh the risks. For pregnant women stable on lithium, one possible approach is careful withdrawal of lithium (or other mood stabilizers) to avoid potential risks to the developing fetus.

In this patient with history of severe relapsing bipolar disorder, however, continuation of the treatment has been decided. Lithium use during the first trimester of pregnancy has been reported to be associated with fetal cardiovascular anomalies (e.g. Ebstein’s anomaly) and midfacial and other defects.

Ebstein’s anomaly is a cardiac defect characterized by downward displacement of the tricuspid valve into the right ventricle. The absolute risk for Ebstein’s anomaly associated with lithium use in the first trimester is approximately 1 in 1000 to 2000 compared with 1 in 20000 in the general population. Despite significant increased risk (10-20 folds), the absolute risk is small (0.05%). Coarctation of the aorta and mitral atresia are other reported cardiac anomalies.

If a pregnant woman has been exposed to lithium during the first trimester, an ultrasound and echocardiogram should be performed at 16-20 weeks gestation to exclude fetal anomalies, especially cardiac anomalies.

Option A: While the patient is on lithium, regular renal and thyroid and parathyroid function tests as well as serum lithium levels are indicated. Liver function tests are not required to be monitored as lithium does not have an adverse effect on the liver.

Option C: Lithium requirements increase in the third trimester; however, dose reduction (not increase) by 25% is recommended before delivery to prevent possible neonatal toxicity that can present with a floppy baby syndrome (hypotonia, cyanosis, poor suckling). The dose should then be increased immediately after delivery due to increased risk of relapse in postpartum period.

Option D and E: Lithium is not associated with vitamin C or B12 deficiency. Supplementation of these vitamines are not recommended while the pregnant woman is on lithium, unless indicated for other reasons.

62
Q

A 27-year-old woman is being assessed in your general practice after her marriage was broken 8 weeks ago. One year ago, she was referred for psychotherapy from work due to frequent conflicts with colleagues and being easily irritated and shouting at them. She complains that after finishing work, she is not able to relax at home. Which one of the following options is more likely to be the cause of this presentation?

A. Bipolar II disorder with depression.
B. Cyclothymic disorder.
C. General anxiety disorder.
D. Major depression.
E. Borderline personality disorder.

A

E. Borderline personality disorder.

Of the given options, borderline personality disorder is most consistent with the history. Important clues to this diagnosis are frequent conflicts with colleagues, being irritated easily, bursts of anger represented by shouting at colleagues, failed marriage, and not being able to relax.

Borderline personality disorder (BPD) is characterized by instability of interpersonal relationships, self-image, and emotions and by impulsivity.

BPD has three components:

1- Impaired relatedness - unstable relationships with others, identity disturbance, and chronic emptiness

Patients with BPD usually have stormy relationships, especially in their close relationships. At one moment a friend or romantic partner may be viewed as trusted or an ideally perfect person, and then this same individual can suddenly be seen as cruel, betraying, very limited, and damaged. When the idealized person is present and supportive, the patient feels strong and solid.

However, if the support person leaves (or is unable to meet the patient’s needs) for a limited period of time, or if the patient thinks that the support person is about to leave, the patient can immediately become angry, demeaning, depressed, hopeless, and suicidal.

Splitting, which is rigidly classifying other as good as bad, is common among patients with BPD and can lead the patient to shift between extreme points of view and to selectively attend to information in a way that confirms his or her current opinion. This tendency to “split” can impact treatment.

Patients with BPD often interpret neutral events, words, or faces as ‘negative’; therefore, the patient is prone to misinterpret relatively minor disagreements or adverse events as a sign that the caretaker wants to terminate the relationship. The patient often reacts with anger or threats of self harm, which can gradually alienate the support person, and result in their real tendency to end the relationship. This heightened sensitivity to actual or perceived rejection can lead patients to feel more comfortable or secure with a transitional object, such as a pet or stuffed animal, than with other people.

2- Affective dysregulation - affective lability, excessive anger, and efforts to avoid abandonment

Patients with BPD experience repeated and marked mood changes throughout the course of a single day, with moment to moment fluctuations often triggered by environmental stressors. Periods of euthymia alternate with intense, episodic dysphoria that includes depression, anxiety, and irritability. Angry outbursts triggered by dissatisfaction with a caregiver are often followed by feelings of shame, guilt, and worthlessness.

3- Behavior dysregulation - impulsivity, suicidality, and self-injurious behavior

Impulsive and self-damaging behavior is common and can take many forms. Patients abuse substances, binge eat, engage in unsafe sex, spend money irresponsibly, and drive recklessly. In addition, patients can suddenly quit a job that is promising or end a relationship that has the potential to last, thereby sabotaging their own success. Impulsivity can also manifest with immature and regressive behavior.

The following are characteristic features of BPD and their prevalence:

  • Affective instability – 95%
  • Inappropriate anger – 87%
  • Impulsivity – 81%
  • Unstable relationships – 79%
  • Feelings of emptiness – 71%
  • Paranoia or dissociation – 68%
  • Identity disturbance – 61%
  • Abandonment fears – 60%
  • Suicidality or self-injury – 60%

Option A: There are no symptoms indicating either mania/hypomania or depression to suggest bipolar disorder. Bipolar disorder is often characterized by shifting episodes of mania/hypomania and depression, none of which are present in the history.

Option B: Cyclothymic disorder presents with many periods of depressed mood and many episodes of hypomanic mood for at least 2 years. Interestingly, the condition frequently coexists with BPD. In the scenario, however, there are no clues to indicate depression or mania.

Option C: Generalized anxiety disorder (GAD) is characterized by excessive, poorly controlled anxiety about life circumstances that continues for more than 6 months. To establish the diagnosis, both physiologic and psychologic symptoms should be present.

Option D: Major depression presents with at least a 2-week course of the following symptoms:

  1. Depressed mood most of the day
  2. Anhedonia during most of the day
  3. Significant weight loss or gain
  4. Insomnia or hypersomnia
  5. Psychomotor agitation or retardation
  6. Fatigue or loss of energy almost everyday
  7. Feelings of helplessness/hopelessness or guilt
  8. Diminished ability to concentrate
  9. Recurrent thoughts about death

Of these symptoms, the first two are necessary for major depression to be present. The history is not consistent with major depression as diagnosis.

63
Q

Which one of the following is not a feature of bipolar disorder?

A. Flight of ideas.
B. Persistently elevated mood for more than one week.
C. Delusions of grandiosity.
D. Decrease in goal directed activities.
E. Impairment of occupational functioning.

A

D. Decrease in goal directed activities.

Features of bipolar disorder include:

  • Inflated self-esteem or grandiosity
  • Decreased need for sleep (e.g. feels rested after only three hours of sleep)
  • More talkative than usual or pressure to keep talking
  • Flight of ideas or subjective experience that thoughts are racing
  • Distractibility (i.e. attention too easily drawn to unimportant or irrelevant external stimuli)
  • Increase in goal-directed activity (either socially, at work or school, or sexually) or psychomotor agitation
  • Excessive involvement in pleasurable activities that have a high potential for painful consequences (e.g. engaging in unrestrained buying sprees, sexual indiscretions, or foolish business investments)

These all must be associated with impairment of social or occupational functioning.

Patients with bipolar disorder have increased (not decreased) goal-oriented activities.

64
Q

Which one of the following drugs is not used in the treatment of cocaine overdose?

A. Diazepam.
B. Aspirin.
C. Diltiazem.
D. Propranolol.
E. Hydralazine.

A

D. Propranolol.

Acute cocaine use is associated with arterial vasoconstriction and enhanced thrombus formation. It causes tachycardia, hypertension, increased myocardial oxygen demand, and increased vascular shearing forces. Chest pain is the presenting symptom in 40% of patients.

The following cardiovascular complications can also occur:

  • Coronary vasoconstriction in a dose dependent fashion leading to myocardial ischemia
  • Acute left ventricular depression and heart failure at high blood concentrations
  • Supraventricular and ventricular arrhythmia (both through direct action or indirectly by producing myocardial ischemia)
  • Aortic rupture (rare)

CNS manifestations of cocaine intoxication include:

  • Psychomotor agitation
  • Headache
  • Seizure
  • Coma
  • Intracranial hemorrhage
  • Focal neurological symptoms

NOTE - Cocaine-induced psychomotor agitation can cause hyperthermia due to peripheral vasoconstriction that prevents the body from dissipating the heat generated from persistent agitation. Hyperthermia in the setting of cocaine intoxication can have a mortality of as high as 33%.

Crack cocaine is associated with upper and lower airway injuries because its use in requires high temperatures for evaporation. Such injuries are not caused by direct toxic effects. Cocaine use can also have a variety of effects on gastrointestinal tract, eyes and muscles.

In majority of cases with acute cocaine intoxication, benzodiazepines are generally sufficient to improve cardiovascular symptoms because most cardiovascular stimulation from cocaine is mediated by sympathetic nervous system. Benzodiazepines also help alleviate the agitation. Intravenous diazepam is the treatment of choice if the patient is not hypoxic or hypoglycemic.

However, in patients with severe, refractory, or symptomatic cocaine-induced hypertension other
antihypertensive medications can be tried. Phentolamine
, given as an intravenous bolus, is the treatment of choice, but other medications used for acute management of hypertension such as hydralazine can be used as well.

Aspirin is given to patients with pain associated with cocaine intoxication; after aortic dissection as potential cause has been excluded. As mentioned earlier, cocaine use is associated with enhanced thrombus formation.

Beta blockers should NOT be used in the treatment of cocaine-induced cardiovascular symptoms and complications owing to the fact that beta blockers may result in unopposed alpha-adrenergic stimulation, resulting in coronary vasoconstriction and end-organ ischemia. In very rare occasion when use of beta blockers is inevitable, phentolamine should be given beforehand to prevent unopposed alpha-adrenergic stimulation.

65
Q

A 21-year-old woma presents to your practice with complaints of anorexia, weight loss, loss of
interest in daily life, and a depressed mood for the past 3 weeks. Evaluation reveals the diagnosis
to be major depression. You advise that she take antidepressants as well as non-pharmacological
therapy, but she refuses to be treated. Which one of the following would be appropriate advice for
her?
A. Your symptoms will progressively increase until you inevitably commit suicide.
B. Your symptoms will not become better untill 6 months later.
C. Your symptoms will disappear spontaneously without medications in 5 years.
D. Only partial remission could be expected as the best outcome.
E. Your symptoms will remit in 2 months.

A

Correct Answer Is B
Without treatment, an episode of major depressive disorder in adults lasts from 6 months to 18
months (average 8 months). The course of the disease in adolescents appears to be different from
that of adults. Although there is no solid evidence, it is suggested that major depression in
adolescents may last from 2 weeks to many years.
Based on such data, the most appropriate advice for this patient is that without treatment she is
unlikely to recover until 6 months.
(Option A) Depression has the most association with suicide. Lifetime risk of suicide among
patients with untreated depression ranges from 2.2% to 15%. Most of depressed patients never
commit suicide.
(Option C) Most patients with major depression witll improve in 6 to 18 months even without
treatment. Telling the patient that her symptoms will last for 5 years is incorrect.
(Option D) Recovery from depression can be complete without residue; although the recurrence
rate is significant.
(Option E) It is unlikely that the patient’s symptoms remit in 2 months without treatment. The
minimum expected time for resolution of major depression is 6 months.

66
Q

During a car chase, a 24-year-old man hits a post lamp and sustains injuries. Accompanied by the
police, he is brought to the Emergency Department. According to the police, he has had multiple
occasions of violence, offense, bar fights, and vandalism, for which he has been arrested several
times during the past 5 years. He drinks alcohol, smokes marijuana, and sometimes uses cocaine.
Which one of the following pieces of information is most helpful in making a diagnosis?
A. History of non-accidental injuries.
B. History of trying to set fire on a neighbor’s car in adolescence.
C. Amphetamine use of his mother while she was pregnant with him.
D. Failure to response to methylphenidate prescribed for suspected diagnosis of ADHD in
childhood.
E. Truancy.

A

Correct Answer Is B
This scenario suggests antisocial personality disorder (ASPD) as the most likely diagnosis. ASPD
is characterized by a pattern of disregard for and violation of others’ rights, routine engagement in
illegal activities (e.g., drug use, assault, and theft), endangering the well-being of others, and
blatantly disregarding their legal rights. They also tend to be aggressive and impulsive and have
difficulty maintaining employment.
ASPD typically begins by mid-adolescence and continues into adulthood. Patients must be 18
years or older to fulfill the diagnostic criteria for the condition. Individuals with ASPD often have a
history of conduct disorder in adolescence presented by aggression towards people or animals,
destruction of property, deceitfulness or theft, or a serious violation of rules. Conduct disorder is
considered a warning sign for the development of ASPD in the future and helps with the diagnosis
in adulthood.
Of the options, the history of trying to set fire to a neighbor’s car is consistent with conduct
disorder in childhood as a risk factor and a warning sign for ASPD.
(Option A) Non-accidental injury in childhood is a significant cause of mortality and morbidity,
especially in infants and young children, and should raise suspicion of child abuse and neglect.
Child abuse increases the risk of developing antisocial personality disorder but is not essential for
establishing the diagnosis.
(Option C) Amphetamines are considered unsafe to use during pregnancy. However, there is no
evidence that in-utero exposure to amphetamines is a risk factor for ASPD. However, ASPD patients
commonly are noted to abuse amphetamine.
(Option D) Individuals diagnosed with ADHD during childhood are at greater risk for developing a
personality disorder in late adolescence. Specifically, studies found elevated rates of narcissistic,
paranoid, borderline, and antisocial personality disorders. Antisocial and paranoid personality
disorder appears to emerge primarily when ADHD is persistent.
(Option E) Some young people, who are not in school due to either truancy or exclusion, are
particularly vulnerable to drug misuse. Others at risk include those who have been in foster care
and those who are homeless. Moreover, people with antisocial personality disorder tend to start
abusing their substance of choice at an earlier age than those who do not have the disorder.
Truancy, however, is not a pointer towards ASPD or necessary to make such a diagnosis.

67
Q

A 55-year-old woman presents with insomnia for the past few days after she found out she has
breast cancer. She is a successful accountant and has enjoyed a healthy life. Although distressed
about the diagnosis, her mood is not significantly down. You recommend sleep hygiene and routine
as the most appropriate initial management of her sleep problem, but she insists that you prescribe
some medications to help her immediately. Which one of the following is the most appropriate
medication for her?
A. Temazepam.
B. Diazepam.
C. Fluoxetine.
D. Olanzapine.
E. Haloperidol.

A

Correct Answer Is A
Insomnia is a common problem in women with breast cancer with a prevalence of 23% to 61%.
Insomnia may present as difficulty initiating or maintaining sleep, or non-refreshing sleep. If
persistent enough, it can cause clinically significant distress or impairment in social, occupational,
or other important areas of functioning.
In approach to patients with insomnia, it is important to educate the patient about sleep hygiene,
which is encouraging habits that promote good sleep and avoiding those that may inhibit sleep.
Patients with anxiety can benefit from cognitive behavioral therapy (CBT). Behavioral therapeutic
approaches for insomnia seem particularly suitable to use in the breast cancer populations
because they have lower risk of interacting with the cancer treatment, do not burden the patients
with additional pharmacological treatments, and can target the treatment towards ameliorating
specific symptoms, like fatigue, that are characteristic in this population.
If such measures fail or the patient requests immediate relief, pharmacologic therapy is indicated.
Temazepam, zolpidem or zopiclone are first-line medications for such purpose. If they are
prescribed, the duration of therapy should be for the shortest time possible. The duration of use
should be determined and discussed with the patient. Also, patient should be fully informed of
potential adverse effects and chances of dependence.
This woman requests medication for immediate relief of insomnia, and of the options, temazepam
is the most appropriate one to consider for her.
(Option B) Diazepam is a long-acting benzodiazepine with unpredictable effects when used orally.
It is not recommended for management of insomnia.
(Options C) Fluoxetine and other SSRIs are used for pharmacological treatment of depression. This
woman, although distressed and anxious, is not depressed and does not need antidepressants.
SSRIs can cause sedation and improve sleep; however, they can induce insomnia as one of their
adverse effects, particularly early in the course of treatment.
(Options D and E) Haloperidol is a first-generation antipsychotic and olanzapine a secondgeneration one. This woman does not have psychotic features justifying the use of antipsychotics.
Haloperidole in particular and olanzapine has sedative effects, but should not be used for the
purpose of insomnia in the absent of other main indications to their use.

68
Q

Anne is a 65-year-old part-time nurse, who is reported for evaluation by her colleagues. For the past
12 months, she has been increasingly apathetic and has lost interest in her daily activities including
gardening that she has been doing for the past 40 years. Recently, she has started to accuse her
colleagues of stealing things from the ward every time something is lost. During the interview, she
has flat affect and reduced eye contact. She can answer to simple questions, but is unable to
process more complex ideas. She denies hallucinations, falls or movement disorders. Which one of
the following could be the most likely diagnosis?
A. Delusional misidentification syndrome.
B. Alzheimer disease.
C. First episode of psychosis.
D. Frontotemporal dementia.
E. Paranoid schizophrenia.

A

Correct Answer Is D
The clinical scenario is most consistent with dementia. Dementia is a progressive decline in
general cognitive function, with normal consciousness and attention. There is impairment of
memory, abstract thinking, judgment, verbal fluency and the ability to perform complex tasks. It is
associated with behavioral and psychological changes, and impairment of social and physical
functioning. Behavioral and psychological symptoms of dementia include psychosis, depression,
agitation, aggression and social disgrace.
Of the given options, frontotemporal dementia (Pick disease) is most consistent with the clinical
scenario.
Frontotemporal dementia is characterized by focal degeneration of the frontal and/or temporal
lobes. The typical age of onset is in the late 50s or early 60s, and the primary clinical
manifestations are changes in personality and social behavior or language, progressing over time
to a more global dementia. Other features include impaired initiation and planning, disinhibited
behavior and social disgrace and mild abnormalities on cognitive testing. Apathy and memory
deficits develop later in the course of the disease. A subset of patients may also exhibit symptoms
of extrapyramidal or motor neuron involvement at some point in the disease process.
(Option A) Delusional misidentification syndromes are rare psychopathologic phenomena that may
occur within the context of schizophrenia or affective or organic illnesses. They include Capgras
syndrome, Fregoli syndrome, intermetamorphosis syndrome, syndrome of subjective doubles,
mirrored self, delusional companions, and clonal pluralization of the self. Misidentification
syndromes show a great degree of overlap and do not represent distinctive syndromes. There is no
clue in the scenario to point towards delusional syndromes.
(Option B) Alzheimer disease is the most common cause of irreversible dementia and is
characterized by forgetfulness early in the course of the disease. Behavioral changes develop late
in the course. Anne’s presenting symptom is behavioral changes that are unlikely to develop early in
Alzheimer disease.
(Option C) Psychosis typically presents with hallucinations and delusions, the absence of which in
the scenario makes this diagnosis unlikely.
(Option E) Patients with paranoid type schizophrenia present with preoccupation with one or more
delusions and/or hallucinations, usually involving grandeur or persecution.

69
Q

Accompanied by his wife, a 63-year-old man presents to your practice for consultation. She is
concerned about his husband because he has been recently behaving childish and bizarre. Last
week he was dismissed from his job as a manager in a local restaurant, because of treating rude to
customers and shouting at his colleagues. He does not shave, bathe or change his clothes as he
did before and is disheveled and unkempt all the time. She denies any falls, gait abnormalities, or
hallucinations in her husband. His memory is not significantly affected. Which one of the following
could be the most likely diagnosis?
A. Depression.
B. Alzheimer disease.
C. Lewy body dementia.
D. Frontotemporal dementia.
E. Schizophrenia.

A

Correct Answer Is D
Cognitive and behavioral changes in aged people are frequently faced in general practice, with
dementia and delirium being the most common underlying etiologies.
Cognitive function is measured by various mental functions, including memory, concentration,
praxis, language, executive functions, and visuospatial skills. Dementia refers to memory loss with
impairment of any other cognitive function that can interfere with social or occupational
functioning.
A myriad of causes have been identified for dementia. These causes can be reversible or
irreversible.
The most common reversible causes of dementia include:
Hypothyroidism
Vitamin B12 deficiency
Hepatic or uremic encephalopathy
Vasculitides affecting CNS
Space occupying brain lesions i.e. abscess/tumors either primary or metastatic
Medications – anticholinergics in particular
Normal pressure hydrocephalus
Central or obstructive sleep apnea
Subdural hematoma
Trauma
Depression
Some of the most common irreversible causes of dementia are:
Alzheimer disease (60-80% of cases )
Vascular dementia including multi-infarct dementia and Binswanger disease
Lewy body dementia
Frontotemporal degeneration (dementia) including Pick disease
Multifocal leukoencephalopathy
The case scenario describes a patient with social inappropriateness as the most concerning
presenting symptom without memory being significantly involved. Of the options, the most
consistent one with such scenario is frontotemporal dementia (Pick disease). In this disease,
social disgrace is the earliest symptom with memory impairment and forgetfulness following later.
Frontotemporal dementia is characterized by focal degeneration of the frontal and/or temporal
lobes. The typical age of onset is in the late 50s or early 60s, and the primary initial clinical
manifestations are changes in personality and social behavior or language, progressing over time
to a more global dementia. Other features include impaired initiation and planning, disinhibited
behavior and social disgrace and mild abnormalities on cognitive testing. Apathy and memory
deficits develop later in the course of the disease. A subset of patients may also exhibit symptoms
of extrapyramidal or motor neuron involvement at some point in the disease process.
(Option A) Patients with depression may present with pseudodementia which is different from
dementia in some aspects. It is less common for patients with pseudodementia to have
disinhibition or social disgrace. The history of disturbances in pseudodementia is often short and
abrupt onset, while dementia is more insidious. On cognitive testing, people with pseudodementia
often answer that they do not know the answer to a question, and their attention and concentration
are intact and they may appear upset or distressed. Those with true dementia will often give wrong
answers, have poor attention and concentration, and appear indifferent or unconcerned.
(Option B) In Alzheimer disease, forgetfulness is usually the presenting symptom. It is very unlikely
for a patient with Alzheimer disease to present with disinhibition and social inappropriateness early
in the course of the disease.
(Option C) although misbehavior and disinhibition is a common early feature in patients with Lewy
body dementia, the absence of other manifestations such as fluctuating cognition, hallucinations,
extrapyramidal deficits (Parkinsonism) and repeat falls makes this diagnosis less likely.
(Option E) Psychotic features such as hallucinations and delusion are a significant diagnostic
component in schizophrenia that is absent here. Moreover, development of schizophrenia at this
age is unusual.

70
Q

An 84-year-old man is brought to the Emergency Department by his neighbors. He lives alone and
has developed forgetfulness and peculiar behavior recently. At night, the neighbors hear him
shouting and saying that somebody is trying to break into his house while there is no body there
when they come and check. This behavior started 2 months ago and has worsened progressively.
Which one of the following conditions is most likely to have caused this presentation?
A. Lewy body dementia.
B. Alzheimer disease.
C. Delusional disorder.
D. Schizophrenia.
E. Frontotemporal dementia.

A

Correct Answer Is A
Some features in the scenario are of diagnostic importance. Recent onset forgetfulness and
peculiar behavior and importantly visual hallucinations suggested by seeing people that do not
exist. These features suggest Lewy body dementia as the most consistent diagnosis among other
options.
Lewy body dementia is a progressive, degenerative dementia of unknown etiology, presenting with
dementia, motor signs, episodes of reduced cognition and visual hallucinations. Unlike Parkinson
disease, in Lewy body dementia, cognitive impairment and dementia precedes extrapyramidal
manifestations.
In summary, Lewy body dementia presents with the following:
Dementia and fluctuations in cognitive function with varying levels of alertness and attention
during a day
Episodes of long staring and disorganized speech
Well-formed, detailed Visual hallucinations (or less commonly, auditory)
Parkinsonian motor features
Early extrapyramidal features
Anterograde memory loss
Visuospatial impairment
(Option B) Although forgetfulness is a prominent feature and an early symptom in Alzheimer
disease, spontaneous motor Parkinsonism, hallucinations and peculiar behavior develops late in
the course of the disease, often in a matter of years.
(Option C) There is no false belief in the scenario suggesting a delusional disorder. The imagined
people entering his house are in fact visual hallucinations, not delusions. .
(Option D) Although hallucinations are a characteristic feature is schizophrenia and other psychotic
disorders, patients with schizophrenia usually have auditory rather than visual hallucinations.
Moreover, the age of the patient is unusual for development of schizophrenia.
(Option E) Despite the fact that early behavioral changes are a feature also seen in frontotemporal
dementia, the presence of visual hallucinations makes Lewy body dementia the more likely
diagnosis. Furthermore, forgetfulness is not a prominent feature early in the course of
frontotemporal dementia.

71
Q

A 71-year-old woman is brought to your practice for evaluation by her daughter. According to the daughter, she has developed progressive forgetfulness for the past 7 months. Recently, she frequently forgets days of the week and the date. Her past medical history is insignificant. She has not had any ischemic heart disease (IHD) or hypertension. On physical examination, she has a blood pressure of 110/65mmHg, pulse of 53 and regular, and a mini-mental status exam (MMSE) score of 14. You decide to start her on medications. Which one of the following would be the most appropriate option for her?

A. Donepezil.
B. Memantine.
C. Rivastigmine.
D. Haloperidol.
E. Galantamine.

A

Correct Answer Is B

This patient has classic presentation of Alzheimer disease, and an MMSE score of 14 indicates moderate dementia in this patient.
The maximum MMSE score is 30 points. A score of 20 to 24 suggests mild dementia, 13 to 20 suggests moderate dementia, and less than 12 indicates severe dementia. On average, the MMSE score of a person with Alzheimer disease declines 2 to 4 points per year.
Currently available pharmacological treatments for Alzheimer disease are cholinesterase inhibitors (e.g., donepezil, rivastigmine and galantamine) and memantine.

Patients with Alzheimer disease (AD) have reduced cerebral production of acetylcholine
transferase, resulting in decreased acetylcholine production and impaired cortical cholinergic function. Cholinesterase inhibitors increase cholinergic transmission by inhibiting cholinesterase at the synaptic cleft.
Cholinesterase inhibitors have a modest benefit to patients with dementia due to Alzheimer disease. These drugs improve alertness and function and maintain cognitive scores at or above the baseline for up to 12 months, but do not prevent disease progression.

Cholinesterase inhibitors are associated with prominent adverse effects including:
Gastrointestinal symptoms as the most prominent adverse effects, including anorexia,
nausea, vomiting and diarrhea;
Insomnia
Fatigue
Vivid dreams
Hallucinations
Asthma
Bradyarrhythmias and cardiac conduction abnormalities
hypertension
Dizziness
Cramps
Headache

Memantine is an N-methyl-D-aspartate (NMDA) receptor antagonist, with a distinct mechanism of action from that of cholinesterase inhibitors. It appears to be neuroprotective. Memantine may be
helpful in patients with moderate to severe dementia who are deteriorating despite initial response to a cholinesterase inhibitor (e.g., donepezil), or in previously untreated patients with moderate to severe dementia.

This patient has moderate dementia. She also has a baseline bradycardia of 52 bpm. Use of cholinesterase inhibitors for this patient, with pre-existing bradycardia, may result in profound bradycardia and cardiac complication; therefore, initiation of memantine in preference of a cholinesterase inhibitor would be the most appropriate treatment.

(Options A, C and E) Donepezil, rivastigmine, and galantamine can be associated with severe bradycardia in this patient who already has bradycardia.
(Option D) Haloperidol is not used for treatment of dementia.

72
Q

Which one of the following is the earliest feature of Alzheimer disease?
A. Impairment in visuospatial skills.
B. Forgetfulness.
C. Disorientation to familiar faces and objects.
D. Social inappropriateness.
E. Ataxia and motor function deficits.

A

Correct Answer Is B.
Memory impairment and forgetfulness is an essential feature of Alzheimer disease and often its
earliest manifestation. Even when not the primary complaint, memory deficits can be elicited in
most patients with Alzheimer disease at presentation. Recent memory is affected early in the
course of the disease while remote memory impairment develops later.
Memory impairment in Alzheimer disease has a distinctive pattern. It starts with memory
impairment for the time of facts and events. Loss or orientation to place follows. Recognizing
familiar faces becomes affected later, as does memory for vocabulary and concepts.
(Option A) Loss of visuospatial skills also occurs, but usually not before forgetfulness. It can be
very prominent at presentation. Visuospatial impairments manifest as misplacement of objects
and difficulty navigating in unfamiliar and then familiar places. A patient with Alzheimer disease
becomes easily lost in unfamiliar places and then familial places such as the neighborhood or even
at home.
(Option C) Visual agnosia that is inability to recognize objects and prosopagnosia, defined as
inability to recognize faces, are due to visuospatial skills. None of these features are common to
precede forgetfulness and memory impairment.
(Options D and E) Misbehavior, social disgrace and motor function deficits usually occur late in the
course of the disease.

73
Q

A 75-year-old woman is brought to your practice by her husband, who complains that she is not
herself anymore. The problem started with reduced self care and episodic disorientation and
forgetfulness. She refuses to bathe and barely changes her clothes. Recently, she has developed
difficulty walking and has had several falls. Last night, she started yelling and screaming because
she thought there are snakes slithering on the floor. On examination, she is a disheveled and
disoriented to time, place, and person. Fine resting tremors of her hands, mostly prominent on the
right, are noted. When she is asked to walk, she has difficulty starting and cannot stop when you
ask her to. Which one of the following is the most likely diagnosis?
A. Alzheimer disease.
B. Parkinson disease.
C. Schizophrenia.
D. Vascular dementia.
E. Lewy body dementia.

A

Correct Answer Is E
They key findings of cognitive impairment and unorganized behavior followed by deficits in motor
functions consistent with extrapyramidal involvement and visual hallucinations (snakes on the
floor), make Lewy body dementia the most likely diagnosis.
Lewy body dementia is a progressive, degenerative dementia of unknown etiology presenting with
dementia, motor signs, fluctuating cognition, and visual hallucinations. In Lewy body dementia,
cognitive impairment and dementia precedes extrapyramidal manifestations.
In summary, Lewy body dementia manifests with the following:
Dementia and fluctuations in cognitive function with varying levels of alertness and attention
Episodes of long staring and disorganized speech
Well-formed, detailed visual hallucinations
Early extrapyramidal features
Anterograde memory loss
Visuospatial impairment
(Option A) In Alzheimer disease, behavioral changes, hallucinations and motor deficits occur late in
the course of disease. In this woman this features have occurred early which is against Alzheimer
disease as a diagnosis.
(Option B) In Parkinson, memory and cognition remain intact until late in the course of the disease.
In Lewy body dementia, cognitive impairment and memory problems precede extrapyramidal
manifestations. On the other hand, visual hallucinations are not a feature of Parkinson disease.
(Option C) Schizophrenia is associated with hallucinations that are mostly auditory, delusions, and
other psychotic feature for at least 6 months. Patients with schizophrenia do not have motor
symptoms. In addition, the age of patient is unusual for development of schizophrenia.
(Option E) In vascular dementia, there is often a history of stroke or cerebrovascular disease. The
condition presents with focal neurological signs on examination, abrupt symptom onset related to
stroke in a step-wise fashion, and evidence of old infarcts on CT scan. Risk factors for
cardiovascular disease such as hypertension, diabetes, smoking, and atrial fibrillation are often
present. It is more common in men.

74
Q

A 50-year-old man with past medical history of bipolar disorder with psychotic features presents to
you, complaining of decreased mood and appetite for the past 2 weeks. On examination, you notice
that he does not talk much and only gives short answers to your questions. He seems to have
decreased concentration and avoid eye contact. He, however, denies hallucinations and paranoid
ideation. He is currently on olanzapine, which he has been taking for the past 2 years. Which one of
the following is the most appropriate initial management of this patient?
A. Continue olanzapine and add escitalopram.
B. Gradually decrease olanzapine and add escitalopram.
C. Increase the dose of olanzapine and wait for the effect.
D. Switch to venlafaxine.
E. Both A and C.

A

Correct Answer Is E
Many mood stabilizers (olanzapine here) have bimodal effect. They can be used for treatment of
both depression and mania. If a patient, who has successfully been stable on prophylactic dose of
a particular mood-stabilizer, develops acute depression, two approaches can be considered with
about the same efficacy:
Adding an antidepressant to the prophylactic mood stabilizer: the choices of the drug would
be the same as for major depression. SSRIs are first line options.
Increasing the dose of prophylactic mood stabilizer.
Either addtion of escitalopram or increasing the dose of olanzapine can be effectively used for this
patient.
NOTE - Cessation or dose reduction of a mood stabilzer can result in relapse of mania and is not
appropriate.

75
Q

A 79-year-old woman presents with tiredness and forgetfulness of recent onset. She scores 28/30
on a mini mental status exam (MMSE). Which one of the following is the most likely diagnosis?
A. Alzheimer disease.
B. Pick disease.
C. Anxiety.
D. Multi-infarct dementia.
E. Pseudodementia.

A

Correct Answer Is E
Mini-mental status exam (MMSE) is a practical test performed for initial assessment of cognitive
function. The total score is 30. An MMSE score≥25 makes dementia less likely, while scores less
than 25 suggest dementia.
Patients with dementia often have MMSE scores of less than 25. Alzheimer disease, Pick disease
(frontotemporal dementia) and multi-infarct (vascular dementia) are associated with genuine
dementia and decreased MMSE scores of less than 25. This patient has a MMSE score of 28 and is
very unlikely to be demented. For him, pseudodementia could be the most likely diagnosis.
Depressive pseudodementia is a term commonly used to describe a condition in a patient who
experiences a cognitive deficit secondary to a primary psychiatric disturbance including major
depressive disorder, bipolar disorder, schizophrenia, Ganser’s syndrome, and adjustment disorder.
Pseudodementia mimics dementia, and is usually reversible once the primary diagnosis has been
treated.
(Options A, B and D) While an MMSE score of <25 is likely in both dementia and pseudodementia, a
score of >25 almost excludes dementia. Alzheimer disease, Pick disease and multi-infarct
dementia are often associated with a marked decrease in MMSE score.
(Option C) Although tiredness can be seen in some patients with anxiety disorders, particularly in
those with generalized anxiety disorder, forgetfulness in not a common feature.
NOTE - The history of disturbances in pseudodementia is often short and of abrupt onset,
whereas dementia is often more insidious. Clinically, people with pseudodementia differ from
those with true dementia when their memory is tested. They typically answer that they do not
know the answer to a question, and their attention and concentration are often intact, and they
may appear upset or distressed when they fail to answer. Those with true dementia will give
wrong answers, have poor attention and concentration, and appear indifferent or unconcerned.

76
Q

Which one of the following is the method of choice in the management of borderline personality
disorder?
A. Cognitive behavior therapy.
B. Dialetical behavior therapy.
C. Exposure therapy.
D. Family therapy.
E. Interpersonal therapy.

A

Correct Answer Is B

Borderline personality disorder (BPD), a member of cluster B (dramatic, emotional) personality disorders, is characterized by a pattern of unstable interpersonal relationships and marked
impulsivity. Patients with this condition swing wildly between devaluing and idealizing others, labeling people as wholly good or wholly bad -a phenomenon known as ‘splitting’. Borderline
patients generally have difficulty calming down and controlling anger once they have become upset. Such patients tend to be unpredictable. As a result, they frequently have anger outbursts and engage in impulsive behavior such as substance abuse, overspending, binge eating, and suicidal or
self-mutilating behavior. Chronic feelings of emptiness are common.

The first step in management is by building a positive therapeutic relationship that plays a central role in the management of BPD and that helps identify and break the self-defeating interpersonal patterns characteristic of the previous relationships. Referral for longer-term psychotherapy with a psychiatrist and/or clinical psychologist is an important consideration in long-term treatment planning.

A specialized form of very intensive cognitive therapy, known as dialectical behavior therapy (DBT), is currently considered to be the treatment of choice for women (not approved for men or adolescents) with BPD for whom reduction in self-harm, anger, anxiety, or depression are treatment goals. DBT significantly reduces self-harm and substance use disorder. People aged 14–18 years with BPD or clinically significant features of BPD should be offered time-limited structured psychological therapies that are specifically designed for BPD. For adolescents, younger than 14 years with features of BPD, clinical psychological support and monitoring and involving their families are recommended.

Medicines should not be used as primary therapy for BPD, because they have only modest and inconsistent effects, and do not change the nature and course of the disorder. There is no current medication that is approved for the management of BPD, although there is some evidence that low-dose atypical antipsychotics and/or mood stabilizers may be effective in treating core symptoms.

(Option A) Cognitive behavior therapy was adapted for use with personality disorders, but the results are still preliminary. Manual-assisted cognitive therapy failed to reduce the repetition of self-harm in borderline personality disorder.

(Option C) Exposure therapy is one of the commonly used strategies of cognitive behavioral therapy (CBT). CBT has been found to be the most effective treatment for anxiety disorders. In exposure therapy, the psychologist guides the person through a series of real or imaginary scenarios to confront specific fears. Through a gradual process of exposure, the person learns to cope more effectively with these fears, and with practice, the anxious response naturally decreases.

(Option D) Family therapy is a type of psychological counseling (psychotherapy) that helps family members improve communication and resolve conflicts. A psychologist or licensed therapist usually provides the therapy. It helps family members solve family problems and conflicts, through improved communication, greater awareness of childhood and adolescence issues, and a better understanding of special family situations such as separation, divorce, remarriage, serious illness, or death in the family as well as mixed marriages in terms of culture, race, or religion.

(Option E) Interpersonal psychotherapy (IPT) is a time-limited, empirically validated treatment for mood disorders. IPT is an add-on therapy to antidepressant medication.

77
Q

A 23-year-old woman presents to your clinic asking for a medical certificate to give to her employer.
She says she was not able to go to work for the past two days because she had diarrhea and a
fever. You explain that you cannot do that because she has never been your patient and you can
make a false statement. Upon your refusal, she becomes irritated and starts shouting at you and
threatening you. You check the clinic records and realize that she was previously treated for a selfinflicted wrist cut in the same clinic. Which one of the following is the most likely diagnosis?
A. Malingering.
B. Narcissistic personality disorder.
C. Borderline personality disorder.
D. Histrionic personality disorder.
E. Munchhausen syndrome.

A

Correct Answer Is C
In this case, the woman seems to malinger to obtain a secondary gain. The term denotes the
deliberate simulation or exaggeration of symptoms for the purpose of obtaining some gain, such
as financial compensation. The presence of a personality disorder is a risk factor for malingering.
Malingering, her history of self-inflicted wrist cuts, and her behavior of difficulty controlling her
anger suggest borderline personality disorder as the most likely diagnosis.
Borderline personality disorder (BPD) is classified under cluster B (dramatic, emotional) personality
disorders, and is characterized by a pattern of unstable interpersonal relationships and marked
impulsivity. Patients with this condition swing wildly between devaluing and idealizing others,
labeling people as wholly good or wholly bad-a phenomenon popularly known as ‘splitting’.
Borderline patients generally have a hard time calming down, and they have difficulty controlling
anger once they have become upset, their moods tend to be unpredictable. As a result, they
frequently have anger outbursts and engage in impulsive behaviors such as substance abuse,
overspending, binge eating, or suicidal or self-mutilating behavior. Chronic feelings of emptiness
are common.
(Option A) Malingering is not a medical diagnosis but a description of behavior. DSM-5 describes
malingering as the intentional production of false or grossly exaggerated physical or psychological
problems. Motivation for malingering is usually external (e.g., avoiding military duty or work,
obtaining financial compensation, evading criminal prosecution, or obtaining drugs). In other
words, malingering is assuming a sick role for a planned gain.
(Option B) Narcissistic personality disorder is classified under cluster B (dramatic, emotional)
personality disorders, and is characterized by arrogance, grandiosity, a need for admiration, and a
tendency to exploit others. Individuals with the condition are preoccupied with fantasies about
success, power, brilliance, beauty, or the perfect mate. There is often a sense of excessive
entitlement. They usually demand special treatment.
(Option D) People with histrionic personality disorder are highly emotional and dramatic, have an
excessive need for attention and approval, and may be obsessed with their appearance. Sexual
seductiveness is another feature.
(Option E) Factitious disorder (Munchausen syndrome) is characterized by the intentional
production of false physical or psychological signs or symptoms to assume the sick role. Unlike
patients with malingering, those with the factitious disorder receive no secondary gain. It is called
Munchausen’s syndrome when patients display symptoms of the factitious disorder that are
predominantly physical in nature. Multiple tests usually show no abnormality, but they will not
accept that they do not have an illness. They may undergo numerous surgeries resulting in scarring
and adhesions.

78
Q

A 25-year-old man, who has history of agitations, multiple crimes, drug abuse, and theft, comes to
you with complaints of feeling sad ,losing interests, insomnia, fatigue, and loss of appetite. He says
that he is not using any drugs now but he has been recently charged with a minor assault. Which
one of the following is the most likely explanation to his behavior?
A. Antisocial personality disorder.
B. Malingering.
C. Bipolar disorder.
D. Substance abuse.
E. Conduct Disorder.

A

Correct Answer Is B
Apparently, this man has antisocial personality disorder (ASPD), which is characterized by a pattern
of disregard for and violation of the rights of others, which typically begins by middle adolescence
and continues into adulthood. Individuals with this condition are aged ≥18 years and have had
conduct disorder in childhood.
Individuals with ASPD routinely engage in illegal activities (e.g. drug use, assault, or theft),
endanger the well-being of others, and blatantly disregard others’ legal rights. They tend to be
aggressive and impulsive, and have difficulty maintaining employment. They frequently lie, and this
explains the malingering behavior in this young man to obtain secondary gain to provide the court
with.
DSM-IV defines malingering as ‘the intentional production of false or grossly exaggerated physical
or psychological symptoms, motivated by external incentives’. It stresses that clinicians should
suspect malingering when two or more of the following conditions are present:
The symptoms are reported within a forensic context
They sharply contrast with objective findings
There is lack of cooperation during diagnostic evaluation
The patient meets criteria for ASPD
The new edition of the DSM (i.e. the DSM-V) does not contain substantial revisions of how it
portrays malingering. Despite these features, however, affected individuals frequently display a
superficial charm that allows for the manipulation of others.
(Option A) Antisocial personality disorder is a psychiatric diagnosis and is considered a risk factor
for malingering. However, the question is asking about the behavior intention not the diagnosis.
(Option C) Bipolar disorder is a mood disorder characterized by periods of mania and depression
Manic episodes involve abnormally elevated mood, grandiosity, decreased need for sleep, racing
thoughts, talkativeness, and impulsivity (e.g. spending exorbitant amounts of money, gambling,
sexual indiscretion) This patient exhibits impulsivity and tension, which are specific to stealing. She
demonstrates none of the other concurrent manic symptoms required for a diagnosis of bipolar
disorder.
(Option D) While this patient denies substance abuse, patients do not always tell the truth. A urine
toxicology screen would be indicated if suspected. However, malingering is more likely here for
evading criminal prosecution not for obtaining a drug.
(Option E) Children and adolescents with conduct disorder often become adults with antisocial
personality disorder. The diagnosis of conduct disorder requires at least three symptoms from the
following:
Aggression towards people or animals
Destruction of property
Deceitfulness
Theft
A serious violation of rules
Adult patients have ASPD not conduct disorder, which is by definition a disorder of childhood and
adolescence.

79
Q

A 27-year-old woman presents to your office for evaluation. She left home when she was 12 and lived on streets where she started using alcohol and marijuana. Her criminal records show multiple arrests for shop-liftings and prostitution. During the interview, she is calm and communicative. When you ask if she has ever considered going back home, she answers I have nothing to do at home, and I am not going back there. Which one of the following is the most likely underlying cause of this history?

A. Borderline personality disorder.
B. Obsessive compulsive disorder (OCD).
C. Substance abuse.
D. Histrionic personality disorder.
E. Narcissistic personality disorder.

A

Correct Answer Is C
This woman has started abusing substances in an early age, and has multiple jail stays due to drug
issues and shop-lifting. At first glance, one might consider the diagnosis of antisocial personality
disorder; however, this woman does not fulfill criteria for such a diagnosis. Evidence of conduct
disorder with onset before age 15 is one of the criteria to make a diagnosis of antisocial
personality disorder. Although this woman has left home at an early age (<15 years) and has been
involved in drug use as well as criminal charges, these do not fulfill criteria for conduct disorder.
For conduct disorder to be present, one of the important criteria is the presence of at least 3 out of
the following 15 in the history:
1. Often bullies, threatens, or intimidates others.
2. Often initiates physical fights.
3. Has used a weapon that can cause serious physical harm to others (e.g., a bat, brick, broken
bottle, knife, gun).
4. Has been physically cruel to people.
5. Has been physically cruel to animals.
6. Has stolen while confronting a victim (e.g., mugging, purse snatching, extortion, armed
robbery).
7. Has forced someone into sexual activity.
8. Has deliberately engaged in fire setting with the intention of causing serious damage.
9. Has deliberately destroyed others’ property (other than by fire setting).
10. Has broken into someone else’s house, building, or car.
11. Often lies to obtain goods or favors or to avoid obligations (i.e., “cons” others).
12. Has stolen items of nontrivial value without confronting a victim (e.g. shoplifting, but without
breaking and entering; forgery).
13. Often stays out at night despite parental prohibitions, beginning before age 13 years.
14. Has run away from home overnight at least twice while living in the parental or parental
surrogate home, or once without returning for a lengthy period.
15. Is often truant from school, beginning before age 13 years.
This woman only has had 2 from the above criteria that are (1) leaving home early at age 12 and (2)
shop-lifting. So she did not have conduct disorder as an essential part for diagnosis of antisocial
personality disorder in adulthood. Moreover, oshe lacks other aspects of antisocial personality
disorder such as aggressiveness, recklessness towards self and others’ safety.
As a matter of fact, drug abuse appears to be the most likely cause of her presentation. Shop-lifting
and prostitution are the most common crimes committed to fund drug habits (economic related
crime).
(Option A) The two most common personality disorders associated with substance use disorders
are antisocial personality disorder and borderline personality disorder. This patient does not meet
the criteria for none.
(Option B) Obsessive-compulsive disorder (OCD) is characterized by obsessions or intrusive
thoughts that cause significant anxiety, or forced engagement in repetitive compulsions in an
attempt to avoid otherwise significant psychological stress. Patients with OCD often recognize the
unreasonable nature of their thoughts and behaviors, but are unable to stop them due to anxiety.
No points in the history favor OCD as a diagnosis.
(Option D) People with histrionic personality disorder are highly emotional and dramatic, have an
excessive need for attention and approval, and may be obsessed with their appearance. This
disorder usually begins by late teens or early 20s and seems to be caused by genetic
predisposition and early childhood events. This patient does not meet the criteria for histrionic
personality disorder.
(Option E) Narcissistic personality disorder is classified as a Cluster B (dramatic, emotional)
personality disorder and is characterized by arrogance, grandiosity, a need for admiration, and a
tendency to exploit others. They are preoccupied with fantasies about success, power, brilliance,
beauty or the perfect mate. Patients with this condition often have a sense of excessive
entitlement and may demand special treatment, Fifty percent of those with narcissistic personality
disorder abuse alcohol and drugs, with many facing addiction and dependency. This patient does
not meet the criteria for narcissistic personality disorder.
TOPIC REVIEW
Diagnosis of antisocial personality disorder:
According to the DSM-5, there are four diagnostic criterion, of which Criterion A has seven subfeatures.
(A) Disregard for and violation of others rights since age 15, as indicated by one of the seven sub
features:
1. Failure to obey laws and norms by engaging in behavior which results in criminal arrest, or
would warrant criminal arrest
2. Lying, deception, and manipulation, for profit or self-amusement
3. Impulsive behavior
4. Irritability and aggression, manifested as frequently assaults others, or engages in fighting
5. Blatantly disregards safety of self and others
6. A pattern of irresponsibility
7. Lack of remorse for actions
The other diagnostic Criterion are:
(B) The person is at least age 18
(C) Conduct disorder was present by history before age 15
(D) The antisocial behavior does not occur in the context of schizophrenia or bipolar disorder

80
Q

A 27-year-old woman presents to your practice with complaints about compromised interpersonal relationships. She has no friends and has been fired from work several times in the past few years.
She relates she could not make any friends at school as well and dropped school early. She also admits to a lack of concentration. Which one of the following could be the most likely diagnosis?

A. Borderline personality disorder.
B. Antisocial personality disorder.
C. Attention deficit - hyperactivity disorder (ADHD).
D. Depression.
E. Asperger syndrome.

A

Correct Answer Is A

Compromises in interpersonal relationships and the inability to hold a job are most consistent with the diagnosis of borderline personality disorder. Borderline personality disorder is classified under Cluster B (dramatic, emotional) personality disorders, characterized by a pattern of unstable interpersonal relationships and marked impulsivity. Patients with this condition swing wildly between devaluing and idealizing others, labeling people as wholly good or wholly bad - a phenomenon known as ‘splitting’.

Borderline patients generally have a hard time calming down, and they have difficulty controlling anger once they have become upset, their moods tend to be unpredictable. As a result, they frequently have anger outbursts and engage in impulsive behavior such as substance abuse, overspending, binge eating, or suicidal or self-mutilating behavior.
Chronic feelings of emptiness are common.

(Option B) Antisocial personality disorder is diagnosed in patients aged 18 years or older, who
engage in illegal activities (e.g., theft or assault) and disregard others’ rights. These individuals often display evidence of conduct disorder as minors.

(Option C) Although this woman admits to a lack of concentration, there is insufficient information to substantiate a diagnosis of attention deficit hyperactivity disorder (ADHD), characterized by
hyperactivity, short attention span, and easy distractibility.

(Option D) This patient has only one symptom of major depressive disorder which is ‘lack of concentration, but does not meet the full criteria including five or more of the following symptoms
for at least two weeks:
* Depressed mood
* Loss of interest
* Early-morning awakening
* Weight loss
* Low energy
* Impaired concentration
* Feelings of worthlessness
* Excessive guilt

(Option E) Patients with Asperger syndrome present in early childhood. It is four times more common in males. Patients with Asperger’s have impairments in reciprocal social interactions and restricted interests. They often desire relationships but lack awareness of social conventions, including the use of nonverbal communication. The absence of language delay (not a feature in Asperger syndrome) helps distinguish Asperger syndrome from autism.

81
Q

A 26-year-old accountant comes to your clinic complaining of flu-like symptoms for three days. He
arrives ten minutes late for his appointment but insists on being seen immediately. He says that he
has some very important matters to attend to and is, therefore, unable to wait. The receptionist
calmly informs him that the official policy is to see patients in order of arrival and that she will call
him when it is his turn. Upon hearing this, he flies into a rage and screams ‘How dare you! Don’t you
realize who I am?’ Given this presentation, which of the following is the most likely diagnosis?
A. Paranoid personality disorder.
B. Obsessive-compulsive personality disorder.
C. Antisocial personality disorder.
D. Narcissistic personality disorder.
E. Borderline personality disorder.

A

Correct Answer Is D
The clinical scenario is consistent with narcissistic personality disorder as the most likely
diagnosis.
DSM-5 criteria for narcissistic personality disorder include most of all of these features:
Grandiosity - an exaggerated sense of self-importance
Fixated on fantasies of power, success, intelligence, attractiveness, etc.
Self-perception of being unique, superior, and associated with high-status people and
institutions
Needing constant admiration from others
Sense of entitlement to special treatment and to obedience from others
Exploitative of others to achieve personal gain
Unwilling to empathize with others’ feelings, wishes, or needs
Being envious of others and believing others envy him
Pompous and arrogant demeanor
Behaving in an arrogant or haughty manner
(Option A) Paranoid personality disorder is characterized by distrust and fear. Individuals with this
disorder frequently have unfounded suspicions and misinterpret the motives of others. They may
find it difficult to confide in or forgive people.
(Option B) Patients with obsessive-compulsive personality disorder have a preoccupation with
orderliness and perfectionism that negatively impacts their functioning. They are often overly
devoted to their work, are stubborn, and believe there is only one way to do things. They can have
difficulty throwing out worn items, often have trouble with relationships because of their need for
perfection, and even need to play games by following the rules precisely. They often do not see
their behavior as a problem. Despite the similar names, There is no specific association between
OCD and any specific personality disorder, including obsessive-compulsive personality disorder.
(Option C) Antisocial personality disorder can be a diagnosis in patients aged 18 years or older,
who engage in illegal activities (e.g. theft or assault) and disregard others’ rights. These individuals
often display evidence of conduct disorder as minors.
(Option E) Individuals with borderline personality disorder show a pattern of instability in
relationships and marked impulsivity They swing wildly between scorning and idealizing people
(with others considered wholly good or wholly bad, a phenomenon known as “splitting”) They tend
to be impulsive and reckless and may demonstrate suicidal or self-mutilating behavior. Feelings of
anger and chronic emptiness are common.

82
Q

A young woman, who lives alone, comes to your practice with a complaint of insomnia. She left
high school to spend countless hours in her room practicing some supernatural medicine. She is
extremely fascinated by her home medicine and says: ‘I have some powers in the cure, I am willing
to discuss it happily only with my clients and those who are interested’. She has no social life or close
friends and does not enjoy attending parties or leaving home that much. Which one of the following
personality disorders she most likely has?
A. Paranoid personality disorder.
B. Obsessive-compulsive personality disorder.
C. Schizotypal personality disorder.
D. Schizoid personality disorder.
E. Avoidant personality disorder

A

Correct Answer Is C
Intentional reclusiveness, odd beliefs about supernatural issues, and lack of interpersonal
relationships make schizotypal personality disorder the most likely diagnosis.
Schizotypal personality disorder is characterized by a pattern of odd and eccentric behavior and
beliefs and a reduced capacity for close relationships. These patients usually exhibit ‘magical
thinking’. They may have bizarre fantasies or believe in telepathy, clairvoyance, or the concept of a
sixth sense. They often have paranoid ideation and unusual perceptual experiences.
(Option A) Paranoid personality disorder is characterized by a pervasive distrust and
suspiciousness of others in a way that their actions are interpreted as malevolent as indicated by:
reluctance to confide in others, preoccupation with unjustified doubts, suspicions that others are
being exploitative, and finding hidden meaning in benign remarks.
(Option B) Obsessive-compulsive personality disorder is characterized by a preoccupation with
orderliness and perfectionism that negatively impacts functioning. They are often overly devoted to
their work, are stubborn, and believe there is only one way to do things. They can have difficulty
throwing out worn-out and useless items, and often have trouble with relationships because of
their need for perfection. They often do not see their behavior as a problem.
(Option D) Schizoid personality disorder is characterized by the pervasive pattern of detachment
from social relationships and restricted emotions, as evident by four or more of the following
characteristics:
Lack of desire for close relationships
No interest in sexual experiences
Indifference to praise or criticism
Emotional coldness
Choosing solitary activities
NOTE - Schizoid personality disorder shares many features with schizotypal disorder. The
distinctive features are bizarre and eccentric appearance and beliefs which are present in
schizotypal personality disorder and absent in schizoid personality disorder.
(Option E) Avoidant personality disorder is characterized by hypersensitivity to criticism, social
inhibition, and feelings of inadequacy. Although these individuals want friendships, they avoid (as
the name suggests) such intimate relationships because they fear ridicule. They also perceive
themselves as inferior and are reluctant to engage in new activities or take risks for fear of being
embarrassed.

83
Q

Which one of the following is the most common cause of death in people with schizophrenia?
A. Cardiovascular disease.
B. Diabetes mellitus.
C. Suicide.
D. Smoking-related lung disease.
E. Accidents.

A

Correct Answer Is A
On average, people with schizophrenia have a lifespan 20 years shorter than the general
population. More than two-thirds of people with schizophrenia, compared with about a half in the
general population, die of coronary heart disease. This is partly due to factors such as smoking,
increased rates of diabetes, and metabolic problems caused by the use of some antipsychotic
medications. These factors often worsen once a cardiac condition arises because people with
schizophrenia are less likely to make the necessary lifestyle changes, such as diet and exercise, to
offset the problem.
Although death due to suicide is a contributing factor, the lifetime risk of suicide in schizophrenia is
usually quoted to be around 10%. However, overall, the evidence is for a lower rate (around 5%). The
highest risk is soon after diagnosis.

84
Q

A 54-year-old man is brought to the Emergency Department on a stretcher while accompanied by
the police. He is agitated and says: ‘I know you are taking me to the prison. I know that you will kill
me.’ Which one of the following conditions could be the most likely explanation to this
presentation?
A. Delirium.
B. Dementia.
C. Delusional disorder.
D. Drug abuse.
E. Paranoid personality disorder.

A

Correct Answer Is A
Of the options, delirium best explains the agitation and delusional state of mind in this man.
Delirium, also called acute confusional state, is a decline from a previously attained baseline level
of cognitive function due to organic causes. Delirium is characterized by a fluctuating course,
attentional deficits, and generalized severe disorganization of behavior. It typically involves other
cognitive deficits, changes in arousal (hyperactive, hypoactive, or mixed), perceptual deficits, and
psychotic features such as hallucinations and delusions that are often short-lived. Delirium itself is
not a disease, but a clinical syndrome.
(Option B) Dementia is characterized by impairment of memory and at least one other cognitive
domain such as aphasia, apraxia, agnosia, or executive function. These must represent a decline
from previous level of function and be severe enough to interfere with daily function and
independence. Although this patient may also have dementia, his current presentation of agitation
and delusions are acute and most likely due to delirium.
(Option C) Delusional disorder is an illness characterized by at least one month of delusions but no
other psychotic symptoms. Delusions are false beliefs based on incorrect inference about external
reality that persist despite the evidence to the contrary. These beliefs are not ordinarily accepted by
other members of the person’s culture or subculture.
(Option D) Drug abuse can cause delirium; however, it is an underlying cause of the condition, not a
diagnosis.
(Option E) Paranoid personality disorder is characterized by distrust and fear. Individuals with this
disorder frequently have unfounded suspicions and misinterpret the motives of others. They may
find it difficult to confide in or forgive people. Agitation, as in this man, is not a feature.

85
Q

Which one of the following is the most common anxiety disorder encountered in general practice?
A. Phobic disorders.
B. Post-traumatic stress disorder (PTSD).
C. Panic disorder.
D. Obsessive compulsive disorder (OCD).
E. Generalized anxiety disorder (GAD).

A

Correct Answer Is E
Anxiety disorders are one of the most common psychiatric disorders encountered in general
practice and at primary care level. In general, anxiety disorders are categorized into the following
six groups:
Panic disorder - sudden bursts of extreme anxiety that are accompanied by symptoms like a
pounding heart, sweaty palms, and shortness of breath or nausea
Agoraphobia - anxiety about being in places or situations from which it is difficult to escape if
a panic attack occurs
Social phobia (also called social anxiety disorder) - strong fear of social interaction or
performance situations because of the potential for embarrassment of humiliation
Generalized anxiety disorder (GAD) - long periods of uncontrollable worry about everyday
issues or events, which is typically accompanied by feelings of fatigue, restlessness or
difficulty in concentrating.
Posttraumatic stress disorder (PTSD) - recurrent and intrusive memories of a trauma, feelings
of emotional numbing and detachment, and increases in emotional arousal, such as
irritability and disturbed sleep, resulting from a previous traumatic event
Obsessive-compulsive disorder (OCD) - repeated thoughts, images or impulses that the
person feels are inappropriate, and repetitive behaviors, designed to reduce the anxiety
generated by the thoughts
Of the above disorders, generalized anxiety disorder (GAD) is the most common anxiety disorder
encountered in general practice and at primary care level. GAD is also the most common anxiety
disorder among older population.
Note that the question asks about the most common anxiety disorder faced in general practice
setting, not the most common anxiety disorder overall. According to the ‘Mental Health of
Australia’, a survey in 2007 showed that PTSD was the most common reported anxiety disorder in a
12-month period.
NOTE - An estimated 12% of all general practice encounters are related to mental health. Of
these mental conditions, depression is tne most common one (34%), followed by anxiety
disorders (14.8%)

86
Q

Michael, 35 years, has recently been started on fluoxetine for treatment of major depression
diagnosed based on findings of low mood, decreased sleep and appetite and loss of concentration.
Today he has presented because despite the fact that his depressive symptoms has satisfactorily
improved with treatment, he has been experiencing terrifying sleep disturbances. He explains that
he wakes up in the middle of the night and sees scary black figures around his bed. He wants to
get off the bed, but is paralyzed to do so. He cannot even move a finger, shout or call for help. The
whole experience is so frightening that he is afraid of going to sleep at all. He does not have such a
problem while awake. Which one of the following is the most appropriate management for him?
A. Reassure that this is normal and resolves with time.
B. Add an antipsychotic medication.
C. Advise that he uses a sleep mask during sleep.
D. Switch to another antidepressant.
E. Advise sleep hygiene and routine.

A

Correct Answer Is D
The sleep disturbance described is characteristic of sleep paralysis. Sleep paralysis is a sleep
disorder in which either during falling asleep or waking up, an individual temporarily experiences
inability to move, speak and react. Sleep paralysis is a transitional state between wakefulness and
sleep, characterized by an inability to move muscles, often accompanied by terrifying
hallucinations to which no reaction can be made due to paralysis. These hallucinations are often
described as a person or supernatural creature suffocating or terrifying the individual,
accompanied by a feeling of pressure on one’s chest and difficulty breathing. Another common
hallucination type involves intruders (human or supernatural) entering one’s room or lurking outside
one’s window, accompanied by a feeling of dread. Feelings such as being abducted by aliens have
also been reported.
The pathophysiology of sleep paralysis has been hypothesized to be a disrupted REM sleep that
normally induces complete muscle atonia to prevent sleepers from acting out their dreams.
Normally muscle tone is regained before the individual wakes up. In sleep paralysis the sequence is
reversed resulting in the person regains consciousness while muscles are still in atonic state.
Proposed risk factors for sleep paralysis (and other sleep disturbances) include sleep disruption,
traumatic life events, anxiety and depression and use of antidepressants.
This patient has developed sleep paralysis after he has been started on fluoxetine. Multiple studies
have shown a link between antidepressants use and sleep architecture disturbances. Interestingly,
selective serotonin reuptake inhibitors (SSRIs) have been used for treatment of some sleep
disorders including sleep paralysis, while they can also induce such disorders. Less so are
serotonin-norepinephrine reuptake inhibitors (SNRIs).
In this scenario, sleep paralysis has developed after starting the patient on the SSRI fluoxetine,
making the medication the likely cause of his problem. Considering this possibility, changing
fluoxetine with another antidepressant with less likelihood of inducing sleep disturbances can be
an appropriate management option.
(Option A) Sleep paralysis is usually a horrifying experience and the patient cannot be reassured.
No evidence suggests that the condition is temporary and resolves with time.
(Option B) Addition of an antipsychotic has not shown to be of proven benefit for management of
sleep paralysis.
(Options C and E) In sleep paralysis the problem is not going to or maintaining sleep. It often
occurs while the patient is falling asleep or is waking up; therefore, measures such as wearing a
sleep mask or sleep hygiene and routine are unlikely to be useful.

87
Q

A 45-year-old woman presents to your practice with complaints of sadness, tearfulness when
alone, and difficulty initiating and maintaining sleep for the past 2 months. She works as a parttime secretary and seems to have difficulties concentrating at work. Four weeks ago, she was
started on St. John’s wort tablets 300 mg, 8-hourly, by a naturopath; however, she has become even
more distracted, tearful and sleepless. Assessment establishes the diagnosis of major depression
but no suicidal ideation or attempts. Which one of the following is the most appropriate
management option?
A. Add lithium, 600mg daily, to augment the effect of St. John’s wort.
B. Add Omega 3, 5 g daily.
C. Stop St. John’s wort, wait for one week as the washout period, and start her on
venlafaxine 75 mg daily.
D. Increase the doe of St. John’s wort to 600 mg 8-hourly.
E. Continue the same dose of St. Johns wort and add paroxetine 20 mg, 12-hourly.

A

Correct Answer Is C
St. John’s wort (SJW) is a natural, herbal medicine that is reported to treat depression. It also has
been reported to be useful for treatment of premenstrual syndrome (PMS), menopausal symptoms,
anxiety, seasonal affective disorder (SAD), and even obsessive-compulsive disorder (OCD).
Therapeutic ingredients of SJW include hypericin, pseudohypericin and various xanthones.
Although the mechanism of action is not fully understood, it appears that the antidepressant
activity of SJW is through a combination of mechanisms including inhibition of serotonin,
norepinephrine and dopamine synaptic reuptake and via the neurotransmitters gammaaminobutyric acid (GABA) and glutamate.
Studies have shown that SJW can be as effective as standard antidepressants in treatment of mild
to moderate depression, but no more effective than placebo in severe depression. However,
patients are more compliant because of less adverse effects. Despite these facts, SWJ has not
been included in mainstream treatment guidelines due to uncertainty about appropriate dose,
persistence of effects, variations in the nature of preparations that makes the effects
unpredictable, and serious drug interactions. One major concern about SJW is its interactions with
other antidepressants, mood stabilizers, and anticonvulsants. Serotonin syndrome is the most
feared condition associated with such interactions. For this reason, concomitant use of St. John’s
wort with such drugs is not recommended and should be discouraged.
NOTE - Another concern is the fact that concomitant use of SWJ and oral contraceptives can
reduce the efficacy of the contraceptives because SWJ is a liver enzyme inducer and, through
this, increases the metabolism of oral contraceptives. It is recommended that concomitant use
be avoided.
This woman has been started on usual dose of SJW for depression (300mg, TDS), but her
symptoms have failed to improve. Under such circumstance, the most appropriate approach is
cessation of SJW and switching to a prescription antidepressant such as a selective serotonin
reuptake inhibitor (e.g., fluoxetine) or a serotonin-norepinephrine reuptake inhibitor
(e.g., venlafaxine). SJW should be withdrawn gradually and the prescription antidepressant started
after the washout period of SWJ, which is one week, to avoid the risk of serotonin syndrome.
(Option A) Lithium is a mood stabilizer used for treatment of bipolar disorder. It has no effect on
unipolar depression. Moreover, addition of lithium to SWJ is associated with increased risk of
serotonin syndrome.
(Option B) Some research has shown that omega 3 has a slight effect on major depression
compared to placebo. While this patient’s depressive symptoms have not improved with SJW, it is
very unlikely that addition of omega 3 adds any benefits.
(Option D) The recommended dose of SJW used for treatment of depression and mood symptoms
by manufacturers is 300mg, 8-hourly. Increasing the dose to 600 mg, 8-hourly neither is safe nor
likely to help this patient’s depression.
(Option E) Addition of the selective serotonin reuptake inhibitor (SSRI) paroxetine (or other
antidepressants) is associated with increased risk of serotonin syndrome and should be avoided.

88
Q

Ali, 45 years, suffers an ischemic stroke and is admitted to the hospital. He is started on life-long
aspirin 80 mg/day for prevention of stroke. He has no past history of a bleeding event such as
gastrointestinal bleeding and is currently on no other medications. During his hospital stay he is
noted to develop sign and symptoms consistent with diagnosis of depression. He is planned to be
started on cognitive behavior therapy (CBT) as well as an antidepressant medication. Of the following options, which one is most suitable to consider for Ali?

A. Citalopram.
B. Fluoxetine.
C. Venlafaxine.
D. Sertraline.
E. Amitriptyline.

A

Correct Answer Is B

Post-stroke depression (PSD) is the most common neuropsychiatric consequence of stroke up to 6-24 months after the stroke onset. Elderly patients are at risk in particular. When depressive symptoms develop in the first few days after the stroke, remission is more likely, whereas later
onset (i.e. after 3 months) is more likely to be associated with a prolonged course.

PSD is less likely to present with dysphoria and more frequently is associated with signs and symptoms of vegetative state.

Antidepressants are not generally indicated in mild PSD because the adverse effects outweigh the
benefits, but are used often in conjunction with cognitive behavior therapy (CBT) for treatment of
patients with moderate or severe PSD.

Selective serotonin reuptake inhibitors (SSRIs) are the first-line treatment options for pharmacological treatment of PSD. Of SSRIs, the drug with the most randomized controlled trials (RCTs) supporting its use in PSD is fluoxetine both in terms of tolerability and effectiveness is fluoxetine that is often safe at a dose of 20mg/day.

One concern with SSRIs, however, is the inhibitory effect of this drug class in the inhibition of
serotonin reuptake into the platelets that may result in bleeding tendency of bleeding and increased
incidence of gastrointestinal (most frequent), genitourinary, and intracranial hemorrhage.
This effect can be more pronounced when the patient is concomitantly on ulcerogenic medications
such as non-steroidal anti-inflammatory drugs (NSAIDs), aspirin and other antiplatelet medications,
corticosteroids, etc. However, a recent study suggest that SSRIs are associated with only a slight
increase in absolute risk of gastrointestinal bleeding with a number need to harm of 3177 in lowrisk and 881 in high-risk patients (the relative risk have been shown to be high, while the absolute
risk is small). Considering this fact and in general, such risk does not preclude SSRIs from being
used as first-line options for treatment of PSD.

(Option A) The SSRI citalopram is highly selective but less potent. It is the second-choice option for
treatment of PSD after fluoxetine.
(Option C) Venlafaxine is a serotine-norepinephrine reuptake inhibitor (SNRI). This drug family
have shown efficacy in treatment of depression as well increased bleeding tendency in few studies.
Whether this class of antidepressants be considered a treatment option for PSD remains
controversial.
(Option D) This SSRI sertraline is the second most studies medication for treatment of PSD. The
starting dose is 50mg/day with gradual increase up to 200mg/day if indicated and tolerated. It can
also be considered a treatment option but not superior to fluoxetine.
(Option E) Amitriptyline is a tricyclic antidepressant (TCA). SSRIs and tricyclic antidepressants are
equally effective in treatment of PSD; however, grade A recommendations are in favor of SSRIs as
the first-line treatment option due to better tolerability and their significantly lower adverse effects
potentials. TCAs are associate with anticholinergic effects such as hypotension, dry mouth,
constipation, urinary retention and confusion.
NOTES –
If SSRIs are considered a treatment option for PSD in patients with high risk of
gastrointestinal bleeding, proton pump inhibitors should be prescribed for prevention of
bleeding.
No evidence supports prophylactic use of antidepressants for prevention of PSD.
The effective duration of antidepressant therapy for PSD is 4 to 6 months with gradual
tapering if appropriate.

89
Q

Exposure and response prevention is the treatment option for which one of the following
conditions?
A. Bipolar disorder.
B. Depression.
C. Agoraphobia.
D. Obsessive – compulsive disorder (OCD).
E. Schizophrenia.

A

Correct Answer Is D
Exposure and response (or ritual) prevention (ERP) is the important and specific core element in
behavior therapy for obsessive-compulsive disorder (OCD).
OCD is characterized by distressing, intrusive obsessive thoughts and/or repetitive compulsive
physical or mental acts often in form of rituals.
The mainstays of treatment of OCD include the use of selective serotonin reuptake inhibitors
(SSRIs) or the TCA clomipramine, ERP, some forms of cognitive behavior therapy), education and
family interventions, and, in extremely refractory cases, neurosurgery.
In ERP, the situations the patients perceive as threatening are ranked and then the patient is
systematically exposed to symptoms triggers of gradually increasing intensity, and is taught to
suppress his or her usual ritualized response. This is generally challenging and often quite
distressing for the patient, but when effectively done, it promotes unlearning of the strong link that
has existed between having an urge and giving into the urge.

90
Q

Which one of the following criteria is essential to make a diagnosis of schizophrenia?
A. Flat affect.
B. Lack of insight.
C. Suicidal ideation.
D. Visual hallucinations.
E. Auditory hallucinations.

A

Correct Answer Is E
According to the Diagnostic and Statistical Manual of Mental Disorders, Fifth Edition, (DSM-5), to
meet the criteria for diagnosis of schizophrenia, the patient must have experienced at least 2 of the
following symptoms:
Delusions
Hallucinations – almost always auditory
Disorganized speech
Disorganized or catatonic behavior
Negative symptoms
At least 1 of the symptoms must be the presence of delusions, hallucinations, or disorganized
speech.
Continuous signs of the disturbance must persist for at least 6 months, during which the patient
must experience at least 1 month of active symptoms (or less if successfully treated), with social
or occupational deterioration problems occurring over a significant amount of time. These
problems must not be attributable to another condition.
Of the given options, only auditory hallucinations are a criterion for a diagnosis of schizophrenia.
(Option A) Flat affect is one of the negative symptoms that can be encountered in patients with
schizophrenia but its absence dose not exclude such diagnosis.
(Option B) Lack of insight is seen in schizophrenia as well as many other psychiatric conditions. It
is not a diagnostic criteria for schizophrenia.
(Option C) Suicidal ideation is a feature common in many psychiatric illnesses such as psychotic
disorders, mood disorders, etc. It is not characteristic of schizophrenia and a diagnostic criterion.
(Option D) Although hallucinations are one of the main criteria to establish a diagnosis of
schizophrenia, such hallucinations are almost always auditory. With visual hallucinations
alternative diagnoses should be considered first.

91
Q

In which one of the following periods, a pregnant woman is most likely to require treatment for
psychiatric problems?
A. First trimester of pregnancy.
B. Second trimester of pregnancy.
C. Third trimester of pregnancy.
D. Puerperium.
E. After 6 weeks postpartum.

A

Correct Answer Is D
There are a wide range of overwhelming negative emotions such as guilt, anxiety, confusion,
frustration, helplessness, sadness and positive emotions such as anticipation, fulfillment,
happiness and excitement that can be experienced during the pregnancy but more importantly and
most commonly experienced in the postpartum period. This makes the postpartum period a highly
vulnerable time for development of psychiatric disorders such as maternity blues, puerperal
psychosis, and postnatal depression.
Puerperium is the first 6 weeks after the delivery. The incidence of the above psychiatric conditions
as well as relapse of pre-existing conditions such as schizophrenia or bipolar disorder
necessitating active psychiatric treatment is highest during this critical period.

92
Q

You are assessing a 15-year-old girl for low weight. She has a body mass index (BMI) of 15. She
hates food because she think it makes her fat. She avoids eating as long as she can but when she
is urged to eat due to hunger, she induces vomiting or uses laxative to get rid of it. During the
interview you ask if she take part in any activity of her interest. She replies: ‘Why should I? What
does the entire world means? Nothing’. Which one of the following is more likely to be elicited from
the rest of the history?
A. Ideas of guilt.
B. Excessive socializing with friends.
C. Sleep disturbances.
D. Feelings of inferiority.
E. Fatigue.

A

Correct Answer Is D
This girl has characteristic features of anorexia nervosa (AN). AN is characterized by a markedly
low body weight, intense fear of weight gain, and body image distortion.
Although not central to the diagnostic criteria of AN, emerging evidence suggests additional
deficits in key aspects of social functioning. AN (and other eating disorders) patients appear to be
socially withdrawn, and they report having smaller social networks, less social interactions, and a
reduced number of close friends. There is also evidence for premorbid social problems such as
increased levels of loneliness, feelings of inferiority, and shyness, and comorbidity with anxiety
disorders, such as social phobia.
Of these, feelings of inferiority is most commonly cited by affected women during clinical
interviews. Interestingly, feelings of inferiority is one of the most important factors leading to the
development of eating disorders.
Ideas of guilt (option A) and sleep disturbances (option C) might as well present in the history but
not as commonly as feelings of inferiority.
(Option B) Patients with eating disorders including AN are often socially withdrawn, feel lonely and
may have no or few friends. Excessive socializing with friends is very unlikely to be present in the
history.
(Option E) Fatigue is common in AN patients due to a number of physical and psychological
factors. Low calorie intake, malnutrition and electrolyte disturbances can all lead to fatigue. This
girl may or may not be present in the history depending on the duration of the disease and severity
of malnutrition, electrolyte disturbances, or other complications of the disorder, but feelings of
inferiority are present in the past or current history.

93
Q

A 17-year-old boy presents to your office for assessment with complaint of excess distress and
anxiety because his college exams are starting. He admits to insomnia, looks anxious, distressed,
and restless. On mental examination, you realize that he adamantly believes the meat in his food is
poisonous and ‘they’ are trying to kill him. Which one of the following does he have?
A. Hallucinations.
B. Delusions.
C. Hypochondriasis.
D. Malingering.
E. Illusions.

A

Correct Answer Is B
The adamant belief that the meat in his food is poisonous is an example of delusional thinking.
Delusions are fixed false beliefs that are held despite contradictory evidence. Delusions are
considered “bizarre” if they are clearly implausible and peers within the same culture cannot
understand them. An example of a bizarre delusion is when an individual believes that his or her
brain is replaces with someone else’s. An example of a nonbizarre delusion is the belief that one is
under police surveillance, or like in this case the meat is poisonous despite lack of evidence.
(Option A) A hallucination is a sensation or sensory perception that a person experiences in the
absence of a relevant external stimulus. That is a person experiences something that does not
really exist except in their mind. A hallucination can occur in any sensory modality — visual,
auditory, olfactory, gustatory, or tactile.
(Option C) Hypochondriasis (recently changed to the term illness anxiety disorder [IAD]) is a strong
belief that one has a serious or life-threatening illness despite having no or only mild symptoms.
(Option D) Malingering is the purposeful production of falsely or grossly exaggerated physical or
psychological complaints with the goal of receiving a reward.
(Option E) Illusions are wrong or misinterpreted perception of sensations. Illusions can occur in
any sensory modality. The difference between a hallucination and an illusion is that in the former
there is an external stimulus which is misinterpreted or perceived differently, while for
hallucinations there is no external stimulus.

94
Q

A 28-year-old man with obsessive-compulsive disorder (OCD) has come for consultation. He has
already been started on clomipramine, and asks if there are other non-pharmacological treatment
options for him. In response to his question, which one of the following options would you consider
for him as a treatment for OCD?
A. Insight-oriented psychotherapy.
B. Explain to him that his compulsive behavior is a means of prevention from obsession -
related anxiety.
C. Exposure and response prevention therapy.
D. Psychoanalytic therapy.
E. Dialectic therapy.

A

Correct Answer Is C
OCD is a chronic illness that usually can be treated in an outpatient setting. The mainstays of
treatment of OCD include pharmacotherapy with clomipramine or selective serotonin re-uptake
inhibitors (SSRIs) or behavior therapy.
Exposure and response (or ritual) prevention is the important and specific behavior therapy for
OCD. In exposure and response prevention (ERP), the situations the patients perceive as
threatening are ranked and then the patient is systematically exposed to symptom triggers of
gradually increasing intensity and is taught to suppress his or her usual ritualized response. This is
generally challenging and often quite distressing for the patient, but when effectively done, it
promotes unlearning of the strong link that has existed between having an urge and giving into the
urge.
(Option A) Insight-orientation psychotherapy, also known as insight therapy, is a form of clientcentered therapy that guides patients towards developing an improved understanding of self.
During each session, the person receiving the therapy will recall situations from his/her life. The
therapist highlights observed patterns of behavior or feeling and encourages the patient to
examine them more closely. This method works through examination and analysis of issues that
have negatively affected the patient’s life, and helping the client understand and change destructive
patterns. Insight-orientation therapy is not an effective treatment for OCD.
(Option B) Patients with OCD are usually aware of and have adequate insight about their
bothersome urge for compulsive behavior as a means to relief the anxiety caused the obsessions.
Their problem is with the irresistible urge for such relief and not unawareness of their condition;
therefore, explaining why their compulsive behavior occurs will not help them as a treatment.
(Option D) Psychoanalytic therapy is used to bring unconscious or deeply buried thoughts and
feelings to the conscious mind so that repressed experiences and emotions, often from childhood,
can be brought to the surface and examined. Working together, the therapist and patient look at
how these repressed early memories have affected the patient’s thinking, behavior, and
relationships in adulthood. Psychoanalytic therapy is used for people suffering from depression,
emotional struggles, neurotic behavior, self-destructive behavior patterns, personality disorders or
relationship issues. Psychoanalytic therapy has not proved significantly effective in treatment of
OCD.
(Option E) Dialectic therapy, a very specialized form of intensive cognitive therapy, is currently
considered the treatment of choice for women with borderline personality disorder. This treatment
is not approved for treating male patients with borderline personality disorder. Dialectic therapy is
not effective for treatment of OCD.

95
Q

It is well-known that countertransference is frequently observed in doctors treating patients with
self-harm or anorexia nervosa. Which one of the following can be the cause?
A. These conditions are rarely seen.
B. These conditions are frequently seen in rich people.
C. These conditions are self-inflicted.
D. The doctor projects his/her inner feelings towards the patient.
E. These conditions are not socially accepted.

A

Correct Answer Is E
Countertransference is defined as the unconscious development of feelings toward the patient
from the treating doctor. Negative countertransference is when such feelings are negative and
prevent appropriate empathy and communication.
Studies suggest that countertransference is especially evident in the context of substance use,
self-harm and anorexia. For substance use and self-inflicted injuries there is often an underlying
undiagnosed borderline personality disorder as well as a comorbid condition. The pattern of
behavior in patients with this co-morbidity further deteriorates the condition and makes the patientdoctor relationship more difficult and delays access to appropriate treatment.
Recent research showed that the most important cause for the negative countertransference in
treating personalities who causing self-harm is stigmatism and not socially being accepted.
Stigma associated with patients with self-inflicted injury and harm (including anorexia nervosa) is
high and in healthcare setting reflects a lack of skills and knowledge in relation to the specific
needs of this population, as well as the negative countertransference commonly experienced in
working with these patients. Indeed, it has been suggested that individuals with BPD constitute the
‘most psychologically challenging patients a primary care physician ever encounters’, and this is
especially true when substance use is also prominent.
Working successfully with patients with BPD requires many layers of support in order to prevent
any negative countertransference adversely impacting treatment planning or the response to crisis
presentations.

96
Q

Jane, 75 years old, had a heart attack 18 months ago. She feels a little anxious, but isvery
independent and mobile. However, she ia afraid of leaving the house, because sometimes as she
thinks of leaving, she feels dizzy, unsteady, unable to breathe, heavy in the chest, clammy, and also
develops palpitation. These symptoms also occur unexpectedly at other times. These often last
approximately 10 minutes before she feels better, but she is really scared as to when they might
occur next. Which one of the following is the most likely diagnosis?
A. Anxiety.
B. Phobia.
C. Axiety with panic attacks.
D. Anxiety and ongoing angina pectoris.
E. Depression and phobia of going out

A

Correct Answer Is C
Jane has the classical symptoms of panic attacks with some pre-existing anxiety.
Panic attacks are defined as a period of intense fear with at least 4 of the following 13
symptoms:
1. Palpitations, pounding heart, or accelerated heart rate
2. Sweating
3. Trembling or shaking
4. Sense of shortness of breath or smothering
5. Feeling of choking
6. Chest pain or discomfort
7. Nausea or abdominal distress
8. Feeling dizzy, unsteady, lightheaded, or faint
9. Derealization or depersonalization (feeling detached from oneself)
10. Fear of losing control or going crazy
11. Fear of dying
12. Numbness or tingling sensations
13. Chills or hot flushes
Other describes symptoms include:
Headache
Cold hands
Diarrhea
Insomnia
Fatigue
Intrusive thoughts
Ruminations
Symptoms must peak within 10 min and usually subside within few minutes, leaving little to
observe, except the person’s fear of another terrifying panic attack.
Many people experience panic attacks without further episodes or complication. In some, however,
panic attacks progress to panic disorder characterized by repeated panic attacks, combined with
major changes in behavior or persistent anxiety of having further attacks.
According to the Diagnostic and Statistical Manual of Mental Disorders, Fifth edition, the following
criteria must be met in order to make a diagnosis of panic disorder:
Panic attacks must be associated with longer than 1 month of subsequent persistent worry
about:
1. Having another attack or complications of attack
2. Significant maladaptive behavioral changes related to the attacks
3. The attacks cannot directly or physiologically result from substance use (intoxication or
withdrawal), medical condition or another psychiatric disorder.
Agoraphobia was traditionally thought to involve a fear of public and open spaces. However, it is
now considered a complication of panic attacks. With agoraphobia, the person fears that they may
have an attack in a situation where escape would be difficult or embarrassing.

97
Q

A 16-year-old boy presents to your GP clinic with complaint of recent decrease in school
performance. He broken up with his girlfriend 3 weeks ago. He has been suffering from lack of
energy and sleep disturbances, and feels like crying almost all the time. Which one of the following
would be the most appropriate next step in management?
A. Empathy about the breakup.
B. Cognitive behavior therapy (CBT).
C. Start him on fluoxetine.
D. Start him on fluvoxamine.
E. Refer him to a psychiatrist

A

Correct Answer Is A
This young boy has presented with symptoms suggestive of depression after an identifiable
stressor (the breakup). He has the criteria for ‘adjustment disorder’ as the most likely diagnosis.
The DSM-5 diagnostic criteria for adjustment disorder are as follows:
Emotional or behavioral symptoms develop in response to an identifiable stressor or
stressors within 3 months of the onset of the stressor(s) plus either or both of (1) marked
distress that is out of proportion to the severity or intensity of the stressor, even when
external context and cultural factors that might influence symptom severity and presentation
are taken into account and/or (2) significant impairment in social, occupational, or other
areas of functioning.
The stress-related disturbance does not meet criteria for another mental disorder and is not
merely an exacerbation of a preexisting mental disorder.
The symptoms do not represent normal bereavement
After the termination of the stressor (or its consequences), the symptoms persist for no
longer than an additional 6 months
Royal College of Australian General Practitioners (RACGP) has proposed a model for interacting
with patients with psychological distress and disturbances regardless of the underlying etiology.
This approach model consists of three steps:
STEP 1- Establishing rapport using active listening and empathy, non-directional supportive
psychotherapy, and psycho-education. Then if required:
STEP 2- Cognitive behavioral therapy (CBT), interpersonal psychotherapy and interpersonal
counselling, structured problem solving, behavioral therapy, and family therapy. Then if
required:
STEP 3- Intensive therapy (e.g., use of medications)

See photo below.

For this patient establishing rapport in this session is a good start. Being empathetic about his
breakup and what he is going through is the most appropriate next step at this stage. This builds
therapeutic connection and builds up trust, and paves the path for further intervention.
CBT (option B) can be an option, among others, to consider for this patient in the STEP 2
afterwards.
Patients with depression in the setting of adjustment disorder usually respond to brief nonpharmacological interventions as mentioned above and do not require anti-depressants such as
fluoxetine (option C) and fluvoxamine (option D).
Referral to a psychiatrist (option E) is not often required for adjustment disorder. Even so, this
should be considered after establishing an appropriate therapeutic relationship with the patient.

98
Q

Monaro is a 19-year-old Aboriginal man who has presented to your practice for consultation. He has been in a correctional facility for the past 3 years for multiple felonies including street fights, battery, minor drug dealing and possession of illicit drugs, and has been released just few weeks ago. He has always been thinking about his past and childhood neglect and feels sad, angry, and frustrated about it. He recently has had anger outburst in confronting other people and is very worried that he might get himself into trouble and go back to jail again. He seeks advice for treatment. Which one of the following is the most appropriate treatment option for him?

A. Cognitive behavior therapy.
B. Dialectical behavior therapy.
C. Antidepressants.
D. Psychoanalytic therapy.
E. Anger management program.

A

Correct Answer Is C

Cognitive behavior therapy (CBT) (option A), dialectical therapy (option B), psychoanalytic therapy(option D), and anger management program (option E) are all therapeutic options with potential benefit for Monaro. For buried frustration from childhood, psychoanalytic therapy may be beneficial. He has spent 3 years in a correctional facility and is very likely to be suffering from post traumatic stress disorder (PTSD). His anger may respond properly to CBT and anger management program. These methods, however, take time to come to effect.

In the meanwhile, Monaro has the acute problem of anger outbursts that are very likely to put him or others at risk. It is recommended that for patients in immediate risk of harm to self or others, or escalation of the current situation, medical therapy is started as a means of rapid stabilization while other non-pharmacological measures are planned and carried out. This is often true regardless of the underlying etiology that can be PTSD, depression, substance abuse, etc.

For anger outbursts, antidepressants, especially selective serotonin reuptake inhibitors (SSRIs) have the highest supportive evidence for efficacy, and should be considered acutely for Monaro while other forms of therapy are arranged.

TOPIC REVIEW
A summary on CBT, DBT, psychoanalytic therapy, and anger management:

Cognitive behavioral therapy (CBT)
CBT is a form of psychological treatment that has proven effective for a range of problems including depression, anxiety disorders, alcohol and drug use problems, marital problems, eating disorders, and severe mental illness. Numerous research studies suggest that CBT leads to significant improvement in functioning and quality of life. In many studies, CBT has been demonstrated to be as effective as, or more effective than, other forms of psychological therapy or psychiatric medications.
Advances in CBT have been made on the basis of both research and clinical practice. Indeed, CBT is an approach for which there is ample scientific evidence showing that it actually produce change. In this manner, CBT differs from many other forms of psychological treatments.

CBT usually involves efforts to change thinking patterns. These strategies might include:
* Learning to recognize one’s distortions in thinking that are creating problems, and then to re-evaluate them in light of reality.
* Gaining a better understanding of the behavior and motivation of others.
* Using problem-solving skills to cope with difficult situations.
* Learning to develop a greater sense of confidence is one’s own abilities.

CBT treatment also usually involves efforts to change behavioral patterns.
These strategies might include:
* Facing one’s fears instead of avoiding them.
* Using role playing to prepare for potentially problematic interactions with others.
* Learning to calm one’s mind and relax one’s body.

Dialectical behavior therapy (DBT)
DBT is a type of talking therapy originally developed by an American psychologist named Marsha
Linehan. DBT is based on cognitive behavior therapy (CBT), but has been adapted to meet the particular needs of people who experience emotions very intensely. It is mainly used to treat problems associated with borderline personality disorder (BPD), such as:
* Repeated self-harming
* Suicide attempt
* Using alcohol or drugs to control emotions
* Eating problems, such as binge eating and purging
* Unstable relationships
The goal of DBT is to help patients learn to manage their difficult emotions by letting themselves
experience, recognize, and accept them. Then as they learn to accept and regulate their emotions,
they also become more able to change their harmful behavior. To help patient achieve this, DBT
therapists use a balance of acceptance and change techniques.

Psychoanalytic therapy
Psychoanalytic therapy is a form of in-depth talk therapy that aims to bring unconscious or deeply buried thoughts and feelings to the conscious mind so that repressed experiences and emotions, often from childhood, can be brought to the surface and examined.
Working together, the therapist and client look at how these repressed early memories have affected the client’s thinking, behavior, and relationships in adulthood. People with depression, emotional struggles, emotional trauma, neurotic behavior patterns, self-destructive behavior
patterns, personality disorders, or ongoing relationship issues, may benefit from psychoanalytic
therapy.

Anger management
Anger management is a psycho-therapeutic program for anger prevention and control. It has been
described as deploying anger successfully. Anger is frequently a result of frustration, or of feeling
blocked or thwarted from something the subject feels is important. Anger can also be a defensive response to underlying fear or feelings of vulnerability or powerlessness.
Anger management programs consider anger to be a motivation caused by an identifiable reason
which can be logically analyzed, and if suitable worked toward. Drug addiction, alcoholism, mental
disability, biochemical changes, and PTSD can all lead to a person committing an aggressive act
against another person.

99
Q

Janet and Li present to your practice for consultation. They met 6 months ago and now they have
decided to move in and live together. Li has a dog and loves it very much but Janet is severely
afraid of dogs. She says that I try to overcome my fear but even speaking of dogs makes me panic
to death. She says that when she was a child, she was bitten by a dog and has been afraid of them
her whole life. They ask for advice as to how to overcome her fear. Which one of the following is
the most appropriate advice she should follow as the first step in overcoming her fear of dogs?
A. She should keep a picture of a dog in her diary.
B. She should keep a picture of a dog in her room and at sight.
C. She should learn and perform relaxation techniques.
D. She should try to imagine patting a dog.
E. She should be exposed to dogs to overcome her fear.

A

Correct Answer Is C
Current guidelines recommend systematic desensitization for patients with specific phobias.
Systematic desensitization is a type of behavior therapy based on the principle of classical
conditioning.
This therapy aims to remove the fear response of a phobia, and substitute a relaxation response to
the conditional stimulus gradually using counter conditioning. There are three phases to the
treatment:
Phase 1 - The patient is taught a deep muscle relaxation technique and breathing exercises, e.g.,
control over breathing, muscle de-tensioning or meditation. This step is very important because of
reciprocal inhibition, where once response is inhibited because it is incompatible with another. In
the case of phobias, fears involves tension and tension is incompatible with relaxation.
Phase 2 - The patient creates a fear hierarchy starting at stimuli that create the least anxiety (fear)
and building up in stages to the most fear provoking images. The list is crucial as it provides a
structure for the therapy. The following table is an example rated by a patient: (see photo below)

Phase 3 - The patient works their way up the fear hierarchy, starting at the least unpleasant
stimulus (thinking about a dog) and practicing their relaxation technique as they go. When they feel
comfortable with this (they are no longer afraid) they move on to the next stage in the hierarchy. If
the client becomes upset they can return to an earlier stage and regain their relaxed state.
The client repeatedly imagines (or is confronted by) this situation until it fails to evoke any anxiety
at all, indicating that the therapy has been successful. This process is repeated while working
through all of the situations in the anxiety hierarchy until the most anxiety-provoking stimulus is
reached.
Based on this therapy, the first step in approaching Janet’s phobia would be teaching her relaxation
techniques and methods to overcome the anxiety brought up by exposure to the graduallyintensifying stimuli.
Other options are stimuli she should face after she learns how to overcome anxiety and relax in a
hierarchical fashion.

100
Q

A 24-year-old woman presents to your office extremely worried about her weight and body size. She
adamantly believes that she is obese and must do something for her obesity. On examination, she
has a BMI of 25 and there is no abnormal finding on the rest of the exam. Laboratory values are all
within normal range. Which one of the following is the most likely diagnosis?
A. Bulimia nervosa.
B. Body dysmorphic disorder.
C. Anorexia nervosa.
D. Hypochondriasis.
E. Generalized anxiety disorder.

A

Correct Answer Is B
This woman is presenting significant yet unrealistic concerns about and is preoccupied with her
weight and body size while she has a near healthy BMI (healthy BMI: 18.5-24.9 kg/m2). Also, there
is no pointers towards other conditions such as anorexia nervosa or bulimia nervosa. One
important diagnostic criterion for anorexia nervosa is a BMI of less than 17.5 kg/m2
. A BMI of 25
kg/m2
in this woman excludes anorexia nervosa (option C) as a diagnosis, and lack of binge eating
and purging in history makes bulimia nervosa (option A) very unlikely.
Given these, body dysmorphic disorder (BDD) is the most likely diagnosis in this scenario.
Specific DSM-5 criteria for BDD are as follows:
The individual is preoccupied with 1 or more perceived defects or flaws in physical
appearance that are not observable by or appear slight to others
At some point during the course of the disorder, the individual has performed repetitive
behaviors (e.g., mirror checking, excessive grooming, skin picking, or reassurance seeking) or
mental acts (e.g., comparing his or her appearance with that of others) in response to the
appearance concerns
The preoccupation causes clinically significant distress or impairment in social, occupational,
or other important areas of functioning
The appearance preoccupation cannot be better explained by concerns with body fat or
weight in an individual whose symptoms meet diagnostic criteria for an eating disorder
(option D) Hypochondriasis is associated with excessive and unduly concerns about a serious
disease. This woman does not have any concerns about a disease rather is obsessively
preoccupied with her physical appearance.
(Option E) Generalized anxiety disorder (GAD) is a disorder characterized by excessive or
unrealistic anxiety about two or more aspects of life such as work, social relationships, financial
issues, etc., often accompanied by physical symptoms of anxiety such as palpitation, shortness of
breath, chest pain, or dizziness. Although preoccupation with the weight can cause anxiety in this
woman, given the absence of worry in other aspects, GAD cannot be a diagnosis.

101
Q

Sid, a 47-year-old local farmer, presents to your practice for a visit due to problem sleeping due his
worries about his debts to the bank after a recent drought this year. During the consultation, you
get to know that his mood has been low in the past 2 months. He does not go out anymore and
stays at home most of the time because he lacks energy. He has poor appetite and lost almost 10
kg. Based on your diagnosis of severe depression, you advise him to be started on
antidepressants. He refuses and says: “Oh doctor. I am not depressed. This is what everybody
says. But I am just exhausted. You were too if in my place.” Which one of the following is the
defense mechanism he is using?
A. Displacement.
B. Reaction formation.
C. Projection.
D. Denial.
E. Rationalization.

A

Correct Answer Is D
This case scenario is a clear example of denial defense mechanism. Sid has a clear diagnosis of
severe depression, but he denies it and believes it is just the exhaustion and that he has just been
through too much.
Denial is one of the most common defense mechanisms. When a situation or fact becomes too
much to handle, the person copes by refusing to experience it. By denying reality, one essentially
protects himself from the need to face and deal with the unpleasant consequences and pain that
accompany acceptance.
Examples of denial are:
A smoker denies that his habit has negative health consequences so he can continue smoking.
A parent denies that her son has dropped out of college even though the school administrator
has left her three voicemails telling her so.
A business owner denies that her business is failing despite declining profits.
A problem drinker denies his alcohol dependence and believes he is just a social drinker.
Someone with failing marriage believes that every couple eventually loses the romance.
(Option A) Displacement occurs when a person represses affection, fear, or impulses that they feel
towards another person. Accepting that it is irrational or socially unacceptable to demonstrate
such feelings prevents them from being converted into actions. However, the feelings are instead
displaced towards a person or animal whom it is acceptable to express such feelings for. A person
who dislikes their teacher after being given low grades may feel that they would be punished if they
express their hostility towards them. Therefore, they may unconsciously displace their negative
emotions onto their best friend.
Examples of displacement defense mechanism are:
A woman who is frustrated because her boss made her work late again comes home and yells
at her son for asking what’s for dinner.
A man who is mad that he can’t seem to get ahead at work constantly criticizes his wife for not
making more money.
An employer who is frustrated with his business’ losses blames his employees instead of
rethinking his business strategy.
(Option B) With reaction formation defense mechanisms, one goes beyond denial and behaves in
the opposite way of which they think or feel. Typically, reaction formation is marked by a blatant
display. For example, a man with strong homosexual desires, vigorously and overtly reasons
against homosexuality.Other examples of reaction formation are:
A woman cares for her aging mother by cooking her meals, cleaning her home and taking her to
appointments while she secretly despises her.
A person with a drinking problem preaches about the virtues of sobriety and not drinking alcohol
instead of taking steps toward moderate drinking.
(Option C) When we experience feelings or desires that cause anxiety, or that we are unable to act
on owing to the negative impact that they would have on us or those around us, we may defend the
ego from resulting anxieties by projecting those ideas onto another person. A person who is afraid
of crossing a bridge with a friend might accuse them of having a fear of heights, for example, and
in doing so, avoids accepting their own weaknesses.
Examples of projection defense mechanism
You really dislike your manager at work who treats you just like they treat everyone else. Instead
of admitting your dislike, you tell everyone the manager has a grudge against you and is trying to
sabotage your career.
A woman is in a bad mood all day and comes home to her husband. After greeting her warmly
and asking about her day, she instantly accuses him of being in a bad mood and ruining her
evening.
A father has body image issues but refuses to make lifestyle changes that would help him lose
weight. Instead, he blames his daughter for her weight, projecting his lack of self-confidence
onto her.
(Option E) Rationalization occurs when a person attempts to explain or create excuses for an event
or action in rational terms. In doing so, they are able to avoid accepting the true cause or reason
resulting in the present situation.
Examples of rationalization defense mechanism are:
A man forgets to take out the garbage even though his wife has reminded him four times.
Instead of apologizing, he rationalizes that she hasn’t loaded the dishwasher so it’s only fair that
he not take out the garbage.
A woman stands up her friend for lunch and tells herself it’s okay because her friend is always
late for their morning yoga sessions.
A student rationalizes cheating on a test by saying the test was too demanding anyway.

102
Q

A 50-year-old farmer is in significant financial debts dues to a recent severe drought. He is planning to commit suicide so that his family can use his life insurance. He feels anxious and is irritable but does not show symptoms of depressive disorders. You know him well and consulted him for alcohol use and problem gambling and know, from his wife, that he is reckless around money, spends a lot in pubs and casinos, and has problem drinking issues. He blames the government and
authorities for what he is going through and believes that if they were more responsible and did their jobs right, he was not like this. Which one of the following defense mechanisms is he using?

A. Denial.
B. Projection.
C. Displacement.
D. Rationalization.
E. Regression.

A

Correct Answer Is B.
Base on the facts in the scenario, this man is most likely utilizing projection as a defense mechanism.

Projection is the misattribution of a person’s undesired thoughts, feelings, or impulses onto another
person who does not have those thoughts, feelings, or impulses. Projection is used especially when the thoughts are considered unacceptable for the person to express. For example, a man may be angry at their wife for not listening, when in fact it is the angry man who does not listen.

Projection is often the result of a lack of insight and acknowledgement of one’s own motivations and feelings.

In this scenario, the drought might really has caused his financial problems, but putting all the blame on the government and authorities and not accepting his share of irresponsibility helps him,
subconsciously, to misattribute the root cause of this feeling on someone or something else. In
other words, he is projecting the negative feelings of recklessness and irresponsibility elsewhere to
protect his ego. In projection, in fact, some negative feelings or perceptions are completely or
partially mirrored out.

Some other examples of projection are:
You really dislike your manager at work who treats you just like they treat everyone else. Instead of admitting your dislike, you tell everyone the manager has a grudge against you and is trying to sabotage your career.

A woman is in a bad mood all day and comes home to her husband. After greeting her warmly and asking about her day, she instantly accuses him of being in a bad mood and ruining her evening.

A father has body image issues but refuses to make lifestyle changes that would help him lose weight. Instead, he blames his daughter for her weight, projecting his lack of self-confidence onto her.

(Option A) Denial is one of the most common defense mechanisms. When a situation or fact becomes too much to handle, the person copes by refusing to experience it. By denying reality, one
essentially protects himself from the need to face and deal with the unpleasant consequences and pain that comes with acceptance.

Examples of denial are:
A smoker denies that his habit has negative health consequences so he can continue smoking.

A parent denies that her son has dropped out of college even though the school administrator has left her three voicemails telling her so.

A business owner denies that her business is failing despite declining profits.

A problem drinker denies his alcohol dependence and believes he is just a social drinker.

Someone with failing marriage believes that every couple eventually loses the romance.

(Option C) Displacement occurs when a person represses affection, fear, or impulses that they feel towards another person because he/she beleives it is irrational or socially unacceptable to
demonstrate such feelings. However, the feelings are instead displaced towards another person or animal. A person who dislikes their teacher after being given low grades may feel that they would
be punished if they express their hostility towards them. Therefore, they may unconsciously displace their negative emotions onto their best friend, making excuses for treating them badly without justification.

Examples of displacement defense mechanism are:

A woman who is frustrated because her boss made her work late again comes home and yells at her son for asking what’s for dinner.

A man who is mad that he can’t seem to get ahead at work constantly criticizes his wife for not
making more money.

An employer who is frustrated with his business’ losses blames his employees instead of rethinking his business strategy.

(Option D) Rationalization occurs when a person attempts to explain or create excuses for an event or action in rational terms. In doing so, they are able to avoid accepting the true cause or reason
resulting in the present situation.

Examples of rationalization defense mechanism are:

A man forgets to take out the garbage even though his wife has reminded him four times. Instead of apologizing, he rationalizes that she hasn’t loaded the dishwasher so it’s only fair that he not take out the garbage.

A woman stands up her friend for lunch and tells herself it’s okay because her friend is always late for their morning yoga sessions.

A student rationalizes cheating on a test by saying the test was too demanding anyway.

A shoplifter blames the high price of sweets to justify their theft of a chocolate bar, when in reality they simply enjoyed the act of shoplifting.

NOTE – The case scenario may be mistakenly interpreted as rationalization as he might sound
as if he is rationalizing his problems by something else which in this case is the government.

There is, however, a very fine differentiating factor between these two in this scenario; that is, he is attributing his exact recklessness and irresponsibility to an outer source. The key difference is
the ‘mirroring process’ which is specific to projection defense mechanism.

(Option E) Regression occurs when a person reverts to the types of behavior that they exhibited at
an earlier age. Stress of adult life and the associated anxiety may lead to a person seeking comfort in things which they associate with more secure, happier times. They might regress by eating meals that they were given as a child, watching old films or cartoons, acting without thought for the consequences of their actions.

Examples of regression are:

A 9-year-old boy is in the hospital to get his tonsils removed and begins sucking his thumb like he did when he was a toddler.

A woman gets in an argument with someone at work and starts sobbing uncontrollably.

Instead of utilizing mature parenting techniques like empathy and boundaries, a father resorts to temper tantrums to scare his children into obedience.

103
Q

A 75-year-old man is being assessed for insomnia. He has metastatic prostate cancer for which he
is taking morphine for pain control. Because of the sleeplessness, he had been using cannabis until
recently when it was no more effective. He feels agitated and edgy most of the time in daytime.
Which one of the following is most likely to help him with his problem?
A. Supportive psychotherapy.
B. Interpersonal therapy.
C. Sleep hygiene and routine.
D. Motivational therapy.
E. Syntonic phototherapy

A

Correct Answer Is C
Insomnia disorder is characterized by inadequate sleep despite adequate sleep opportunity,
accompanied by daytime dysfunction. Both pharmacologic and psychological interventions could
be used for treatment of insomnia disorder; however, selecting the right treatment depends on the
chronicity of symptoms, taking into account medical and psychiatric factors.
Current guidelines recommend cognitive behavioural therapy for insomnia (CBT-i) as the gold
standard and first-line treatment for insomnia. This may be complemented with short-term
pharmacological intervention.
The following diagram illustrates a stepwise approach to treatment of insomnia: see photo below.

The very first step in management of patients with insomnia is gathering a detailed sleep history
and treatment of any contributing factors and comorbidities. The second step is utilizing CBT-I.
CBT-I includes the following components:
Implementation of good sleep hygiene
Sleep restriction* and stimulus control programs, and relaxation procedures*
Cognitive therapy for insomnia
Sleep hygiene and routine is a term that encompasses most of the above and is the next best step
and in fact the most important arm of treatment for this patient. However, if CBT was an option, it
would be a more exact and appropriate one to choose.
Sleep hygiene and routing include the following pieces of advice for patients suffering from
insomnia:
Following a nightly routine:
Budgeting 30 minutes for winding down before sleep (soft music, light stretching, reading
and/or relaxation exercise
Dimming the light
Unplugging from electronics (e.g., TV, mobiles, tablets, etc.)
Relaxation techniques
Not tossing and turning in the bed and walking out of the bedroom if not fell sleep within 15-
30 minutes
Cultivation of healthy daily habits:
Adequate daytime light exposure
Day time regular exercise
Cutting down on alcohol
Cutting down on caffeine in the afternoon and evening hours
Smoking cessation
Avoiding late dining
Avoiding using the bed for anything but sleep and sex
Bedroom optimization:
Comfortable beddings
Setting a cool yet comfortable temperature
Blocking out noise and light in the bedroom
Using calming scents
(Option A) Supportive psychotherapy is a talking-based therapy designed to allow a person with
mental health issues to voice their concerns and receive encouragement and help in finding
practical solutions. Supportive psychotherapy is used to help people dealing with serious addiction,
eating disorders, stress, and other mental health issues. This method works to build a person’s
adaptability and resilience and better coping strategies for the future. Although this treatment
could be beneficial to treatment the underlying mental issues associated with or underlying
insomnia, it is not a recommended stand-alone therapy for it.
(Option B): Interpersonal therapy focuses on the interpersonal relationships of the depressed
person. The idea of interpersonal therapy is that depression can be treated by improving the
communication patterns and how people relate to others.
Techniques of interpersonal therapy include:
Identification of emotion — Helping the person identify what their emotion is and where it is
coming from.
Example - Roger is upset and fighting with his wife. Careful analysis in therapy reveals that he
has begun to feel neglected and unimportant since his wife started working outside the home.
Knowing that the relevant emotion is hurt and not anger, Roger can begin to address the
problem.
Expression of Emotion - This involves helping the person express their emotions in a healthy
way.
Example - When Roger feels neglected by his wife, he responds with anger and sarcasm. This in
turn leads his wife to react negatively. By expressing his hurt and his anxiety at no longer being
important in her life in a calm manner, Roger can now make it easier for his wife to react with
nurturance and reassurance.
Dealing with emotional baggage - Often, people bring unresolved issues from past
relationships to their present relationships. By looking at how these past relationships affect
their present mood and behavior, they are in a better position to be objective in their present
relationships.
Example — Growing up, Roger’s mother was not a nurturing woman. She was very involved in
community affairs and often put Roger’s needs on the back burner. When choosing a wife, Roger
subconsciously chose a woman who was very attentive and nurturing. While he agreed that the
family needed the increased income, he did not anticipate how his relationship with his own
mother would affect his reaction to his wife working outside the home.
(Option D) Motivational therapy/interviewing is a therapy technique that empowers patients to
make actionable behavioral changes in their lives through an interview-style discussion with a
licensed mental health professional. This technique focuses on change talk where patients
verbalize their need and desire to make changes in their lives.
Since the benefits of motivational interviewing for mental health are numerous, it has become a
common treatment method for people struggling with various mental health problems and cooccurring disorders like substance abuse. This therapy technique identifies and accepts the fact
that individuals who need to make changes in their lives approach counselling at different levels of
readiness to change their behavior. In some cases, patients may have thought about making a
behavior change but have not taken the necessary steps to make that change happen. During this
fragile time, people may express a fear of change and deal with internal struggles that need to be
broken down. This is where motivational interviewing steps in. A motivational interview allows
individuals to learn how to effectively change unhealthy behaviors to aid the addiction recovery
process.
(Option E) Syntonic phototherapy (light therapy) uses visible colored lights to enhance visual
attention and decrease the symptoms of stress and trauma. The goal of this therapy is to balance
the autonomic nervous system which controls visual perceptual fields. Colored light delivered
through the eyes helps control biological development and function. For example, red light
activates the sympathetic nervous system (‘fight-or-flight’ responses such as heart rate and blood
pressure) and blue/indigo stimulates the parasympathetic nervous system (‘rest and digest’
responses like salivation, urination, and even arousal).
Syntonic phototherapy consists of patients focusing their eyes at a dot of color at the end of a
50cm tube. Patients view specific safe light frequencies for approximately 20 minutes at a time
over a prescribed number of sessions (usually around 20, done over the course of 4-5 weeks). As
light enters the eyes, it travels from nerves connected to the retina to brain centers that influence
chemical, hormonal, and electrical balances. These centers control the balance of all body
functions, and years of study have proven that selected light frequencies delivered through the
retina can create favorable results that benefit the entire body. In addition to red and blue lights and
their systemic healing of the parasympathetic and sympathetic nervous systems, green lights
promote physiological balance. Once this balance is achieved, the effect is long lasting for most
individuals.
Because the practice affects the entire nervous system, the potential benefits of syntonic therapy
are numerous and varied. Syntonics can enhance visual acuity by improving blurred vision,
contrast, night vision issues, and light and allergy sensitivity. This therapy is also used to help
alleviate pain, headaches, eyestrain, and mental and emotional stress levels. Based on lack of
evidence for usefulness of this method for insomnia, it is not currently a recommended treatment
option for sleep problems.
*Sleep restriction – The aim of sleep restriction is to increase sleep drive and reduce the time awake
in bed. Time in bed must be aimed to align with the patient’s sleep duration. Gradually, more time is
spent in bed as sleep improves. Sleep restriction is typically administered by trained professionals and
requires close monitoring of daily sleep–wake patterns.
**Stimulus control – Behaviors that require wakefulness (e.g., watching TV, reading books) in bed
could result in bed being associated with hyperarousal, thereby perpetuating sleep difficulties.
Stimulus control helps to re-associate bed with being asleep. The patient is instructed to go to bed
only when sleepy. This helps re-associate bed with sleepiness. Activities in bed should be limited to
only sleeping and sex. The patient is instructed to leave the bed and bedroom if unable to fall sleep
within what feels like 15–30 minutes (without looking at the clock). During the time out, the patient is
instructed to complete a non-stimulating task, returning to bed when they feel comfortable. The same
morning wake-time is recommended even if sleep the night before is poor.
**Relaxation – Relaxation strategies can include progressive muscle relaxation and diaphragmatic
breathing. The goal is to release tension and arousal; however, close monitoring of patients is
important as relaxation may become sleep effort in disguise.

104
Q

In which one of the following situations a depressed patient has the highest risk of attempting
suicide?
A. When the depression deteriorates.
B. On commencement of treatment.
C. On improvement with treatment.
D. After discharge from the hospital.
E. On admission to the hospital.

A

Correct Answer Is D
Studies suggest that in patients with depression severe enough to mandate hospital admission, the
highest risk of suicide is within 4 weeks post-discharge after inpatient care of a depressed patient.
One study showed that 43% of those who were treated as inpatient for depression attempted
suicide within 1 month of discharge and 47% died before first appointment. Overall, the risk of
suicide within 4 weeks after psychiatric inpatient care is 100 higher compared to that of general
population.
A strong belief has long been held that during the early improvement phase after commencement
of treatment, the risk of suicide is high because the apathy symptoms resolve but depressed mood
remains; this believe, however, has been questioned according to recent studies.
NOTE – In general, suicide risk in depressed patients is highest when there is an initial
symptoms improvement brought about by treatment. In this period, the depressed patient is out
of the low mood and apathy and more likely to act on a suicide plan. However, admitted patients
are monitored and cared for during their stay and risk of suicide is lower compared to after
discharge when patient is more likely to feel left on his/her own and there is no ongoing support
or monitoring.

105
Q

A 35-year-old man is brought from prison to the Emergency Department by the police and prison
staff for assessment. He is doing time in prison. Recently, he has caused problems inside. He has
picked up fights with inmates over trivial issues and had several anger outbursts. He writes sanity
words on his cell walls such as ‘I’m all ok’, ‘beat it and keep it up’, ‘you are doing just fine’, etc. He
has mutilated himself several times and has cuts on both his wrists. According to the prison staff,
his condition has worsened progressively in the past few days. Which one of the following could be
the most likely?
A. Psychosis.
B. Schizophrenia.
C. Antisocial personality disorder.
D. Borderline personality disorder.
E. Factitious disorder.

A

Correct Answer Is D
At first glance, there might be diagnostic confusion because critical pieces of information in history
and mental status exam (MSE) are missing. The question, however, asks about the most likely
diagnosis based on the given information rather than a certain diagnosis.
There are some pieces of the puzzle that can be elicited from the scenario. This patient has been
having anger outbursts and fights over trivial issues that suggest ‘impulse control’ problems. He
has been self-mutilating, self-cutting, and refusing to eat or drink. Moreover, his sanity writings on
the cell walls indicates reassurance-seeking behavior. Since no one can reassure him, he is doing it
himself. These features make borderline personality disorder (BPD) the most likely explanation
with just this amount of available information (read the DSM 5 diagnostic criteria for BPD below).
This patient has been in imprisoned, and this could suggest antisocial personality disorder (ASPT)
as well. In ASPD there is a persistent display of impulsive, reckless, and aggressive behaviors and
no remorse about them. There is often a history of violent relationships, legal challenges, and
substance abuse if you have this personality disorder.
BPD and antisocial ASPD share some similarities. Both ASPT and BPD belong to cluster B
personality disorders (dramatic, emotional, and unpredictable behavior) and have disinhibited
behavior as a presentation. They both also have impulsivity in common. However, in ASPT the
person involves in violence and aggressiveness because they can (there is the element of control
and decision) while in BPD it happens because it cannot be controlled. Another differentiating
factor is that in ASPT the harm targets others while in BPD the harm is directed towards self. This
makes ASPD less likely of a diagnosis.
Psychosis (option A) and schizophrenia (as a psychotic disorder) (option B) are associated with
features such as impaired speech, auditory hallucinations, and delusional thinking, none of which is
present in the scenario. This makes them a less likely diagnosis.
Factitious disorder (option E) is a falsification of illness in oneself or another person (factitious by
proxy) without any obvious gain. By obvious gain, it is meant that the patient does not have a
known external motivation or achievement for producing the symptoms.
Features seen in factitious disorder are:
Inconsistencies between patient history and medical observations.
Vague details that seem plausible on the surface but that don’t hold up to scrutiny.
Lengthy medical records with multiple admissions at different hospitals.
Willingness to accept any discomfort and risk from many medical procedures, even surgery.
Overdramatic or outlandish presentation of a factitious illness, or hostility when challenged.
Although a detailed mental history might make this condition a diagnosis in this patient, later on,
the absence of the above features in the scenario makes such a diagnosis less likely compared to
BPD.
DSM-5 diagnostic criteria for BPD
A pervasive pattern of instability of interpersonal relationships, self-image and affects, and
marked impulsivity beginning by early adulthood and present in a variety of contexts, as indicated
by five (or more) or the following:
1. Frantic efforts to avoid real or imagined abandonment (Note: Do not include suicidal or selfmutilating behavior covered in Criterion 5)
2. A pattern of unstable and intense interpersonal relationships characterized by alternating
between extremes of idealization and devaluation
3. Identity disturbance: markedly and persistently unstable self-image or sense of self
4. Impulsivity in at least two areas that are potentially self-damaging (e.g. spending, sex,
substance abuse, reckless driving, binge eating) (Note: Do not include suicidal or selfmutilating behavior covered in Criterion 5)
5. Recurrent suicidal behavior, gestures, threats, or self-mutilating behavior
6. Affective instability due to a marked reactivity of mood (e.g., intense episodic dysphoria,
irritability , or anxiety usually lasting a few hours and only rarely more than a few days)
7. Chronic feelings of emptiness
8. Inappropriate, intense anger or difficulty controlling anger (e.g., frequent displays of temper,
constant anger, recurrent physical fights)
9. Transient, stress-related paranoid ideation or severe dissociative symptoms

106
Q

Which one of the following conditions is more likely to be associated with morbid jealousy?

A. Alcoholism.
B. Schizophrenia.
C. Bipolar disorder.
D. Depression.
E. Obsessive compulsive disorder.

A

Correct Answer Is B.

Jealousy a common yet complex emotion which has been well-recognized throughout mankind history. Jealousy is defined as ‘feeling or showing a resentful suspicion that one’s partner is attracted to or involved with someone else’. This definition indicates that it is a belief in the presence of rivalry that is the key issue and that whether or not such a rivalry truly exists is less important.

While normal jealousy is considered a normal human emotion, morbid jealousy is pathological. Morbid jealousy is not a diagnosis rather it describes a range of irrational thoughts and emotions that is accompanied by unacceptable or extreme behavior, in which the dominant theme is a preoccupation with a partner’s sexual unfaithfulness based on unfounded evidence.

The following features in morbid jealousy can distinguish it from normal jealousy:
* Excessive amount of time taken up by jealous concerns
* Difficulty in putting the concerns out of the mind
* Impairment of the relationship
* Limitation of the partner’s freedom
* Checking on the partner’s behavior

There are three forms of morbid jealousy: (1) delusional jealousy, (2) obsessional jealousy, and (3)
overvalued jealousy.

In delusional jealousy the characteristic feature is the presence of strong false belief of the partner being unfaithful (certainty) while in obsessive jealousy the affected individual suffers from unpleasant and irrational jealous rumination that the partner could be unfaithful (considering a possibility not in the form of a strong belief) which is followed by compulsive checking of the partner’s behavior. The difference between these two is the presence of a strong beliefs in unfaithfulness in delusional jealousy versus the bothersome lingering thoughts of the possibility of unfaithfulness.

Overvalued jealousy shared many features with obsessional jealousy. In overvalued jealousy, there is an acceptable and comprehensible idea which is excessively pursued by the patient beyond the bounds of reason. The main difference could be lack of compulsions and compulsive behavior in overvalued jealousy.

Morbid jealousy is not a diagnosis rather a comorbidity seen in many psychiatric conditions, personality disorders, and other settings including but not limited to drugs and alcohol abuse, psychotic disorders and schizophrenia, mania, depression, obsessive compulsive disorder (OCD), and anxiety disorders.

A recent study showed that of all psychiatric illnesses, schizophrenia is most likely to be associated with morbid jealousy, most commonly in form of delusional jealousy. The study concluded that depression (option D) could be the second most significant condition associated with morbid jealousy. Interestingly, all three forms of morbid jealousy were identified across the patients with depression in the study. Other conditions identified to be potentially associated with morbid jealousy in order of strength of such association were drug and alcohol abuse (option A), bipolar mood disorders (option C), and lastly other conditions such as anxiety disorders, OCD (option E), dementia and other.
The same study concluded that morbid jealousy in more common in men than woman, and also more common among married versus single men.

Treatment of morbid jealousy is with psychotherapy (e.g., CBT) and pharmacotherapy (e.g., SSRIs or antipsychotics depending on the underlying condition and diagnosis).

NOTE - Of the personality disorders, borderline personality disorder is most prone to morbid
jealousy.